You are on page 1of 187

A Management Paradise (un) Ltd .Endeavor.

I,MP

Management Of Cooperatives
Cooperation, does not demand that we Love Each Other, but it does demand
that we live with Mutual Tolerance for Each Other
Blinding ignorance does mislead us.
His labor's fruit a Holy wrath incurred.
Adorned in masters' loving art, She lies.
She rests at last beneath the starry skies.

It is not Where We Reach, but more importantly How we reach mattersGo on- 823 -.

A Management Paradise (un) Ltd .Endeavor.I,MP

Management Of Cooperatives..

The above Phrase contains the 2 most important terms of Times to come. These words have
powers to create a Revolution of any sorts

Well, The Following will explain the True Meaning of Each Of the Term.:
ManagementParadise.Com: A True Example of Co-operatives..
Just a Hope was needed to start MP.Com. Although it does have a Commercial Extension, it still
continues to be a glaring example of Cooperation. And it is not because we claim it to be
Cooperative, but the Members who constitute MP have that belief for it.

Imagine. If MP was non-existent, then how different the World would be. Would it that simple
to get access to the Projects, Papers? And even if that is unimaginable, MP today also does what
it was started forit stands as a Medium to Exchange Information. To attain this Macro Goal,
what we need is most importantly Cooperation in its Truest Sense. Sometimes it is Rightly said..
To Learn, is as good as to Implement
Reading this Book will get you a More Truer Meaning of Cooperation..
- Team MP
It is not Where We Reach, but more importantly How we reach mattersGo on- 824 -.

A Management Paradise (un) Ltd .Endeavor.I,MP

Management Of Cooperatives November, 2001


Section I
a. Spell out the three most important aspects of Cooperation?
Solution:
Co-operation touches everyones life, directly or indirectly. The house you may be staying in or
the shop you conduct your business from may be a co-operative society. The tea, which you
drink, has sugar and milk in it, in all possibilities the sugar has come from a co-operative sugar
factory and the milk from a Dairy Co-operative. The clothes that you wear may be from a cooperative Textile Mill.
Everybody needs co-operation; after all we need to co-operate to live!
Our society has developed towards a more self reliant one. Therefore the factor of selfregulation is not far. Any civilized society thrives on self regulation as to be civilized is a selfchosen path.
Co-operation offers the solution for the times. A completely democratic set-up with laws, which
borders on moderation.
The Co-Operative Approach Implies:
 Treating people as origins of action, not as objects to be manipulated or serviced
 Encouraging people to work together and help one another solve mutual problems
 Designing useful structures, processes, products and services so as to meet people's needs
rather than for profit-making purposes alone
The true messiah of the world is its belief in the principles of Cooperation.
The Ten Commandments, Message of Jesus, or The Holy Grail, by whatever name you call it,
cooperation has stood the test of time, from Formation to the Judgement day.

It is not Where We Reach, but more importantly How we reach mattersGo on- 825 -.

A Management Paradise (un) Ltd .Endeavor.I,MP


b. Briefly discuss the differences if any between the traditional approach and modern
approaches to cooperation?
Solution:
Man is a social animal. For him co-operation is a necessity rather than a choice. World over the
co-operative movement started in order to bring people together for a common cause.
Traditional approach:
Primitive concept of Co-operation
Right from the don of civilization, the concept of co-operation has existed. In a primitive
society, a mans life was simple with only a few wants. As the civilization grew, instead of an
individual, the villages emerged with a large number of farmers and a few carpenters, cobblers,
goldsmiths, etc. Co-operation was in those days a means to lead a life better. There was a mutual
aid and collaboration among different individuals and society. There is a mention of the cooperative movement in the Rig Veda and also co-operative bodies managed the temples in South
India.
Co-operation is not new to India, but the versions were different. In ancient India, it was in form
of 4 principles like the Kula, Gram, Sreni and Jati.
(Hierarchy based on units)

Kula Family

Jati Caste

Gram Village

Sreni Classification at different levels

The Kula was the 1st form of a co-operative activity; it was both a political and socio-economic
activity. As the society stabilized and expanded the aspects of economic and social co-operation
narrowed to the limits of joint family. Where the land is owned and cultivated commonly and
the adult members share the duties of the family and live under one roof. Co-operation at the
Gram (village) level emerged after the Kulas were stabilized. The gram sabha was a cooperative engine which undertook, the works for economic and social progress of the village
commons such as land, pastures, roads, highways, paths, common gardens, etc.
It is not Where We Reach, but more importantly How we reach mattersGo on- 826 -.

A Management Paradise (un) Ltd .Endeavor.I,MP


The artisans and cultivators often combine for the purpose of co-operative working and the
members jointly undertook, the use of selling of tools and implements of production. The Sreni
has been a development of the post Vedic era.
The co-operative and economic organization in this time was more of artisans, industrial and
handicrafts workers, merchants, traders, bankers, agriculturists, house builders, etc. The Sreni
worked basically as bankers and merchants guilds but also extended their role as charitable
institutions.
The co-operation at Jati level was mainly for achieving social goals such as education, charity,
and relief work.
Modern concept of Co-operation
The modern concept of co-operation is altogether different from the primitive one. It denotes a
special method of doing business. It is an off-shoot of the industrial revolution which is a name
given to the series of changes that came about in the method of production (18th Century) in
Europe, especially in Great Britain.
The end of 18th century, gave rise to capitalism. This resulted in concentrating the tools of
production in few hands, which gave them a complete control over the labor class, which was
although very large in number, but was poor and weak.
Dishonesty, capitalization, rising rates and black marketing, was the rule of the day to maximize
profits. The workers were exploited and were becoming poorer day by day. The Capitalist Class
in its greed to make more profits started exploiting the working class
like payment of low wages, increase in working of working hours, insecurity in employment etc.
This led to frustration and discontent among the working class. They therefore looked about for
an alternative to save themselves from the grip of inhuman employers. Few learned people came
forward to encourage the labor class and one of them was Robert Oven. Social reformists like
Robert Owen and Dr. William King, gave a thought to the formulation of a new philosophy. As
a result the co-operative movement began on the world stage.

It is not Where We Reach, but more importantly How we reach mattersGo on- 827 -.

A Management Paradise (un) Ltd .Endeavor.I,MP


c) Outline the main duties of a Secretary of a cooperative society?
Solution:
Duties of the Secretary
1. To fix the date and time of the meeting in consultation with the chairman or on requisition
from the director.
2. To prepare the agenda and notices and dispatch them to the directors.
3. Keep all documents and necessary statements available for the meeting.
4. Make necessary arrangements for the conduct of the meeting.
5. To take down the notes of the proceedings and finalize the minutes in the minutes book.
6. Carry out the orders and instructions of the board.

d) What are the important functions of the Registrar of Cooperative societies?


Solution:
The Registrar occupies crucial position in the co-operative system. He is the facilitator in the
implementation of the declared policy of the state-government towards rural development
through the co-operative sector. He is the most important media through which a democratic
government fulfills the social and economic aspirations of the people. He has also to coordinate between the government and the non-official leadership. He has also to maintain
administration realism and thus give correct advise to the popular government.
The Registrar of the co-operative society executes the co-operative law.


The powers in regard to Formation/Registration, to widening up of co-operatives remain


with the Registrar, while the financial help and other assistance is done by the co-operative
department.

The Registrar is assisted by the Assistant Registrar, District Registrar and other officers
appointed by the Co-operative Department.

It is not Where We Reach, but more importantly How we reach mattersGo on- 828 -.

A Management Paradise (un) Ltd .Endeavor.I,MP


 The powers and functions of the Registrar are prescribed by the respected Co-operative
Acts. The Government can also delegate some additional powers to the registrar for
promoting co-operative movements.

Functions of the Registrar can be divided into:


1.

Statutory Functions: The statutory functions of the registrar are to carry out duties
entrusted on him by the Maharashtra Co-operative Acts 1960. These are registration,
amendment of byelaws, audit of Accounts, execution of awards, liquidation process etc. In
certain conditions the registrar has the authority to suspend the management of the society
for its malpractices in accounts. He also has to see the day-to-day affairs of the society are
carried out without hindrances. He is also responsible for the settlement of disputes between
and within societies.

2.

Development and Promotional Functions: The co-operative movement has to play an


important role in the social and economic development of India. The registrar being a
facilitator and coordinator of the developmental process, he has to do various functions as a
friend, guide and philosopher in relation to co-operative society.
Criticisms on the Role of the Registrar
The Registrar is being criticized that instead of being a friend, philosopher and guide of
societies he has become the inspector general of co-operative police carrying out functions
like imposition of penalties, suspension and filling legal charges against societies.
Excessive control of Registrar has lead to ill effects like corruption, rivalry between
societies, high handedness, beaureacratic delays and red-tapism.

It is not Where We Reach, but more importantly How we reach mattersGo on- 829 -.

A Management Paradise (un) Ltd .Endeavor.I,MP


e) Enumerate the principal responsibilities of a cooperative towards its members?
Solution:
Education for its members
Since most members are poor and illiterate they are more or less unaware about the principles
and practices of co-operatives. Therefore, for an active participation it is necessary to evolve a
system, which will educate these members and keep in formed about the decisions and activities
of the society.
 Education for office bearers
The office bearers are the elected members of the co-operatives and as they are interested with
the responsibility to conduct the business on co-operative lines, it is necessary for them to
acquire the technical skill and thorough knowledge of co-operatives. Right from the elementary
level to the advanced level techniques.
 Education for prospective co-operative members
To bring in more participation from the general public it is necessary that the people should
have faith and for this it is the responsibility of the office bearers, to keep the general public
involved about the principles, aims, achievements and future plans of the co-operative
movement.
Co-operatives also provide training for their members, elected representatives, managers and
employees so that they contribute effectively to the development process. They also inform the
general public on the nature and benefits of the Co-operatives.
Self help and Mutual help:
As the co-operative enterprise is meant for the economic weak, they come together; form a cooperative, pool their resources, work together for mutual benefit and the benefit is shared by all
the participants.
Individually a person may not be capable to withstand the market forces and exploitation, but
together they can strengthen their position and pursue their economic interests.

It is not Where We Reach, but more importantly How we reach mattersGo on- 830 -.

A Management Paradise (un) Ltd .Endeavor.I,MP


EPICenter
"Experience is merely your interpretation of an event"

Let me tell you a story that may explain.

About 170 years ago Robert Collyer was a child working in a factory England and every morning
he was wakened by the harsh clanging of the factory bell.

Years later he emigrated to America where he became a preacher, scholar and poet of distinction,
and he often spoke of the melancholy sound of that bell. When he heard that the factory where he
had worked as a child was being demolished he wrote to a friend and asked if he could get a piece
of the bell as a memento. To his consternation he did not get a piece of the bell, he got the whole
bell.

What to do? He did not want the bell, but then he remembered that one of the departments of
Cornell University had no bell, so he offered it to them, warning that it "made the worst clangour
that ever vexed the heavens".

Well, the university accepted the gift and had the bell installed then invited Robert to ring it for
the first time. He went to pull the rope prepared to hear the harsh clanging he remembered from
his childhood. To his surprise the bell rang clear and sweet.

He realised that the memory was caused by his association with the sound. As a sleepy
overworked little boy he could find no pleasure in anything that caused him to wake up and go to
work.

Be prepared to accept that there is a possibility that you are wrong, that your interpretation of the
event may not be accurate. Realise that there will be different points of view, other people are
interpreting the event based on their emotional state and associations.

Be prepared to question the interpretation, ask questions like "What makes you say that? Why?"
"What makes you think that? Why?" "What made you do that? Why?"

It is not Where We Reach, but more importantly How we reach mattersGo on- 831 -.

A Management Paradise (un) Ltd .Endeavor.I,MP


Realize that "Experience is merely your interpretation of an event"

Case Study
The Adarsha Co-operative Housing Society Ltd. (the Society) was located in one of the suburbs
of Mumbai. The Society's premises housed a total of six buildings and had a total of 72 flats.
Established over 25 years ago, the Society had a fairly respectable group of people as it is they
were professionals, while the remaining few also lead rather hectic lives in their day to day
work.
Mr. Vishwas Joshi was the chairman of the Society. He had served the society in different ways.
Mr. Joshi had been a treasurer, the secretary and now he was the Chairman.
While the members of the Society were good-humoured and decent people, they were
somewhat indifferent, especially when it came to undertaking specific work for the Society. As
a rule, hardly anybody was interested in becoming a member of the Managing Committee (MC)
of the Society. Even those who were finally somewhat persuaded to become MC members, took
very little interest in actually running the society effectively.
This state of affairs of the Society made Mr. Joshi, basically a public-spirited man, somewhat
sceptical about his involvement with the Society. While the members of the Society did not
prefer to actively shoulder the responsibility of running the Society, they usually emerged as its
staunchest critics. Such criticism was seldom constructive. Over a period of time the true spirit
of cooperation appeared to have been lost sight of. The net result of all this was that nobody
was really concerned with how the society was being run.
Mr. Joshi was frankly tired of doing a thankless job. His patience also appeared to be running
out.

One fine morning he abruptly decided to put in his papers and resigned from the

Chairmanship of the society.

It is not Where We Reach, but more importantly How we reach mattersGo on- 832 -.

A Management Paradise (un) Ltd .Endeavor.I,MP

(a)

How would you account for the near lack of service motive amongst the members of

the Society?
Solution:
1.

The Society is basically located in Suburbs. It may be a mentality of the people that

doing work free of cost would earn them only name sake reputation, while they will be
spending their time, money and energy.
2.

It is said that Intelligent People think alike, in this case the people were trying to be

intelligent, but in the end they forgot that it is their own building.
3.

The lack of Interest amongst the members can be from the fact that Society functions as

per the bye-laws. That is the rules are quite rigid.


4.

It can also be that once you have a respectable post in the Managing Committee, you will

have to perform functions which may spoil personal relations. e.g. Collection of Dues from the
members is a tough job.
5.

The lack of Interest can also be accounted to maintaining different records like Minutes

etc. up to date and it has be shown whenever demanded.


6.

It can also be that the people think that they will have perform various tedious tasks like

paying municipality bills, sending maintenance bills etc. hence they want to stay away from all
these unnecessary jobs.
7.

There are also cases wherein members accuse the Managing Committee of withdrawing

money for personal use from Society accounts.


8.

Every body wants Friends, but nobody wants enemies. A single good deed by the

Managing Committee would be a prey of all the members proving to be best of the critics.
9.

It can also be said that as the job is non monetary, people dont want to work.

b) What steps would you suggest to inculcate the true spirit of cooperation among the
members?
Solution:
The following steps are to be taken to inculcate the spirit of cooperation amongst the members:
It is not Where We Reach, but more importantly How we reach mattersGo on- 833 -.

A Management Paradise (un) Ltd .Endeavor.I,MP


i) The members should develop a feeling of Patriotism for their Society. For this, they should be
made aware of the fact that all the great cooperatives were at one time in Primitive stage. Hence
one should sell them the dream of promising them a bright future about society.
ii) The members should not have any hard feelings amongst themselves. That is even if they
have, then they should be sorted amicably.
iii) There should be Get-Together function frequently organised. So that everybody is aware of
what is happening around him.
iv) That Society should implement the FISH philosophy. That is having fun while you work.
v) Assuming that certain days like Republic Day, Independence Days etc. are celebrated, they
should have luncheons wherein all the members get food from their places. Sharing would
inculcate the value of love and tolerance amongst the members.
vi) The society should frequently organise picnics, sports day etc., wherein all the members can
actively participate.
vii) Even if nobody is interested, still the Managing Committee should feel the passion and
proud of working for the Society.
They should act as Gardener for the Society.

c) Do you agree with Mr. Joshi's decision?


Solution:
Yes, for the time being it would be appropriate for Mr.Joshi to resign as the society members
need to be taught a lesson, to make them realize their responsibilities towards society.
But with the Current Scenario at hand, wherein nobodys interested, Mr. Joshi would be forced
to withdraw his resignation. If the situation demands so, he should promptly pull it back, but
should atleast convey the no-nonsense attitude to the members acting as critics.
After conveying the responsibilities towards society, still if there is no change in the members
attitude, then Mr. Joshi should put up the papers again, but this irrevocably
It is not Where We Reach, but more importantly How we reach mattersGo on- 834 -.

A Management Paradise (un) Ltd .Endeavor.I,MP

(d) How far would it be correct to argue that the problem faced by the Society is basically a
reflection of a much larger problem of disinterested members who constitute the first tier
of cooperation?
Solution:
Reference: See Q.3 Problems of Cooperatives.

Thoughts to Live/Work By.:

It is not Where We Reach, but more importantly How we reach mattersGo on- 835 -.

A Management Paradise (un) Ltd .Endeavor.I,MP


Section II

3) Discuss the main problems involved in effectively managing cooperatives in India.


Solution:
"A co-operative is a user owned and user-controlled business that distributes benefits on the
basis of use
The concept of co-operation and mutual help is not new to Indians, since ancient times Joint
family system has been a practical ideology for Indians. Principles of co-operation are the find
of Joint family system.
Co-operation is a movement of people. It is essentially an activity of the people for mutual help
and collective progress. It is fundamental right of a citizen to form an association. At the same
time voluntary membership is essence of Co-operation, Co-operative Society is not for earning
profits. Beneficiaries are the members, who work together and share together.
The International Labor Organization (ILO) points out that A Co-operative Society is an
association of persons, usually of limited means who have voluntarily joined together, to achieve
a common end, through the formation of a democratically controlled business organization,
making equitable contribution to the capital required, and accepting a fair share of risks and the
benefits of the undertakings.
According to International Co-operative Alliance A Co-operative is an autonomous
association of persons united to meet their common, economic, social, cultural needs and
aspirations through a jointly owned democratically controlled enterprise.
According to Lambert A Co-operative Society is an enterprise formed and directed by an
association of users, applying within itself the rules of democracy and directly intended to serve
both its own members and community as a whole.
Special Features of Co-operative Movement in India
1. It is a voluntary organization. It is an association of persons (one person one vote)
2. It is democratic, every individual has a say in the co-operative society.
It is not Where We Reach, but more importantly How we reach mattersGo on- 836 -.

A Management Paradise (un) Ltd .Endeavor.I,MP


3. It is an undertaking
4. Less stress on Profits because the main objective is to meet common economic ends through
mutual help.
5. It is not based on Exploitation
6. There is no place for middlemen
7. Common objective of people
8. Concept of equality
9. Wider interest of Community
10. No outside interference
11. Each for every one (self help through mutual help)

Problems In Effectively Managing Co-operatives


Inspite of a historical record of co-op movement, still we face problems which has hampered the
development.
1.

Government influence: Recent years have shown an increasing interference of the


government by imposing restrictions, which has seriously hampered the democratic function.
Further the corrupted political ideas have taken co-operatives as their personal kingdoms,
which have seriously hampered the purpose of co-operative movement. Government has
practically influenced the co-operative movement as the major parts of the financial
assistance still come from the government and therefore it has not really reached the peoples
movement.

2. Political Influence: The political influence and pressure on officials in day to day function
of granting of loans to undeserving people, corruption amongst officials is sickening the cooperatives. The political interference and administrative high handedness has been the root
cause for loosing the purpose. Further growing vested interests, ineffective federal structure
and reliance on government shows the dependency which is coupled with the processing
It is not Where We Reach, but more importantly How we reach mattersGo on- 837 -.

A Management Paradise (un) Ltd .Endeavor.I,MP


handled by several departments, lack of co-ordination and export guidance have together
arrested and influence co-operatives.
3. Bureaucrats on deputation: Appointment or deputation of government officials has proved
to be harmful as well as anti-democratic. Further these bureaucrats are more rules bound
and less business like in order to complete the procedures they sacrifice the business
opportunity. The expenditure incurred for these bureaucrats are much more an acts of
financial burden because equally qualified people can be recruited at much lesser cost from
the open market.
4. Problems of deduction: A pathetic state of socio-economic structure, poverty and illiteracy
has been a reason due to which the members fail to elect a leader who is wise, efficient and
capable and therefore a leader from outside becomes the only choice. The co-operative
movement has concentrated more on providing a rather allied infrastructure, which would
have made the co-operative movement more profitable and served the fundamentals purpose
of economic upliftment of the members. It was recommended by All India Rural Credit
Survey report of 1954 that it is necessary to have an integrated scheme of co-operative
development rather than merely of co-operatives.
5.

Confined to Credit Co-operatives: The Co-operative Credit Society has been providing
loans only for agricultural operations and did not extend loans to meet the other
requirements of the farmer because of which they had to depend on the moneylenders and
the resulting exploitation thus it could not be achieved.

6. Small size: The primary agricultural co-operatives have been very small with a limited area
of operation but the liability has been unlimited. Also the honorary workers were expected to
work much more which adversely affected the successful working of co-operatives.
7. Management in wrong hands: Management of most of the co-operatives has been in the
hands of the illiterate, ignorant and untrained people who were unfit for the work entrusted
on them.
8. Conflict within societies: Intra society conflicts have also been a serious retarding factor.
Under these circumstances a society will either not do much work or favoritism and
hypnotism would play the role in controlling the funds and in the process restricting the
funds to few favored members.
It is not Where We Reach, but more importantly How we reach mattersGo on- 838 -.

A Management Paradise (un) Ltd .Endeavor.I,MP


9. Excessive overdues: A major problem of many societies is a high percentage of outstanding
and low recovery rates.
10. Unproductive loans: Large loans have been given for unproductive purposes and that too
the people who have concealed or hidden their own debts. The management has also been
reluctant to take action against the defaulters.
11. Window Dressing: To show a lesser figure of the overdue often the accounts are
manipulated and made up which is possible due to a defective audit. Loans are shown to
have been paid and new loans are issued thus claiming a good rate of recovery. Defective
audit, inspection, embezzlement of funds and dishonest management are common problems.
12. Lack of funds: The main problem of the co-operatives is lack of funds. This is because the
members do not save or deposit enough of their savings to contribute to the working capital
of the organisation. Besides, there is also lack of response from general public. Hence, most
co-operatives organisations fail to attract the desired deposits.
13. Lack of professionalism: The co-operative sector is still an underdeveloped sector of the
Indian economy. Under the 5-year plans, keeping in mind the ultimate socio-economic
goals, appropriately enough with state recognition guidance a number of co-operatives were
set up. While the quantitative coverage on account of government support, one can come
across very few well-managed, large-sized and modern co-operatives. Due to the lack of
professionalism, growth and development is hampered in the face of the changing
management scenario in the country.
14. Problem of Member Involvement: Many co-operatives have a problem of lack of member
involvement. This is because of the indifferent attitudes of the members. Also the principle
of unlimited liability prevents people from joining co-operative organisations. Hence, cooperatives organisations fail to involve the desired the number of members.
15. Illiteracy: In India, people have been highly illiterate, ignorant and extremely conservative
towards co-operative organizations.

This is because people fail to understand the real

meaning and objectives of co-operation.


16. Dual Control: Let's take an example to explain this point.

The Registrar, who is a

representative of the state government and also the Reserve Bank of India, controls cooperative banks. Similarly all other co-operatives have dual control. There is no one fixed
It is not Where We Reach, but more importantly How we reach mattersGo on- 839 -.

A Management Paradise (un) Ltd .Endeavor.I,MP


line of hierarchy. This is a problem because if one employee has two bosses, he has twice the
problems.
17. Problem of Diversification: It must be approved that co-operatives in India exhibit
excessive sensitivity towards memberships and other aspects of the organisation and pay
little attention to exploring new areas of diversification. This is a serious problem.
18. Traditional society: This is another problem for co-operatives. This is because most of the
villages or small towns where co-operatives operate have members who are very traditional
and conservative. Even in the competitive scene in India, it is becoming more and more
difficult for co-operative to function efficiently on the basis of traditional type of
management.
19. Over-riding powers to Registrars: This is again a very serious problem for co-operatives.
The registrar being a representative of the State Government has been given a lot of powers.
Due to rising corruption, these powers are misused to a large extent, thus posing as a
problem.
Other problems include:
1. Dependence on external sources
2. Conforming financial assistance through few rich due to favoritism
3. Unemployment
4. Poverty
5. Lack of Infrastructure
6. Unequal distribution of wealth and resources
7. Low per capita income
8. Casteism
9. Religion fundamentalism
10. Linguistic Conflicts
It is not Where We Reach, but more importantly How we reach mattersGo on- 840 -.

A Management Paradise (un) Ltd .Endeavor.I,MP


11. Regionalism
12. Lack of women empowerment
13. Population Explosion
14. Corruption

4) What are the important qualities of an effective leader of a cooperative movement?


Illustrate your example with suitable examples.
Solution: See paper 2004
Examples:
Profile of a Cooperative leader: Dr. Verghese Kurien
Back in 1949, a young man from the lush green state of Kerala decided to take up the
unfashionable task of organizing milk production and all that it involved at the other end of the
country. Anand in Gujarat, the town which Dr. Kurien brought on the national map was starting
point of the most innovative and successful cooperative movement in the country. After taking
over an existing dairy cooperative, Dr. Kurien totally changed the concept of dairying and
introduced the innovations such as a network of veterinary services and cattle breeding centres.
Sixteen years later, he became the head of the National Dairy Development Board, to develop
and replicate the Anand pattern all over the country. In Gujarat alone, the movement leads to
collection of over six million litres of milk per day from 10755 village cooperative societies and
then storing, processing and producing milk products at the twelve district dairy unions. Amul
today is one of the largest food brand in India with an annual turnover of Rs. 2300 Crores p.a.
Today the primary aim of Dr. Kurien (Although he is sacked from the NDDB and Amul, but still
his vision remains the same) is to build an economically strong society through a cooperative
network to provide quality service and products to consumers and good return to members.

It is not Where We Reach, but more importantly How we reach mattersGo on- 841 -.

A Management Paradise (un) Ltd .Endeavor.I,MP


EPICenter
"Wealth is more a state of mind than a state of pocket"

The story of the "Rich farmer - Poor farmer" should explain.

One morning a farmer got up and as usual did his rounds of the farm. Though it was a small farm
he was very proud of it, having built it up from nothing. He looked out on his orchards laden with
fruit. The crops were growing well in the fields. His cattle and sheep were fat and healthy and
there had been a fine crop of calves and lambs.

Though there were many things he could not afford he was contented and felt truly wealthy with
such abundance. Later that day he received a visitor, who, while they enjoyed a cup of coffee told
him of the discovery of diamonds and the fantastic wealth that was to be had from a diamond
mine.

After the visitor had left the farmer sat thinking about the possible riches that he had been told
about. He thought of how easy it would be to merely dig a few stones out of the ground, none of
this back breaking, daily grind of farming.

Now, when he looked out over his farm he saw how small, and limited, it was. He thought about
how hard he had to work to earn his meagre living. Suddenly he did not feel rich and contented
any more. He no longer looked at his farm with pride, seeing how well it was doing. No, now he
looked at it with resentment and noticed all the things that were wrong. He resented all the work
he would have to do to keep it going.

When he got up that morning he was a contented and wealthy farmer yet when he went to bed
that night he was a discontented and impoverished farmer. There is no problem with wanting
more than you now have, but the key to wealth is to value and appreciate what you have while
you strive for what you would like to have. Develop a wealth consciousness by appreciating what
you have,

Realize that "Wealth is more a state of mind than a state of pocket"

It is not Where We Reach, but more importantly How we reach mattersGo on- 842 -.

A Management Paradise (un) Ltd .Endeavor.I,MP

5) The philosophy of co-operation attempts to outline a rational and constructive


approach to Socio- Economic Problems. Discuss
Solution:
The Mac lagan Committee in India (1950) has defined co-operation as The theory, which
maintains that an isolated and powerless man can, by association with others and by moral
development and mutual support, obtain to that extent the material advantage available to the
wealthy and powerful persons and thereby develop himself to the fullest extent of his natural
ability.
Pandit Jawaharlal Nehru once said:
"The idea of co-operation is something much more than merely an efficient economic way of
doing things. It is economic, it is fair, it equalizes and prevents the disparities from growing.
But it is something even deeper than that. It is really a way of life and a way of life which is
certainly not a capitalist way of life and which is not a hundred percent socialist, though it is
much nearer socialism than capitalism. Anyhow, it is a way of life."

The Co-Operative Approach Implies:


 Treating people as origins of action, not as objects to be manipulated or serviced
 Encouraging people to work together and help one another solve mutual problems
 Designing useful structures, processes, products and services so as to meet people's needs
rather than for profit-making purposes alone.

There is no doubt in saying that the philosophy of cooperation attempts to outline a rational and
constructive approach to Socio-Economic problems. This is justified by the following:
 A general body meeting of the members of the society is the supreme authority in
conducting affairs of society.
 1 member 1 votes irrespective of the individual share holding.
It is not Where We Reach, but more importantly How we reach mattersGo on- 843 -.

A Management Paradise (un) Ltd .Endeavor.I,MP


 The rule of majority and control of board of management in accordance with, the
democratically expressed will, of the members.
 The board of management elected through a process agreed by the members and the
board is accountable to the members.
 Member Economic Participation: Members contribute equally and also democratically to
control capital of their co-operatives. The capital is the common property of cooperatives. Members usually receive limited compensation, if any on the capital
subscribed as a condition for membership. Members allot surplus for the following
purposes:
Development of infrastructure
Setting up of reserves
Allied activities relating to the co-operatives
 Autonomy and Independence: Co-operatives are autonomous self-help organization,
controlled by their members. If they enter into agreements with other organization,
including government or raising capital from external sources they do so on terms that
ensure democratic control by the members to maintain co-operative autonomy.
 Education, Training and Information: Education becomes a prime factor to make a cooperative successful.


Co-operation among Co-operatives: Its a new concept that concentrates on


developing and strengthening of different co-operative sectors by making them inter
dependent. Co-operatives serve their members most effectively to strengthen their cooperative movement by working together through Local, Regional, National and
International Structures.

 Concern for community: Co-operatives work for their sustainable development of


communities through various programs and policies approved by their members.
 Values: Co-operatives are based on the values of Self-help, Self-responsibility,
Democracy, Equality and Solidarity. In the tradition of their founders the members
believe in the ethical values of honesty, openness, social responsibility and caring for
others.
 Equitable distribution of surplus: A profit earned by a co-operative is to be distributed
according to the decision of members. A limited interests or dividend can be paid in
proportion to their transactions with the society after making provisions of business and
social welfare activities. In a joint stock company the surplus is distributed on the
number of shares held by an individual irrespective of their transactions with the
It is not Where We Reach, but more importantly How we reach mattersGo on- 844 -.

A Management Paradise (un) Ltd .Endeavor.I,MP


company, but in a co-operative the surplus is distributed on the basis of the number of
dealings with the society.
 It is not obligatory to pay dividend out of surplus as the members may divert the use of
these funds for overall development of the society and its members. It is not obligatory
to pay dividends to members entitled for logically it also means they are obliged to bear
loses if any, Usually any such loses are adjusted from the reserve funds.
 Self help and Mutual help: As the co-operative enterprise is meant for the economic
weak, they come together; form a co-operative, pool their resources, work together for
mutual benefit and the benefit is shared by all the participants. Individually a person
may not be capable to withstand the market forces and exploitation, but together they
can strengthen their position and pursue their economic interests.

Gandhiji once said, Mother Earth has enough resources for everybodys needs but not for
everybodys greed. This can be effectively said to be the prime basis of Cooperatives as
they show us the path of Socio-Economic Justice.

Thoughts to Live/Work By.:

It is not Where We Reach, but more importantly How we reach mattersGo on- 845 -.

A Management Paradise (un) Ltd .Endeavor.I,MP

6) Discuss the differences between Cooperatives and other forms of Business


organizations?
Solution:
(Give a brief summary of Cooperatives.)
Joint Stock Company: a JSC is basically a corporate and legal entity which is basically
formed with a Vision, Mission and Objectives. The sole aim of a corporate is Profitability
and it operates on the dual aspect of Management and Ownership. It enjoys limited liability.

Partnership: A partnership is basically wherein two or more person come together, for a
common objective to fulfill. In this type of Association, the owners representing the
business are called as Partners. They enjoy Unlimited liability.

Co-operative Organization and Joint Stock Company


1. Definition:
Co-operative Organization This is an association of persons who have come together on a
voluntary basis to promote their economic interest.
Joint Stock Company It is an artificial person, recognized by law with a distinctive name,
common seal, common capital, consisting of transferable share, carrying a limited liability and
having a perceptual succession.

2. Basic Objective:
Co-operative Organization The primary objective of a co-operative society is to provide
basic or essential service to its members. Profit earning is a secondary motive.
Joint Stock Company Basic objective is to earn profit out of the business, since it is a purely
commercial organization.

3. Minimum Members
Co-operative Organization A minimum of 10 adults are required to form a co-operative
organization.
It is not Where We Reach, but more importantly How we reach mattersGo on- 846 -.

A Management Paradise (un) Ltd .Endeavor.I,MP


Joint Stock Company For a Pvt. Ltd. Company minimum 2 members are required and for a
Pub. Ltd. Company minimum 7 members are required.
4. Share List
Co-operative Organization It is always kept open for new members. A person can become a
member of the society by purchasing a share. Co-operative shares are not sold beyond their face
value and hence they are not subject to speculation.
Joint Stock Company The share list is closed as soon as issued capital is subscribed. Thus
shares of a company are subject to speculation and normally the face value and market value
differ.

5. Voting Rights
Co-operative Organization They are based on the principle of One Man One Vote.
Joint Stock Company They are based on individual holdings. The principle is One Share
One Vote.

6. Transfer of Shares
Co-operative Organization Shares are non-transferable. A member of the society can
withdraw his capital by giving a notice.
Joint Stock Company A Share holder cannot withdraw his capital during the lifetime of the
company. Shares are freely transferable in the market.

7. Distribution of Surplus
Co-operative Organization Co-operative provide a certain amount of Interest on Capital.
Surplus is not distributed in proportion to the capital contribution. The return to the members are
called as Bonus
Joint Stock Company Distribution of surplus is in the form of dividend depending upon the
holding of the individual.

8. Privileges
It is not Where We Reach, but more importantly How we reach mattersGo on- 847 -.

A Management Paradise (un) Ltd .Endeavor.I,MP


Co-operative Organization The basic objective being service they enjoy lot of incentives
from the Government and also have exemptions from Income Tax, Registration Fees, Stamp
Duty etc.
Joint Stock Company Since the basic objective is profit they dont enjoy and privileges.

9. Area of Operation
Co-operative Organization Generally co-operatives operate in a limited area such as a Town,
Locality, Tribe etc.
Joint Stock Company The shareholders are scattered in the various parts of the country. Thus
joint stock companies have a vast area of operation.

10. Members Interest


Co-operative Organization Members take personal Interest in the working of the society.
Joint Stock Company Members do not take personal interest in the day to day affairs of the
company.

11. Capital Holding


Co-operative Organization An individuals holding is generally limited. Dividend on a share
cannot exceed 12% of the net profit of the current year.
Joint Stock Company An individuals capital holding is not limited. Dividends are paid on
the basis of net profit of the current year.

12. Management
Co-operative Organization Election by democratic process. Management is responsible to
the members.
Joint Stock Company Management composed of elected Board of Directors.

13. Financial Resources

It is not Where We Reach, but more importantly How we reach mattersGo on- 848 -.

A Management Paradise (un) Ltd .Endeavor.I,MP


Co-operative Organization They are limited because they dont have the capacity to raise
and borrow funds.
Joint Stock Company They are unlimited, since the capacity to borrow and raise funds is
very high.

Proprietorship, Partnership and Co-operatives


1. Proprietorship Formation is easy without legal formalities.
Partnership Formation of Partnership is very easy.
Co-operatives Formation is not as easy as it is an association of people.

2. Proprietorship Not controlled by any special act or law.


Partnership The Indian Partnership Act governs it.
Co-operatives It is governed by the Maharashtra State Co-operative Society Act 1960.

3. Proprietorship It need not be registered


Partnership Registration is optional
Co-operatives It has to be registered

4. Proprietorship Only One member is required to form.


Partnership Minimum number of members is 2.
Co-operatives Minimum number of members is 10.

5. Proprietorship Unlimited Liability


Partnership - Unlimited Liability
Co-operatives Liability is Limited

6. Proprietorship It is not separate from the proprietor.


Partnership It has no separate legal entity. There is no distinction between partners and
their firm.
Co-operatives The identity is separate.

7. Proprietorship It comes to an end at the death of the Proprietor.


Partnership It comes to an end at the death of any of the Partner.
Co-operatives It does not come to an end by the death of any of the members.
It is not Where We Reach, but more importantly How we reach mattersGo on- 849 -.

A Management Paradise (un) Ltd .Endeavor.I,MP


8. Proprietorship Surplus is enjoyed by the Proprietor
Partnership Surplus is shared by the Partners in agreed Ratios.
Co-operatives Not more than 12% of the net profit can be distributed as bonus.
7) Write Short notes on any two of the following:
1.

8ational Bank for Agricultural And Rural Development (8ABARD)


NABARD is an apex institution accredited with all matters concerning policy, planning and
operations in the field of credit for agriculture and other economic activities in rural areas in
India.
The committee to review arrangements for institutional credit for agriculture and rural
development was setup by RBI under the chairmanship of Shri. Sivaraman to study the
scope of rural finance. It recommended the establishment of a national bank which will deal
exclusively for agriculture and rural development. Thus the Indian Parliament through the
act 61 of 1981 approved the setting up of NABARD.
The bank came into existence on 12th July 1982 and it was dedicated to the nation by the
then Prime Minister Mrs. Indira Gandhi on 5th November 1982.
NABARD took over the function of the erstwhile Agriculture Credit Department and Rural
Planning and Credit Cell (RPCC) of RBI and the Agricultural Refinance and Development
Corporation (ARDC).
Its subscribed and paid up capital was Rs. 100 crores which was enhanced to Rs. 500 crores
which was contributed by the Government of India and RBI in equal proportion. Today the
subscribed and paid up capital is Rs. 2000 crore.
1. NABARD is an apex institution, which performs the refinance function for the institution
providing investment and production credit for promoting the various departmental activities
in rural areas.
2. It takes measures towards institution building for improving capacity of the credit delivery
system, monetary, formulation of rehabilitation schemes, restructuring of credit institutions,
training of personnel etc. It co-ordinates the rural financing activities of all the institutions
engaged in developmental work at the field level and maintains liaison with the Government
of India, RBI and other national level institutions concerning with policy formulation.
3. It prepares on annual basis, rural credit plans for all districts in the country.
It is not Where We Reach, but more importantly How we reach mattersGo on- 850 -.

A Management Paradise (un) Ltd .Endeavor.I,MP


4. It promotes research in the fields of rural banking, agriculture and rural development.

Mission
Promoting sustainable and equitable agriculture and rural development through effective
credit support, related services, institution building and other innovative initiatives.
Functions
1. To develop policy and plans with respect to matters relating to agriculture and allied
activities.
2. To undertake developmental activities in the rural areas.
3. To undertake programs for training, consultancy, research in respect to credit of
agriculture and rural development.
4. To provide refinance facilities to the RRB (Regional Rural Bank) and various cooperatives.
5. Inspection of Banks.
6. Coordination.

b)

8ational Dairy Development Board (8DDB)


The successful program undertaken by Anand Milk Producer Union Ltd. encouraged the
government of India to setup a National Level Board. Thus NDDB was setup in 1965.
The main objective of the board is to replicate the Anand type all over the country. The World
Food Program in 1970 helped the board with financial assistance of Rs120crores. This fund was
for the development of Milk Supply Co-operatives. Finally this program was named Operation
Flood.
Objectives:
1. To introduce the development program for development of dairy operation and also
prepare program for agro-based industries by providing financial and technical aids.
It is not Where We Reach, but more importantly How we reach mattersGo on- 851 -.

A Management Paradise (un) Ltd .Endeavor.I,MP


2. To assist setting up of Milk Societies under the Co-operative Sector.
3. To conduct research on development of agriculture and horticulture.
4. To provide necessary technical support for societies collecting, storing, selling milk and
also producing various milk products.
5. NDDB helps in construction of buildings, provision of machinery etc. in respect to dairy
development.
6. To provide consultancy services for management of co-operative milk producers
societies.
7. To advice the Government of India on Rural Development Programs.
8. To help in export of dairy products and also to import technical goods.
Also refer paper 2002 Dairy Cooperatives.

c)

Sugar Co-operative
Indian Sugar Co-operative can be broadly classified into 2 sub sectors:
1. The organized sector e.g. Sugar Factories.
2. The unorganized sector e.g. Manufacture of Gur and Khandsari.
Out of the 453 sugar mills in the country, 252 are in the co-operative sector, 134 are in the
private sector and 67 are in the public sector. The government of India has appointed for tariff
board to examine the position of the sugar industry, due to this board the Indian Legislation
passed the sugar industry protection act in 1932.
Sugar is controlled a commodity in India. It is covered under the preview of the essential
commodities act 1955. The government controls sugar capacity editions through individual
licensing, determines the price of the major input which is sugarcane, decides the quantity that
can be sold in open market, fixes the prices of sugar etc. Government control is over all aspect of

It is not Where We Reach, but more importantly How we reach mattersGo on- 852 -.

A Management Paradise (un) Ltd .Endeavor.I,MP


production and sale of sugar extends to the level of wholesalers need to obtain a license issued
by the government before they can begin to operate.

Problems
1.

De-zoning: The co-operative sugar industry was so far protected from competition from
other sugar factories. Due to the policy of zoning which were compulsory for a farmer to sell
and supply sugarcane to the prescribed factory within the zone of its field. This policy was
dismantled in 1996-97 due to which the factories, which could not give better price to the
farmers had to close down.

2. Shortage of Sugarcane: Many sugar factories are located in the main shadow region due to
which there is a lack in sugarcane produced which leads to un-utilization of processing
capacity.
3. High cost of water and electricity: The high cost of water and electricity increases the cost of
production.
4. Huge financial loses: Due to improper irrigation facility, scanty or no rainfall result in huge
financial loses. Also there are high loses due to improper storage facilities.
5. Corruption: Corruption is one of the major problems, which is faced by the co-operatives.
The main concept of Self help through Mutual help is lost in corruption.
6. Political influence and interference: There is too much of political influence and interference
in the co-operative society.
7. Problems of Trade Union: The trade unions come up with strikes and lockouts, which result
in poor functioning of the co-operatives.
8. Weak Capital Base
9. Audit Problems
10. Uneconomic operations
It is not Where We Reach, but more importantly How we reach mattersGo on- 853 -.

A Management Paradise (un) Ltd .Endeavor.I,MP


11. No return to the State Exchequer.
Critical factors to be kept in mind while solving the case studies related to Sugar Factories:
i)

Plant Size.

ii)

Cane supply and Early Cane Processing

iii)

Coordination of harvesting with processing.

iv)

Steady flow of fresh cane to sugar factories.

v)

The length of Crushing Season (150-180 days)

vi)

Scheduling of transport.

vii)

Scheduling of harvesting, early or intermediate or late ripening


variety.

viii)

Maintenance of plant.

ix)

Use of by-products.

d)

Khadi And Village Industries Commission (KVIC)


KAVIC was setup in 1957.
The main Objective of setting up this commission was: 1. To reduce the unemployment problem in India.
2. To boost the Rural Economic Development through promotion of small village industries.
3. To create awareness about the hidden struggle through promotion of Gandhian Values.
Activities
1. To prepare programs on promotion of Khadi and Village Industries and implement those on
a National scale.
It is not Where We Reach, but more importantly How we reach mattersGo on- 854 -.

A Management Paradise (un) Ltd .Endeavor.I,MP


2. To help in training and development of village artisans and managers involved in this
industry.
3. To make arrangement for raw-material storage and to facilitate in the distribution of finished
goods.
4. To study the problems of village economies and to suggest recommendations.
5. The commission provides assistance to industries like Tanning, Pottery, Paddy, Processing,
Khadi Sari, Sugar and Jaggery.

It is not Where We Reach, but more importantly How we reach mattersGo on- 855 -.

A Management Paradise (un) Ltd .Endeavor.I,MP

Management Of Cooperatives November 2002

Strategy: for this paper, start with Section I as the concepts and case studies are
quite easy, and then move on to Section II.
Section I
Concepts
1) Outline the principles of Democratic management in the cooperatives?

Solution:
Democratic member control:
Such a control is a cardinal principle of co-operation, which will definitely have a healthy
influence in the political and economic structure in a country like India. It has been pointed out
rightly that if democracy is to be attained it will come not by voting, not by taxing nor by a
revolution but by putting into operation, the co-operative democracy, beginning with a smaller
scale and inter-expanding it on a larger scale.
The main purpose of a co-operative is to promote and protect the interests of the members and
nobody can protect the interests, of a person than he himself and therefore it becomes necessary
that the administration of co-operatives must be carried out democratically by the members
themselves.
The members will enjoy equal rights of voting and participation in the decision making process.
The principle of democratic control implies the following:


A general body meeting of the members of the society is the supreme authority in
conducting affairs of society.

 1 member 1 votes irrespective of the individual share holding.




The rule of majority and control of board of management in accordance with, the
democratically expressed will, of the members.

It is not Where We Reach, but more importantly How we reach mattersGo on- 856 -.

A Management Paradise (un) Ltd .Endeavor.I,MP



The board of management elected through a process agreed by the members and the
board is accountable to the members.

2) What are the main items on the agenda of A.G. M?


Solution:
These meetings are held annually to enable the members who are the real owners to exercise
ultimate control over the affairs of the co-operative. In the case of the co-operative the interval
between 2 Annual General Meetings shall not exceed 15months unless extended by the registrar.
Procedure:
1. The meeting shall be convened by the secretary under intimation to the registrar who may
depute his representative to attend.
2.

The meeting shall be presided over by the president or a member who will be elected to
preside in their absence.

3. The secretary shall read out the agenda for the meeting and the items of discussions.
4.

Voting can be undertaken for any particular cause, which will be conducted by the
president.

5. If all items in the agenda cannot be finished on the same day, the meeting may be postponed
to a suitable day not exceeding 30 days or as decided by the members present.
Agenda for the AGM:
A. Election of a president for the co-operative.
B. Admission of new members.
C. Receiving statement of accounts and reporting all transactions entered into by the
promoters upto 14days before the meeting.
D. Constitution of a provisional committee under regular election is held.
Any other matter as decided by the chair.
It is not Where We Reach, but more importantly How we reach mattersGo on- 857 -.

A Management Paradise (un) Ltd .Endeavor.I,MP

3) Discuss the principle of concern for community as a social responsibility?


Solution:
Refer Paper 2004

4) Define a Cooperative society?


Solution:
The International Labor Organization (ILO) points out that A Co-operative Society is an
association of persons, usually of limited means who have voluntarily joined together, to
achieve a common end, through the formation of a democratically controlled business
organization, making equitable contribution to the capital required, and accepting a fair share
of risks and the benefits of the undertakings.
According to International Co-operative Alliance A Co-operative is an autonomous
association of persons united to meet their common, economic, social, cultural needs and
aspirations through a jointly owned democratically controlled enterprise.
According to Lambert A Co-operative Society is an enterprise formed and directed by an
association of users, applying within itself the rules of democracy and directly intended to
serve both its own members and community as a whole.

5) Describe briefly the functions of a chairman of a cooperative?


Solution:
The chairman is responsible for the overall management and guidance to the co-operatives.
He performs all such duties and exercises all such powers as may be confined on him by the
respective state co-operative act. His role is most crucial as he performs the leader of the top
management, which is the board of directors.
The powers of the Chairman:
It is not Where We Reach, but more importantly How we reach mattersGo on- 858 -.

A Management Paradise (un) Ltd .Endeavor.I,MP


1. To conduct and regulate the proceedings in the meeting.
2. To decide the priority of the speakers.
3. To regulate their speeches.
4. To order and conduct poll.
5.

To exclude certain matters from the minutes if they are irrelevant or detrimental to the
society.

Duties of the Chairman


1. To see that the meeting is duly and properly constituted in accordance with the rules and
regulations
2. To see that the business is conducted in accordance with the agenda unless willing changed
by the members.
3. To maintain order and discipline in the meeting.
4. To act impartially.
5. To accept valid demand for election and make arrangements for it.
6. To declare the meeting closed.
7. To ensure that correct minutes are entered by the secretary in the minute book and to sign
them.

It is not Where We Reach, but more importantly How we reach mattersGo on- 859 -.

A Management Paradise (un) Ltd .Endeavor.I,MP

Case Study
Shri Murthy was a graduate in commerce with a diploma in management. He stated that he
had no problems Because the Chairman and the Directors managed everything. Asked
about his position he said he was a Glorified Head Clerk.
The Board of Directors had decided to open ten shops within a month as Government grant
in respect of shops had to be utilized before the end of the month. The chairman and other
directors were hunting for accommodation and booked any accommodation they could come
across. For purchasing the stocks for trade the board has constituted a sub-committee
consisting of the chairman and two directors. The sub-committee went to local wholesale
dealer who was introduced to the sub-committee by the local municipal councilor as a
Reputed Dealer. But for two items, jaggery and chillies the committee made monthly trips
during the season to the Agricultural Produce Market about 200 Kms away. As for the staff
appointments which were supposed to be made by the staff/sub-committee consisting of the
chairman and three other directors, they were actually made by the Chairman himself.
The Board or the members of the purchase committee would not trust him, yet they had little
idea about scientific purchases. They had stuffed the godowns and shops of the society with
large stocks which lay unsold for more than a year. Suggestions for quicker sale of these
stocks at reduced prices would cause losses to the society. They suspected that manager in
making suggestions has some personal interests in selling these to his friends. The directors
had insisted upon credit sales being Effected against his advice on the ground that consumers
were very poor and could not afford to pay cash. Credit dues gradually mounted and a large
part became irrevocable. The members of the staff appointed by the chairman approached
the chairman for everything. The whole atmosphere was demoralizing. So at first
opportunity the manager resigned.
It is not Where We Reach, but more importantly How we reach mattersGo on- 860 -.

A Management Paradise (un) Ltd .Endeavor.I,MP

a)

Who are to be blamed?


Solution:

a)

Government is the main party who is needed to be blamed. This method of financing does
only make matter worse and is also not of any help to the cooperatives. That is Govt. should
have appointed a person or incharge (this may again lead to same consequences, but atleast in
this case you are employing the people which is a positive point) to look after the spending of
the grant. Similarly, the person should have also looked after the investments of shops made by
the cooperative in an undeveloped area where people are very poor. Hence it was fruitless to
invest in such areas. If this was not enough, they should have kept an annual check on the
operating policy of the cooperative, which by and large failed, hence the main fault was of
Govt., and they set the ball rolling.

b)

Staff / Blue collared Workers gave Blue: the problem of staff in the given cooperative is
that they think they have got a Sinecure Job. That is they dont have to work. People often Quit
work, when they find a job. That is how the staff would be described as. Looking at their
attitudes, it is not hard to make out that they are the same staff selected by the chairman. The
attitude of these workers is quite cooperative, that is they have made good friends at the
workplace, and are enjoying their work (Although none), but only if they thought bout the
organization. An important quality of the staff would be to Pass the Buck, as it is given that
ultimately the staff hangs up their boots, that is they are not interested in working altogether.

White Collared but not White Knights: looking at the attitude of the lower level workers,
the attitude of the upper tier executives is quite contradictory. Here they want to do all the
work. They are also interested in doing others work but only because of the Vested interests.
The Board of Directors themselves decided to open 10 outlets within a month. Now that is
quite demanding, without any of them taking any real interests. The purchasing committee
which was made by the BOD didnt have a single member from their side, which means that
they are totally un-interested in the work they are doing, but only interested in the So-Called
Posts of BOD. Their ignorance was the key to failure of the cooperative. Ideally, they should
have performed the following tasks: Functions
1. Admission of new members and allotment of shares.
2. Approval for transfer of shares.
It is not Where We Reach, but more importantly How we reach mattersGo on- 861 -.

A Management Paradise (un) Ltd .Endeavor.I,MP


3. Preparation of guidelines for the conduct of the operation as per byelaws, rules and acts.
4. Approval of annual budget and its submission to general body.
5. Sanction of the expenditure for the management in the allotted budget.
6. Ensuring maintenance record and its submission to the registrar.
7. Raising resources for managing the day-to-day affairs.
8. Sanctions of investment of funds.
9. Appointing staff and agencies.
10. Appointing sub committees or exclusive committee and delegating necessary powers to
them.
11. Electing the chairmen amongst themselves.
It is also quite possible that the chairman is their good friend, and is acting on the behalf of the
group (BOD) for their Vested interests.
Chairman: this guy is seriously big guns. Perhaps he needs to be given an Electric Chair. The
only person working in the cooperative but not for the cooperative. He makes staff
selections, orders purchases etc., not at all logistically driven. He can make trips to
Agricultural Produce markets but cannot see the potential of bulk buying and economies of
scale in transportation. But most importantly, the Foundation Stones are not in places, then
how can one guarantee long term success. The selection of the location itself proved to be
wrong. They failed to define the target market and as the result failed to trigger their brand.
Even if the Target market was wrongly classified, marketing would have done wonders but
there is no mention of Kotlers innovative methods. Perhaps, the chairman never intended to
sell the goods and promote the name of cooperatives. The only positive thing seen in the
case for chairman is that Professionally he is Quite unprofessional but at the personal level
he seems to be relationship oriented.
c) Customers, but ultimately Organization paid the price: lastly, the customers of the
cooperative are to be blamed at large. That is although, they received the inferior quality
goods but they raised their voice, the reasons are obvious, they were given these goods on
credit.
d) Perhaps the political parties running the government can also be blamed. (mind you, I am
not from the opposition.). This all seems to a political drama, if i can say so, ultimately the
votes of the poorer sections counted and that is what they have targeted on.
It is not Where We Reach, but more importantly How we reach mattersGo on- 862 -.

A Management Paradise (un) Ltd .Endeavor.I,MP

b) What is the major issue you identify in this case?


Solution:

Inflaiteration of Vested Interest: According to a study majority of presidents of co-operatives


in India are big landlords. Co-operatives have turned to be a pocket constituency of an
influential family or a group of family. The ruling government always tires to induct
members of its political parties as nominees on the elected goals.
Quantitative expansion without quality: Despite arithmetical expansion co-operatives face a
qualitative challenge from all quarters for e.g. In the credit sector loans are sanctioned to upper
crust elite in the names of animal and vegetables like Tamatar Singh, Baigan Singh.
Do ministerial status: Co-operation has to be included in the state subject list and therefore
every state government has a ministry of co-operatives with a separate minister incharge. But
at the national level there is no independent ministry for co-operation. It is just one part of
the 24 divisions in the agriculture ministry. Thus it appears that co-operation has been given
a step-motherly treatment by the union government.

Empire Building by Federations: The co-operative apex federations were setup to facilitate
growth of its constituents, improve performance, increase optimal efficiency and extend such
services, which individuals societies couldnt undertake by themselves. But in practice these
federations have built empires within themselves and are doing roaring business at the cost of
primary.

Excessive Political Interference: There is hardly any state in the Indian union where cooperatives dont act as instruments in the hands of politicians. The sugar lobby in Maharashtra is
being extensively used to serve political ends. As these institutions represents a powerful vote
bank. Infact as soon as assembly elections are over mass scale supervision (takeover) of the
elected boards are done by flimsy grounds on the vims and fancies of the political party in
power.

It is not Where We Reach, but more importantly How we reach mattersGo on- 863 -.

A Management Paradise (un) Ltd .Endeavor.I,MP


Problem of Member Involvement: Many co-operatives have a problem of lack of member
involvement. This is because of the indifferent attitudes of the members. Also the principle
of unlimited liability prevents people from joining co-operative organisations. Hence, cooperatives organisations fail to involve the desired the number of members.
Window Dressing: To show a lesser figure of the overdue often the accounts are manipulated
and made up which is possible due to a defective audit. Loans are shown to have been paid
and new loans are issued thus claiming a good rate of recovery. Defective audit, inspection,
embezzlement of funds and dishonest management are common problems.
Lack of professionalism: The co-operative sector is still an underdeveloped sector of the Indian
economy.

Under the 5-year plans, keeping in mind the ultimate socio-economic goals,

appropriately enough with state recognition guidance a number of co-operatives were set up.
While the quantitative coverage on account of government support, one can come across very
few well-managed, large-sized and modern co-operatives. Due to the lack of professionalism,
growth and development is hampered in the face of the changing management scenario in the
country.

c) If you are the manager, what action you would have taken?
Solution: if i had been the manager, the following steps would have been taken by me:
1. Strengthening of human resources: The deficiencies and limitation in the recruitment and
training should be removed. Competent persons and individuals with a capacity and
unquestioned integrity should be posted on the various responsible positions.
2. Avoiding political influence: Groupism, class-consciousness, political interference, which
normally plagues should be avoided.
3. Improved recovery: It is necessary that the loans and credit facilities should reach the poor
farmers and those who have taken the undue advantage should be made to repay it, at the
same time it should also be avoided to write-off loans unless absolutely necessary and
justified.

It is not Where We Reach, but more importantly How we reach mattersGo on- 864 -.

A Management Paradise (un) Ltd .Endeavor.I,MP


4. Developing a sense of solidarity: It is the most essential aspect necessary for the growth of
co-operatives. If there is no solidarity among the members it will definitely affect the
functioning and prosperity of the co-operatives but might also face unforeseen adversities.
5. Public Contribution and participation: It is very much necessary that the co-operatives
should become self-funding rather than on government finances and subsidies. It should
have more of public contribution and participation.
6. Effective leadership: The co-operatives being a peoples movement must ensure that the
officers and the leaders should have and must acquire necessary training to run the co-op
movement more effectively as these members will be the ones who will be able to do justice
towards the cause than any outsider.
7. Multipurpose basis: The old objective, of keeping the co-operatives confined to providing
credit has to undergo a change. It is necessary to expand into an allied and infrastructural
service as per the purpose of the co-operatives.
8. Production plans: In addition to the primary function if the co-operative aims at expanding,
improving and deciding the agricultural production it can help the villagers to specialize and
pool in their resources to increase the total production and thereby achieve scale of
economics.
9. Marketing and processing: After supplying credit and producing goods the co-operative
movement has often faced failure because of inadequate and improper processing and
marketing infrastructure and therefore this also needs to be developed.
10. Professionalism: The solution for lack of professionalism could be that the co-operative
organisation must adjust by adopting new management techniques to meet the demands.
11. Diversification: The solution could be that co-operative organisations should introduce new
products, new processes new techniques and tools of productivity etc. to build up its strength
in the market and maintain it on a continued basis.
Some more remedies include
 Reorganizing of central and state co-operative banks
 Tapping of local savings
 Intensive training programs
It is not Where We Reach, but more importantly How we reach mattersGo on- 865 -.

A Management Paradise (un) Ltd .Endeavor.I,MP


 Increasing non-official control

EPICenter
"Are you a thermometer; merely reacting to your environment or are you a thermostat,
controlling your environment?"
All around you there are constant reminders of how bad life is, of how little reason there is to be
happy.
It is easy in these circumstances to give up, to go with the flow, to think "what's the use." You
try to be positive and people tell you to "get real"

You have the ability to control your attitude so that you banish fear and worry. Look at the
things that are around you, are they positive and uplifting?

Listen to your talk, is it inspiring and encouraging? Surround yourself with positive messages.
Put photographs of your favourite people where you can see them all the time. Every time you
get a compliment write it out and stick it on the wall. Develop a "brag" book of all your
achievements, put in it everything that ever made you feel great.

Make a tape or CD of your favourite music then play it whenever you travel. Become an
optimist, believe that the world is out to help you. An optimist is aware of problems but realises
that there are solutions. An optimist knows that problems are opportunities for growth. An
optimist experiences difficulties but believes that they can be overcome. An optimist sees the
negative but looks for the positive. An optimist may have reason to complain but chooses to
smile.

The happiest people do not necessarily have the best of everything but they do make the most of
everything they have. So just for today Take a look around you and see just how much you
really have to be grateful for.

It is not Where We Reach, but more importantly How we reach mattersGo on- 866 -.

A Management Paradise (un) Ltd .Endeavor.I,MP

Section II
3) Trace the history of Co-operatives movement in India.
Solution:
The concept of co-operation and mutual help is not new to Indians, since ancient times Joint
family system has been a practical ideology for Indians. Principles of co-operation are the find
of Joint family system.
The word co-operation is quite familiar to a common man. For him co-operation is simple
working together in any sphere of human activity. In this sense, the roots of co-operation can
be traced as far back as the beginning of human civilization. The modern biologist have claimed
that co-operation are the group instinct in man which enabled him to live together, work
together, and help one another in times of difficulty, has been biologically one of the most
important and vital instincts.
Relevance Of Co-Operation Today.. & Tomorrow!!
Our society has developed towards a more self reliant one. Therefore the factor of selfregulation is not far. Any civilized society thrives on self regulation as to be civilized is a selfchosen path.
Co-operation offers the solution for the times. A completely democratic set-up with laws, which
borders on moderation.
Development of Indian Co-operatives
The Co-operative Movement, which has an ultimate goal of strengthening the position of
economically weaker sections, can make a real contribution towards the progress of national
community. Over 96 yrs co-operative activity has been undertaken and since, it got a legal
acceptance after the co-operative credit society act was passed in 1904.
The main objective was to combat indebtness and provide credit to the agriculturalist. With
reference to the agro based Indian economy the co-operative sector has played a substantial role
in the process of lending a helping hand for economic upliftment.
It is not Where We Reach, but more importantly How we reach mattersGo on- 867 -.

A Management Paradise (un) Ltd .Endeavor.I,MP


With the passing of time, the co-operative act was amended, with a net result that co-operation,
expanded its role, from a mere agriculture credit provider to helping, in other inputs, like seeds,
fertilizers, pesticides, storage, marketing and processing. It also expanded into wholesaling and
retailing trade, consumer distribution, rural and small-scale industries, urban credit, training of
co-operative personnel etc.
The Co-operative Movement has left no sector of the economy untouched. But inspite of its
existence, for almost a century, it has definitely not shown an outstanding growth. To study the
growth and progress of co-operative movement we have to look back in the history.
The first legislation in the field of Co-operation in India was on the Co-operative Credit Society
for releasing the farmers from the clutches of moneylenders. It was Co-operative Credit
Societies Act 1904, thereafter came the Co-operative Societies Act 1912. In order to suit the
circumstances existing in different provinces, need was felt to have separate legislation for the
particular province .The Bombay province then enacted the Bombay Co-operative Societies Act
1925. It was then replaced by Maharashtra Co-operative Societies Act 1960 and Rules 1961.
The entire Co-operative Movement can be divided in 3 phases:
I Period prior to the enactment of the 1st co-operative society act 1904
II Period between 1904 1951 before the beginning of planning in India
III Growth of co-operatives during the planned period (5 yr. plans)
Present Situation
1. Phase I : Co-operative Movement prior to 1904
The credit of initiating the co-operative movement as seen earlier goes to the Rochdale Pioneers
from England. The policies of organizational setup adopted by them, have survived the test of
time and have become the principles of co-operative movement.
Co-operation is not new to India, but the versions were different. In ancient India, it was in form
of 4 principles like the Kula, Gram, Sreni and Jati.
(hierarchy based on units)

Kula Family

It is not Where We Reach, but more importantly How we reach mattersGo on- 868 -.

A Management Paradise (un) Ltd .Endeavor.I,MP


Jati Caste

Gram Village

Sreni Classification at different levels

The Kula was the 1st form of a co-operative activity; it was both a political and socio-economic
activity. As the society stabilized and expanded the aspects of economic and social co-operation
narrowed to the limits of joint family. Where the land is owned and cultivated commonly and
the adult members share the duties of the family and live under one roof. Co-operation at the
Gram (village) level emerged after the Kulas were stabilized. The gram sabha was a cooperative engine which undertook the works for economic and social progress of the village
commons such as land, pastures, roads, highways, paths, common gardens, etc.
The artisans and cultivators often combine for the purpose of co-operative working and the
members jointly undertook, the use of selling of tools and implements of production. The Sreni
has been a development of the post Vedic era.
The co-operative and economic organization in this time was more of artisans, industrial and
handicrafts workers, merchants, traders, bankers, agriculturists, house builders, etc. The Sreni
worked basically as bankers and merchants guilds but also extended their role as charitable
institutions. The co-operation at Jati level was mainly for achieving social goals such as
education, charity, and relief work.
2. Phase II : Co-operative Movement (1904 1951)
Co-operation became legalized and was introduced as official movement in 1904. This was
considered as a turning point in the economic and social history of India. The period 1904
1951 can be studied better if it is divided into the following stages.
I.

1904 1912

1904 act provided an impetus for organization of primary credit societies and stress was laid on
promoting agricultural credit. There were 8187 societies by 1912, which had a membership of 4
Lakhs and working capital of 3357.7lakhs. One of the major features was that the government
was the prime mover of this movement but the credit was marked by insufficiency and delay,
and the recoveries were far from satisfactory.

It is not Where We Reach, but more importantly How we reach mattersGo on- 869 -.

A Management Paradise (un) Ltd .Endeavor.I,MP


The loans system was also defective. Mr. C.S. Campbell, the registrar of co-operative society
Bombay had correctly pointed out that, the lazy secretary of a dull sort, will collect and pay out
again to the same person by the book of transaction.
The worse of the system is lump on another day, meanwhile a guest comes, a cow dies, etc are
some of the excuses.
II.

1912 1919

1912 saw the amendments, where another co-operative societies act was enacted and the defects
of the 1904 act were removed. Some important changes of the new act were:
a.

It enabled the registration of non-credit societies so as to ensure all round progress of the
movement and the economy. The original act confined the registration, only to primary,
credit society.

b. Registration of Central societies was now possible.


c.

A provision was made for the distribution of profits in order to protect the interests of the
investors.

This period saw a rapid growth in the number of co-operative credit societies but also of nonagricultural credit societies. Though the development was uneven in different provinces. By
1919 the number of seats had gone upto 28,000
III.

1919 1929

With the introduction of reforms act of 1919, it gave an option, of modifying the existing act
1912, at the provincial level. Certain provinces did enact their own acts, which gave rise to a
rapid growth of the movement in this time. This is referred as the period of unplanned
expansion.
IV.

1929 1939

The great economic depression gave a serious set back to the co-operative sector. The falling
prices of the agricultural products threatened the existence of some societies and disturbed the
economic stability of others.

It is not Where We Reach, but more importantly How we reach mattersGo on- 870 -.

A Management Paradise (un) Ltd .Endeavor.I,MP


Other major causes of the failure where, lack of requisite education and adequate supervision
and guidance. During this period of stress, rectification and rehabilitation was the main aim of
the co-operative departments. Rather than expansion, one of the significant developments was
the establishment of Reserve Bank of India in 1935. Its agricultural credit department was
entrusted the responsibility of studying the problems of the agricultural credit.

V.

1939 1947

The Second World War and the rising prices made it possible to recover the over dues, with the
scarcity of resources due to war, lot of control were imposed on the distribution of essential
commodities. This helped the consumer co-operatives for a better organization. The co-operative
movement, which was till then, confined more or less as a credit movement started diversifying
its activities. The important landmark was the setting up of co-operative planning committee
under Mr. R.G. Sargiya, who suggested that the credit supply covered only one aspect in the life
of a cultivator, but should extend its cover to his entire life. This period is treated as the period
of recovery.
3. Phase III : Co-operatives in Independent India (5 Year Plans)
The most important event in the post independence era was the appointment of rural credit
survey committee, by the RBI 1951. The report submitted by the committee, revealed that,
inspite of a co-operative effort, over the last 50 yrs. Rural credit is still considerably controlled
by the private agencies, whereas the institutional agencies played a significant role. The cooperatives provided 3.5% of total borrowings by the agriculturist and that too was taken up by
the wealthy members of the credit society.
The committee recommended for an integrated scheme of rural credit, which will have state
partnership, creation of special funds with RBI, today called national agricultural credit
stabilization, national agricultural relief and guarantee funds.
It is also recommended, establishment of large sized credit society, linking credit with
marketing, training and co-operative personnel and effective supervision and audit.
The 1st plan recognized (1951 56) co-operative as an instruments of planned economic action
in democracy. It also stated that since the purpose of the plan is to change the economy from
It is not Where We Reach, but more importantly How we reach mattersGo on- 871 -.

A Management Paradise (un) Ltd .Endeavor.I,MP


individualistic to socially regulated economy, co-operation could be an effective tool, to
implement this ideal.
The 2nd plan (1956 61) was based on the recommendations of the All India Rural Credit
Survey committee. The RBI acts was amended thereby, the rural credit facilities could be
improved. There were provisions made for the creation of special funds as recommended by the
committee. The national co-operative development and warehousing board and state
warehousing co-operatives were setup. This plan also gave attention to the marketing and
processing co-operatives. This period saw a rise in the number of societies from 2.4lakhs
between (1953 56) to 3.32lakhs in (1960 61), 18,000 primary marketing societies, 390
processing units, 30 co-op sugar factories and an intensive program for co-operative training
was implemented.
The 3rd plan 1961 66 saw the use of co-operative movement in implementing, the schemes of
economic development. By the end of the plan, primary agricultural credit societies covered
89% of villages and total number of societies was 1.92lakhs. The emphasis was more on making
the societies more viable than expansion. Inspite of the efforts of revitalizing, 31% societies still
worked at loss. This period also saw as emergency and development of agro based processing
industries e.g. sugar factories, butter processing, rice mills, oil crushing, fruit and vegetable
co-operatives etc. where as the co-operative stores raised from 7058 in 1960 61 to 13077 in
1965 66. There was also an attention paid to the formation of consumer and industrial cooperatives.
The 4th plan 1966 71 had a keynote of growth and stability. The agricultural and consumer cooperatives occupy a central position in the co-operative movement. A committee was setup by
the governor of RBI to review the supply of rural credit and intensive agricultural program,
which gave a series of recommendation like
 Reorganizing of rural credit in RBI
 Setting up of SFDA (Small farmers development agencies)
 Rural electrification
The 5th plan 1971 had a keynote of growth and social justice. More attention was therefore given
to consolidate and strengthen the co-operatives so as to make them, viable, democratic
It is not Where We Reach, but more importantly How we reach mattersGo on- 872 -.

A Management Paradise (un) Ltd .Endeavor.I,MP


institutions. An attempt to establish stable and fair price shops, through the consumer cooperative was made.
The same trend of strengthening continued in the 6th and 7th plan in which the dairy and fishery
co-operatives were initiated along with development of professional management in cooperative institutions and development of managerial levels were also touched upon.

4. Present Situation
Today there are 14 national level co-operative federations, there is also a co-operative union and
a national council for co-operative training and 24 states level co-operative unions. Pune has a
national level training institute for co-operative management i.e. Vaikunthbhai Mehta
Institute of Co-operative Management. Other than this, there are 17 colleges and 73 training
centers for training and education in co-operative. At state level there are 26 state co-operative
banks and 344 district central co-operative banks.
Important sectors where co-operative is active are:
Credit marketing, industries, storage, processing, consumer, farming, housing, transport,
education, and supply etc.
Co-operatives for weaker section includes co-operative rural banks, dairy, poultry, fishery, coir,
silk, honey, handloom, etc. Amul is the best example in Asia for its milk and milk products.
5. Recommendations of Co-operative Movement:
I. Co-operative Movement (1904)
II. All India Rural Credit Survey Committee (1954)
III. Co-operative Credit Committee (1960)
(I) Co-operative Movement (1904)
The first Indian co-operative act was passed in 1904 under the Viceroy Lord Curzon. During the
period of 1904-1912 the co-operative movement made rapid strides in different fields. To boost
the development process the government of India appointed a committee under the
chairmanship of Sir E.D. Mac lagan to observe and suggest recommendations.
It is not Where We Reach, but more importantly How we reach mattersGo on- 873 -.

A Management Paradise (un) Ltd .Endeavor.I,MP

Recommendations
1. There should be one co-operative society for one village
2. The size of co-operative society should not be very large
3. There should be unlimited liability of a co-operative society
4. Society should give importance to recovery of loans
5. Society should increase the reserve funds
6. While sanctioning to any members their should be consideration of his ability to repay and
also his honesty towards the society
(II) All India Rural Credit Survey Committee (1954)
To recommend the developmental majors for co-operative sector in Rural India, the
Government of India appointed a committee under the chairmanship of A.D. Gorewalla in 1951.
This committee submitted its recommendations in 1954.
Recommendations
1. There should be mergers of primary co-operatives to make them financially sound
2. The purpose of the loan should be taken into account while sanctioning the loan
3.

No money lender/Commission Agent should be provided credit. Care should be given to


give credit only to the weaker sections of the society

4. Primary co-operative society should be provided with more finances by the National Credit
Institutions
5.

Members who dont pay arrears in time should be dismissed from the society on legal
grounds

6. There should be a link between Agricultural Credit Societies and Marketing Societies

(III) Co-operative Credit Committee (1960)


In May 1960 the committee under the chairmanship of Dr. Vaikunthbhai Mehta gave its report
to suggest recommendations on strengthening the co-operative credit structure.
It is not Where We Reach, but more importantly How we reach mattersGo on- 874 -.

A Management Paradise (un) Ltd .Endeavor.I,MP

Recommendations
1.

Service co-operative should be organized on the basis of the village community as the
primary unit. Number of villages within a radius of 3-4 miles should be grouped together

2. The society should have sound management to enhance the deposition confidence
3. RBI should liberalize its Credit limits for the central co-operative banks
4. The credit limit of a member should be fixed keeping in mind his repaying capacity
5. Care should be taken to see that the loan given is applied to agricultural productive purpose
only.

6. Co-operation In Maharashtra
Maharashtra is one of the major States of India. It is also the most urbanized and industrialized
State. Co-operative movement is widespread and has a long history in
Maharashtra and even today it plays an important role in the economy of Maharashtra. The
genesis of the co-operative movement in Maharashtra can be divided into six stages.
1. The pre-co-operative stage (1870-1903) with the Deccan Agriculturists Relief Act, The
land Improvement Loan Act 1883 and the Agricultural Loans Act, the Nickolson Report.
2. The Initial Stage (1904-1911) from the Agricultural Credit Co-operative Societies Act 1904
to the creation of the Bombay Central Co-operative bank.
3. The Evolution Stage (1912 to 1924) After the Co-operative societies Act of 1912, the
movement passed through a new phase of re-organization, formation of co-operative financing
agencies, formulation of co-operative educational schemes and organization of non-credit
societies.
4. The Stagnation Stage (1925-1947) The enactment in 1925 of the Bombay Co-operative
Societies Act widened the scope of the movement in the Bombay province both horizontally as
well as vertically. The Bombay Co-operative Insurance Society was established in the year
1930. In the same year the Co-operative Land Mortgage Bank was also formed for long term
financing for redemption of debts, land improvement and purchase of land.
It is not Where We Reach, but more importantly How we reach mattersGo on- 875 -.

A Management Paradise (un) Ltd .Endeavor.I,MP


5. The Growth Stage (1948-1961) There was an all round progress during these two decades
after India attained Independence from the British Rule. The movement diversified especially in
the rural area where sugarcane was grown. The agriculturists pursued this concept of self-help
and made the best use of the credit facilities given to them for augmenting the production.
Examples like the Pravaranagar Sugar Co-operative inspired many to organize co-operatives in
the sugar sector with long-term goals in mind. Significant attitudinal changes had occurred at the
grass root level.
This was also the period of emergence of rural leadership through co-operative movement.
The Apex Bank also started to strengthen its organization and the working of the secondary level
central financing agencies. This was also the period where institutional foundation was
strengthened.
6. The Diversification Stage (1962 onwards) Expansion, accompanied by extensive vertical
and horizontal diversification embraced all fields of socio-economic activity with gains varying
from 2 to 6 times.
The State government initiated policies and programs to strengthen the co-operative effort.
Some of the noteworthy features of the movement during this period were the increased
mobilization of resources, strengthening the co-operative effort in the sphere of agricultural
production and the building of rural leadership.
EPICenter

"A picture paints a thousand words yet it takes only one word to paint a picture"

It has been said that a picture paints a thousand words and this is true, you will see in a picture
whatever it is that you want to see. However in our lives we must understand that our words
paint pictures in our minds and then we live up to those pictures. What pictures are your words
painting?
Your mind responds to the pictures that you give it to work on. It does not care if the picture is
something that you want or something that you do not want. It will go to work to make the
picture a reality. Your mind supports you without question and will do for you whatever you
ask.
Therefore it is important that you learn to ask for what you actually want.
It is not Where We Reach, but more importantly How we reach mattersGo on- 876 -.

A Management Paradise (un) Ltd .Endeavor.I,MP


(4) What are the functions and problems of Dairy Cooperatives?
Solution:
Dairy Co-operative Society
The milk producers at the village level form these types of societies.
Functions
1. They arrange for collection center and also provide the distribution network.
2. They distribute cattle feed to the milk producers and also provide veterinary services.
3. They arrange for processing of milk into milk products.
4.

Research and Development activity are carried out by the society to improve the
productivity rate.

Objectives
1. To encourage co-operation and self-respect among the members.
2. To provide financial assistance to poor farmers to purchase cows and buffalos.
3. To arrange for the sale of milk and also to pay milk producers in coordination with various
unions and federations.
4. To prepare and undertake necessary projects for dairy development in the area.

Historical Perspective
India today is the largest producer of milk in the world. This success is due to the White
Revolution which was aimed to increase the milk production. NDDB spearheaded operations
flood under the leadership of Dr. Verghese Kurien. He has developed the world famous Amul
Brand which has proved one of the most successful milk co-operatives. Due to the success of
operation flood, 1st government of India introduced operation flood and 2nd to undertake the
activities such as:
It is not Where We Reach, but more importantly How we reach mattersGo on- 877 -.

A Management Paradise (un) Ltd .Endeavor.I,MP


1. To make arrangements for marketing of milk and milk products.
2. Provide technological support to the dairy industry.
3. Raise the hygiene standards of the cattle.

PROBLEMS OF Dairy Cooperative:


1. Excessive Political Interference: There is hardly any state in the Indian union where cooperatives dont act as instruments in the hands of politicians. The sugar lobby in
Maharashtra is being extensively used to serve political ends. As these institutions represents
a powerful vote bank. Infact as soon as assembly elections are over mass scale supervision
(takeover) of the elected boards are done by flimsy grounds on the vims and fancies of the
political party in power.
2. Vested Interest: According to a study majority of presidents of co-operatives in India are
big landlords. Co-operatives have turned to be a pocket constituency of an influential family
or a group of family. The ruling government always tries to induct members of its political
parties as nominees on the elected goals.
3.

Empire Building by Federations: The co-operative apex federations were setup to


facilitate growth of its constituents, improve performance, increase optimal efficiency and
extend such services, which individuals societies couldnt undertake by themselves. But in
practice these federations have built empires within themselves and are doing roaring
business at the cost of primary cooperatives.

4.

Do balanced growth: Co-operatives in India have developed unevenly leading to


geographical imbalances and friction among member states. Eastern states of India, Assam,
Bihar, Orissa, have made little progress in co-operative sphere. While state like Maharashtra,
Gujarat, Tamil Nadu, Punjab, Uttar Pradesh have made significant progress

It is not Where We Reach, but more importantly How we reach mattersGo on- 878 -.

A Management Paradise (un) Ltd .Endeavor.I,MP


Steps for Functioning of Dairy Cooperatives:
i)

A Dairy Cooperative is formed.

ii)

In each village, milk collection centers are opened by the Dairy Cooperatives.

iii)

At milk collection center, milk is graded accordingly to the fat content.

iv)

Milk routes are established connecting the various milk collection centers with the
milk factory.

v)

Dairy factory is established by the dairy cooperative unions/federation.

vi)

At the dairy factory, the milk is boiled, chilled, pasteurized and packed and
transported to the market.

Current Trends:
i)

Under the milk and milk products order, units with the installed capacity in excess of
10, 000 litres of milk per day or 500 tonnes of milk solids per year have to be
registered.

ii)

GOI has setup an advisory board under milk and milk products output to advise on
products sale, purchase and distribution of milk.

iii)

Production of ice creams was earlier reserved for SSI, now it has been de-reserved.

Case Study, ( For this Question)


SHREE WARADA SAHAKARI DUDH UTPADAK PRAKRIYA SADGH LTD.
(Sangh)
i)

Sangh thought of purchase of buffaloes from Gujarat, a cross breeding of cows,


upgrading of buffaloes, vet aid, fodder production, supply of cattle feed etc.

ii)

It made efficient arrangements for milk collections, grading, chilling and transport to
its own dairy plant for processing, packaging and manufacturing of milk products.

iii)

MILK COLLECTION:
a) Collection of milk from Sangli and Kolhapur and also from the border districts of
Karnataka.
b) Defined 18 milk routes in the morning and then in the evening.
c) Annual collection of milk approximately 600 Lac Litres.

It is not Where We Reach, but more importantly How we reach mattersGo on- 879 -.

A Management Paradise (un) Ltd .Endeavor.I,MP


d) Split up of Cattle population: 71.5 % Buffaloes and 21.5 % Cows.
iv)

Milk and Milk products:


a) Supply Various grades of milk
b) Ghee, whole milk powder, skimmed milk powder and Shreekhand.
c) Warana Spray.
d) Started Research and Development and marketing butter, baby food, flavored
milk and lassi etc.
e) It also started marketing milk products.

v)

Marketing started marketing of milk and milk products in Mumbai and Pune from
1979.

vi)

Quality circles:
a) 12 committees to maintain quality standards.
b) Kaizen quality circles.
c) Its malted milk factory received PanchGanga Science Awards (1994) for saving
raw materials, water, electricity and furnace oil to the tune of Rs. 74 lacs.

Critical Factors for the analysis of Dairy Cooperatives:


i)

Forming a milk cooperative society of milk producers and stipulating a minimum


quality of milk form each producer.

ii)

Upgradation of Milch animals.

iii)

Provide a Veterinary, Supply of free fodder etc.

iv)

Establishment of milk collection centres.

v)

Defining milk routes and provisioning/providing refrigerated vans.

vi)

Establishment of dairy factory for boiling, chilling, processing, packaging of milk


and milk and various value added products.

vii)

Proper marketing and distribution system.

viii)

Helping farmers through finance, insurance and Subsidiaries.

It is not Where We Reach, but more importantly How we reach mattersGo on- 880 -.

A Management Paradise (un) Ltd .Endeavor.I,MP


(5) Explain the role of Professional Management in Cooperative Organization?
Solution:
The Cooperatives today hold an important place in the process of the countrys economic
development. The economy may function in any way, like it can be a Capitalist, Socialist or a
Mixed democracy like India. The development of any country will always depend on the level of
cooperation amongst its people. That is the reason a need is felt for cooperatives to take a
leading part in the development of the country. In every nation, cooperatives have played an
active role in the empowerment of the rural areas. In India, they are filling a major gap left by
the Public Sector and Private Sector.

It can be seen that cooperatives have treaded the path where no public or private sector could
reach and have worked in empowering in the masses at the grass roots level to help themselves
so that they can help others. But for efficient working of the cooperatives of the cooperatives
Principles of Cooperation should be combined with the Principles of the Management. Where
Management means Coordination and cooperation amongst the various segments.
The cooperative enterprise when analyzed critically, it was found that not only profit but the
participation of members both in business and management should be kept in view. When these
So-called Efficient organization are working on the principles of traditional principles of
management, the work of decision making is undertaken by the elected board, while technical
It is not Where We Reach, but more importantly How we reach mattersGo on- 881 -.

A Management Paradise (un) Ltd .Endeavor.I,MP


functions which require decision making have to be taken by those people who are experienced
and experts in their fields.
Few of the problems discussed below are the outcome of not being Professional in the
Approach:
Government influenced: Recent years have shown an increasing interference of the
government by imposing restrictions, which has seriously hampered the democratic function.
Further the corrupted political ideas have taken co-operatives as their personal kingdoms, which
has seriously hampered the purpose of co-operative movement. Government has practically
influenced the co-operative movement as the major parts of the financial assistance still come
from the government and therefore it has not really reached the peoples movement.
Political Influence: The political influence and pressure on officials in day to day function
granting of loans to undeserving people, corruption amongst officials is sickening the cooperatives. The political interference and administrative high handedness has been the root
cause for loosing the purpose. Further growing vested interests, ineffective federal structure
and reliance on government shows the dependency which is coupled with the processing
handled by several departments, lack of co-ordination and export guidance have together
arrested and influence co-operatives.
Bureaucrats on deputation: Appointment or deputation of government officials has proved
to be harmful as well as anti-democratic. Further these bureaucrats are more rules bound
and less business like in order to complete the procedures they sacrifice the business
opportunity. The expenditure incurred for these bureaucrats are much more and acts as a
financial burden because equally qualified people can be recruited at much lesser cost from
the open market.
Management in wrong hands: Management of most of the co-operatives has been in the
hands of the illiterate, ignorant and untrained people who were unfit for the work entrusted
on them.
Window Dressing: To show a lesser figure of the overdue often the accounts are manipulated
and made up which is possible due to a defective audit. Loans are shown to have been paid
and new loans are issued thus claiming a good rate of recovery. Defective audit, inspection,
embezzlement of funds and dishonest management are common problems.
It is not Where We Reach, but more importantly How we reach mattersGo on- 882 -.

A Management Paradise (un) Ltd .Endeavor.I,MP


Lack of funds: The main problem of the co-operatives is lack of funds. This is because the
members do not save or deposit enough of their savings to contribute to the working capital
of the organisation. Besides, there is also lack of response from general public. Hence, most
co-operatives organisations fail to attract the desired deposits.
Lack of professionalism: The co-operative sector is still an underdeveloped sector of the
Indian economy. Under the 5-year plans, keeping in mind the ultimate socio-economic
goals, appropriately enough with state recognition guidance a number of co-operatives were
set up. While the quantitative coverage on account of government support, one can come
across very few well-managed, large-sized and modern co-operatives. Due to the lack of
professionalism, growth and development is hampered in the face of the changing
management scenario in the country.
Problem of Diversification: It must be approved that co-operatives in India exhibit excessive
sensitivity towards memberships and other aspects of the organisation and pay little attention
to exploring new areas of diversification. This is a serious problem.
MADAGEMEDT ROLES ID COOPERATIVES:
As defined by Harold Koontz, Management is basically a function of its various variables. These
variables are planning, directing, staffing, etc.
Planning: when this function of the management is implemented effectively and in a proper way,
cooperative management and corporate management wont have any difference of operating.
This can be seen from the increased investments in the research in the product, services and
marketing opportunities. The process should be such that the work duties of the members should
not be considered while assembling the various parts of the Management. All the members
should be involved in this process. As said, You always plan to Plan, and nothing is planned.
Thus planning is a continuous process with each of the other process intertwined to it.
The plans should be act as means and not as ends. They need to be revised, amended, and
continuously altered so as to make the organization more progressive. The best examples of
these are behemoths like Google, Microsoft, etc. although these are not cooperatives but it takes
a lot of planning to act as a Monopolist. It avails effective communication within an
organization. These plans include analysis of capital requirement, manpower, expansions,
diversifications, or downsizing etc.
It is not Where We Reach, but more importantly How we reach mattersGo on- 883 -.

A Management Paradise (un) Ltd .Endeavor.I,MP


Organizing:
This function of the management provides coordination of positions and effective
communication within the organization. Duties and responsibilities are laid down and the span
of control is established. Today, one can that the organizations prefer to have a flatter structure
that is they have forgotten the Pyramidal style. In a corporate, the membership is shown at the
top, followed by the delegated body- Board of Directors, Managing Committee, and the
Chairman in succeeding order indicating the flow of the authority. But in cooperative it should
be other way round.
Directing:
The managers and the members of the cooperative have to direct action to achieve the objectives
of the plan, with reference to the key result areas. The managers role is to provide leadership to
people working under them. And those in cooperative organization have to face the challenge of
meeting the difficult and ever changing requirements of customers, members and subordinates.
Directing functions must attract increasing attention of the managers as key element for
achieving their objectives.
Staffing:
In order to achieve the objectives, an organization must emphasize on sound, human practice
and concentrate on attracting dynamic managers. Ex. The Cooperative of Amul, after the change
of leadership of Mr. Verghese Kurien will have to adopt a new strategy for Successive leaders.
Hence absorbing Proper Candidates with proper skills is an essential staffing function in a
cooperative.

It is not Where We Reach, but more importantly How we reach mattersGo on- 884 -.

A Management Paradise (un) Ltd .Endeavor.I,MP


Professionalisation in Management:
The following are the main criteria for proper working of the cooperative,:
a) Remuneration should be paid with due regards. As it works as an intoxicant for the staff.
b) There should be scope and facilities to take up education for the cooperative
management.
c) The practice of management claiming to be a profession must have a systematic body of
knowledge.
Professionalisation of Management
Management Techniques

1. Planning

Functions
1. Finance (proper auditing procedure, activity based
costing, proper formulation of balance sheet)

2. Organizing

2.

Marketing (MIS, production planning, advertising,


quality management, SWOT analysis)

3. Directing &

3.

HRD (HRD audit, recruitment policy, training and


development)

4.

Production (JIT, Kaizen, worker participation, quality


circles, SBU, management control system, corporate
governance, MBO)

Process

Leading
4. Coordinating

Role of Leadership in Co-operatives


Dr. Verghese Kurien
1. Educating the people on co-operative principles.
2. Influencing the masses to form co-operative organization.
3. Organizing the group and explaining to them the objectives and goals of the venture.
4. Motivating the masses.
5. Giving direction to the movement and coordinating and controlling the activities.
6. Explaining a vision to the people and showing uttermost commitment and faith towards it.
7. Reforming the role of a councilor and a troubleshooter.
It is not Where We Reach, but more importantly How we reach mattersGo on- 885 -.

A Management Paradise (un) Ltd .Endeavor.I,MP


Examples of effective leaders
 Dr. Vasant Rao Dada Patil Vasant Rao Dada Co-operative Sugar Factory, Sangli (largest
sugar factory in Asia).


Dr. Balasaheb Vikhe Patil Co-operative Sugar Factory at Pravaranagar, Ahmednagar


district.

 Dr. Vaikunthbhai Mehta


 Dr. Sharad Pawar Baramati
Influence of co-operative movement on the political life in India:
Examples
1. Sharad Pawar Defence minister of India, CM of Maharashtra for 12 years, Deputy
Chairman of National disaster management committee of India.
2. Vilas Rao Deshmukh CM of Maharashtra, controls co-operative lobby in Latur, controls
Co-operative Sugar Industry.
3. Dr. Jayant Patil Finance Minister of Maharashtra, controls co-operative sugar factory in
Islampur.
4. Dr. Patangrao Kadam Founder of Bharti Vidya Peeth, a dean university, Minister at State
Cabinet, controls education lobby.
5. Dr. D.Y. Patil Engineering colleges, Dental colleges.
Hence, these are the impact of adopting professional management in the cooperatives sector.

Thoughts to Live/Work By.:

It is not Where We Reach, but more importantly How we reach mattersGo on- 886 -.

A Management Paradise (un) Ltd .Endeavor.I,MP


(6) Explain the working and problems of Agricultural Credit Societies highlighting the role of
DCC bank, MSC bank, and 8ABARD.
Solution:

Introduction
India lives in its villages and in rural India a substantial portion of the population is
engaged in agriculture or allied activities. The farming community consists mainly of
small farmers and agricultural laborers. Poverty is generally wide spread in the rural
areas, with hardly any pockets of prosperity. As rural economy is running short of
capital, it must be assisted with adequate capital, appropriate technology and required
training in modern technology of production. Hence provision of adequate financial
assistance to agricultural, rural industries and rural artisans is necessary. The welfare
and prosperity of the nation rests on the development of agriculture and allied
activities.

Purposes of Rural Credit


 For purchase of farm implements viz. indigenous wooden implements, improved iron
implements, agricultural implements, hand tools etc.
 For purchase of tractors with accessories, threshers, power tillers, combine harvesters,
power sprayers.
 For purchase of oil engines, electrical engines, pump sets, construction of wells, leveling
of ground for irrigational purposes etc.
 For the purpose of construction and repairs to farm buildings/structure of the type viz
bullocks shed, farm store, godowns, animal farm etc.
 For the purposes of bunding, terracing, leveling, drainage, reclamation of ravine lands,
moisture conservation practices
 For purchase of seeds including high yielding/hybrid fertilizers, manure, pesticides,
fungicides etc.
 For meeting capital expenditure and working capital on units and dairy, poultry, piggery
etc. for construction of buildings, purchase of animal equipment, feeds, medicines,
vehicles etc
It is not Where We Reach, but more importantly How we reach mattersGo on- 887 -.

A Management Paradise (un) Ltd .Endeavor.I,MP

Structure of Rural Credit


The credit facilities are available to rural agriculturists and artisans through financial
and non financial institutions which are:
 Non Institutional
-

Professional money lenders

Agricultural money lenders

Relatives and friends

Traders and commission agents

Land lords and

Others

 Institutional
-

Government

Cooperative Banks and

Commercial banks

The non institutional credit sources are considered as exploitative and high cost system.
However, they are very much accessible and easily negotiable with the lenders. It is observed
that non institutional source of credit is continued to be an important source in rural areas.

Institutional lending or credit or loans refers to loans provided by financial institutions.

It is not Where We Reach, but more importantly How we reach mattersGo on- 888 -.

A Management Paradise (un) Ltd .Endeavor.I,MP

Commercial

Cooperative
Societies

Banks

Regional Rural
Bannks

Rural Branches

Branches

Long term Credit


(Investment credit)

Short term credit


(Production credit)

State Cooperative Banks


Federal Structure

Unitary structure
District Central
Cooperative Banks

State level
Agricultural and
Rural Development
Bank

Primary
Development
banks

State level
Agricultural and
Rural Development
Bank

Primary Agricultural
Credit Society

Branches

It is not Where We Reach, but more importantly How we reach mattersGo on- 889 -.

A Management Paradise (un) Ltd .Endeavor.I,MP

Characteristics of Indian Rural Economy and Rural Borrower and


the Related Problems
There are certain characteristic features of the Indian rural economy and the rural
borrower. Each of these features creates some hindrance in the effective development of
rural banking.
 The Indian rural socio-economic scene is still feudal in nature largely still in the midst of
illiteracy. Besides this, the Indian rural psyche is deeply entwined with the cultural ethos.
Problem for banking system:


There exists a traditional anti-loan psychosis and people prefer to avoid credit as far as
possible. Even when they do borrow, moneylenders have a traditional stronghold on the
minds of the rural borrowers.

The availability of credit from banks is dependent on a number of formalities. Even


literate rural customers prefer to avoid such complexities of documentation, restricted
working hours, loan amount, proximity factors, purpose of loan .

 Rural borrowing may be seasonal in nature due to the heavy dependence in these areas on
agriculture and allied activities. Timely availability of funds is crucial.
Problem for banking system:


The procedures involved in availing bank finance sometimes delay the actual receipt of
funds. The money if received late may be of no use to the borrower. The next time
finance is required he will approach a source that guarantees timely delivery of money
usually local moneylenders.

In such situations the cost of borrowing may be immaterial. This can be seen from the
fact that rural customers borrow heavily from moneylenders despite their astronomical
rates of interest.

 The economic profile of most rural borrowers is very weak. The average amount of credit
required is relatively low and savings deposited may be as low as Rs. 10 or 20 per month.

It is not Where We Reach, but more importantly How we reach mattersGo on- 890 -.

A Management Paradise (un) Ltd .Endeavor.I,MP


Problem for banking system:


Banks may not find operations economical, as sometimes the transaction and follow up
costs are more than the amount of credit. Rural banks and rural branches that are
compelled to operate in this milieu do so unprofitably

Repayment period of loans fixed by banks are shorter than required for the type of
activity financed.

Gestation period is not considered while fixing the due dates.

Due dates of repayment of loan installments were not fixed according to income
generation.

Working capital was not given on a need based approach or not released at the right time
for want of compliance of various conditions of sanction.

Besides there is also some amount of subsidized credit available to rural borrowers from
the Government. This further reduces the efficiency and independence of the system, as
it cannot generate enough to support operations let alone lower interest rates.

Compounding to this is the problem of non-repayment of loans.

Another problem is that banks generally do not give loans for consumption. In cases
where day-to-day living itself is at question, banks, their strict conditions on the use of
money borrowed and the numerous delays are avoided.

It is not Where We Reach, but more importantly How we reach mattersGo on- 891 -.

A Management Paradise (un) Ltd .Endeavor.I,MP


DABARD

Genesis
National Bank for Agriculture and Rural Development (NABARD) was established on 12 July
1982 by an Act of Parliament. The agriculture credit functions of the Reserve Bank of India
(RBI) and refinance functions of the then Agricultural Refinance and Development Corporation
(ARDC) were transferred to NABARD on its formation.

Mandate
NABARD is set up by the Government of India (GoI) as a development bank with the mandate
for facilitating credit flow for promotion and development of agriculture, small-scale industries,
cottage and village industries, handicrafts and other rural crafts. It also has the mandate to
support all other allied economic activities in rural areas, promote integrated and sustainable
rural development and secure prosperity of rural areas, as also for matters connected therewith
and incidental thereto.
It is not Where We Reach, but more importantly How we reach mattersGo on- 892 -.

A Management Paradise (un) Ltd .Endeavor.I,MP


Share Capital
The share capital of NABARD is Rs.2000 crore, subscribed by the GoI (Rs.550 Crore) and the
RBI (Rs.1450 Crore).

Management
The Management of NABARD vests with the Board of Directors. The Board of Directors of
NABARD comprises the Chairperson, Managing Director, representatives of RBI, GoI, State
Governments and
Directors nominated by the GoI.

Mission
Promote sustainable and equitable agriculture and rural prosperity through effective credit
support, related services, institution development and other innovative initiatives

Role
Facilitating credit flow for agriculture, rural infrastructure and rural development
Promoting and supporting policies, practices and innovations conducive to rural
development
Strengthening rural credit delivery system through institutional development

Functions in Brief

Credit and Credit Planning


v Preparation of district-wise credit plans annually that indicate exploitable potential available
for development through bank credit under agriculture, allied activities, Rural Non-Farm Sector
etc.
v Preparation of State Focus Paper based on district credit plans.
v Monitoring the flow of ground level credit.
v Issuing policy and operational guidelines to rural financial institutions (RFIs).

Financial Services
v Refinancing RFIs for providing loans for investment and production purposes in rural areas.
v Loans to State Governments for strengthening of cooperatives and also developing physical
and social infrastructure in rural areas.
It is not Where We Reach, but more importantly How we reach mattersGo on- 893 -.

A Management Paradise (un) Ltd .Endeavor.I,MP


v Support for micro-credit innovations of Non-Governmental Organizations (NGOs) and other
non-formal agencies.
v Monitoring and Evaluation of financed projects.

Promotion and Development


v Institutional development of client organizations.
v Capacity building in partner institutions.
v Supporting experimentation with new development models and practices in credit delivery.
v Dissemination of innovative products and ideas.
v Supporting Research and Development (R & D).
v Assisting RBI / GOI in formulation of policies relating to rural credit.
v Promotion of rural non-farm sector.
v Promotion of Kisan Credit Card (KCC) Scheme.
v Promotion of micro-credit innovations.
v Consultancy Services.

Supervision
v On-site inspection of cooperative banks (Coop Banks) and Regional Rural Banks (RRBs).
v Off-site surveillance of the health of Coop Banks and RRBs.

More about the Functions:


District Level Planning
v NABARD prepares Potential Linked Credit Plans (PLPs) for all the districts of the country. It
maps the potentials available for development in agriculture and rural sectors in the district and
projects credit requirements, taking into account long-term physical potential, availability of
infrastructure, extension services and marketing support and the strengths and weaknesses of the
RFIs in the district.

State Level Planning


v NABARD prepares a State Focus Paper for every State. This presents a comprehensive picture
of potentials available in the State for development of agriculture and allied sectors. It also
provides a road map of the opportunities available for further investments in these sectors. It can
be used by bankers and other agencies for preparing their action plans for making these
investments.
It is not Where We Reach, but more importantly How we reach mattersGo on- 894 -.

A Management Paradise (un) Ltd .Endeavor.I,MP


v State Credit Seminars are convened by NABARD annually where all agencies concerned viz.,
the
State Government, banks, NGOs, etc. participate and discuss policies and operational measures
required to be taken for tackling constraints in development of potentials available in agriculture
and allied sectors in the State.

Dational Level Planning


v NABARD facilitates policy decisions by GoI and RBI in the areas of credit flow to agriculture
and rural development.

Financial Services:
Short Term Credit
v Seasonal Agricultural Operations (SAO) to Coop Banks and RRBs.
v Marketing of Crops to Coop Banks and RRBs.
v Marketing and distribution of inputs like seeds, fertilizers, etc. to Coop Banks.
v Production and marketing activities of cottage, village and small-scale industrial cooperative
societies to Coop Banks.
v Production, procurement and marketing activities of primary/apex weaver societies and State
Handloom Development Corporations (SHDCs) to State Cooperative Banks (SCBs) and
Commercial Banks (CBs).
v Production, procurement and marketing activities of State Handicraft Development
Corporations (SHnDCs) to SCBs and CBs.
v Working capital requirements of pisciculture activities to Coop Banks and RRBs.
v Any other activity connected with rural /agricultural sector to Coop Banks and RRBs.

Medium Term (MT) Credit


v MT Conversion Loans (on account of crop loss due to natural calamities) to Coop Banks &
RRBs.
v MT (Non Schematic) loans for agriculture and allied activities to Coop Banks & RRBs.

Investment Credit
v Refinance to State Co-operative Agriculture and Rural Development Banks (SCARDBs),
SCBs, Scheduled Banks such as CBs and RRBs for:
Agriculture and allied activities
It is not Where We Reach, but more importantly How we reach mattersGo on- 895 -.

A Management Paradise (un) Ltd .Endeavor.I,MP


Financing artisans /Khadi Village Industries (KVIC)/ Rural Non Farm Sector, etc.
Rural housing

Direct Credit
v Loans to State Governments for share capital contribution to cooperative credit institutions.
v Loans under Rural Infrastructure Development Fund (RIDF) for rural infrastructure projects to
State Govts., Panchayati Raj institutions, SHGs and NGOs, etc.
v Co-financing of hi-tech/export oriented agriculture projects involving large outlays/ sunrise
technology
etc. with CBs.
v Bulk lending for micro finance activities and promotional projects to NGOs.

CREDIT PRODUCTS ADD CLIEDTS


Short Term (ST) Credit
v Seasonal Agricultural Operations (SAO)
In order to ensure availability of timely credit to farmers, banks follow production-oriented
system of lending. The system has features like assessment of credit needs based on area under
cultivation and crop wise scales of finance, extending credit for purchase of agricultural inputs,
etc.
Refinance is provided for production purposes at concessional rates of interest to SCBs and
RRBs by way of sanction of credit limits. Each drawal against the sanctioned credit limit is
repayable within 12 months.

v Marketing of Crops
Refinance facility is extended to Coop Banks and RRBs for meeting the marketing credit
requirements of farmers with a view to reducing incidences of distress sale and enhancing their
holding capacity. Each drawal against the sanctioned credit limit is repayable within a maximum
period of 12 months.

v Distribution of agricultural inputs


In order to ensure timely supply of agricultural inputs a line of credit is made available to Coop
Banks and RRBs for financing Apex/Primary Societies for stocking and distribution of

It is not Where We Reach, but more importantly How we reach mattersGo on- 896 -.

A Management Paradise (un) Ltd .Endeavor.I,MP


agricultural inputs by way of sanction of yearly limits. Each drawal is repayable within a period
of 120 days.

v Financing industrial societies


Refinance is provided to Coop Banks for financing production and marketing activities of
industrial cooperative societies engaged in one or more of the 22 approved broad groups of
cottage and SSIs.

v Weavers finance
Given the importance of handloom sector in rural employment generation, refinance is extended
to meet the working capital requirements of primary weavers societies and procurement,
stocking, distribution and marketing activities of Apex Weavers societies and SHDCs.

v Financing handicrafts sector


Rural craftsmen are provided support by financing SHnDCs through SCBs and CBs for
production, procurement and marketing of handicraft goods.

v Pisciculture
Refinance is extended to Coop Banks and RRBs for meeting working capital requirements of
fishermen engaged in Inland and Marine fishery activities.

v Dew line of credit for ST agricultural /allied and marketing activities


A new line of credit is extended to Coop Banks for financing agricultural/allied and marketing
activities against security of gold / security other than charge on crops.

It is not Where We Reach, but more importantly How we reach mattersGo on- 897 -.

A Management Paradise (un) Ltd .Endeavor.I,MP

v Eligible Institutions
NABARD provides refinance support to SCARDBs/ SCBs/ RRBs/ CBs/ Scheduled Primary
Urban Cooperative Banks/ North East Development Finance Corporation Ltd. (NEDFi) etc.
against their Investment Credit in the Rural Sector.
During 2003-04 was Rs.7605.29 crore The target for refinance support for Investment Credit for
the year 2004-05 is set at Rs. 8800 crore

Medium Term credit


v Medium Term (Conversion) Loans
When crop loss on account of natural calamities is substantial, affecting the farmers ability to
repay production credit dues to banks, refinance by way of medium term loans is granted to
Coop Banks and RRBs to enable them to convert the S T loans of farmers into medium term
loans. The repayment period of the converted loan is three years, which may go upto a
maximum of 7 years in case of recurring calamities.

v Medium Term (Don - Schematic) Loans


Refinance is provided to SCBs and RRBs for financing farmers to acquire productive assets for
certain approved agricultural investment purposes.
It is not Where We Reach, but more importantly How we reach mattersGo on- 898 -.

A Management Paradise (un) Ltd .Endeavor.I,MP


Investment Credit
Investment credit leads to capital formation through asset creation. It induces upgrading of
technology resulting in increased production, productivity and incremental income to farmers
and entrepreneurs.
Direct Credit
Loans to State Governments
v Supporting Cooperatives
In order to strengthen the owned funds position of cooperative credit institutions NABARD
provides loans to State Governments to contribute to the share capital of these institutions.

v Rural Infrastructure Development


With the objective of assisting State Governments in the completion of ongoing rural
infrastructure projects and taking up new projects, the Rural Infrastructure Development Fund
(RIDF) was set up in NABARD in 1995-96 with contributions by way of deposits received from
CBs. The shortfall in agri/priority sector lending deposited by CBs with NABARD forms the
corpus of RIDF. The total corpus of RIDF, covering RIDF I (1995-96) to IX (2003-04), is
Rs.34000 crore. Sanctions under all tranches of RIDF as on 31 March 2004 were Rs.34678.07
crore and disbursements Rs. 21067.17 crore.

v Anticipated Benefits
It is anticipated that the projects sanctioned upto 31 March 2004 under RIDF would result in:
Creation of additional irrigation potential in 85.52 lakh ha. Addition of 152483 km of rural
road network & 296000 meter bridge length. Contribution to the GDP to the tune of Rs. 9712
crore. Generation of recurring employment of 44.59 lakh jobs and non-recurring employment of
12095 lakh person days due to increased irrigation. Generation of non-recurring employment
expected from non-irrigation projects: 14354 lakh person days.

Co-financing
To ensure substantial credit flow to agriculture and rural sector and to instill confidence in banks
for

financing

hi-tech/export

oriented

agriculture

projects

involving

large

financial

outlays/sunrise technologies, etc., NABARD has entered into agreements for co-financing with
12 CBs on a risk- sharing basis. During 2003-04, 7 projects were sanctioned under this
agreement in areas such as floriculture, organic farming, milk processing, ethanol production,
It is not Where We Reach, but more importantly How we reach mattersGo on- 899 -.

A Management Paradise (un) Ltd .Endeavor.I,MP


infrastructure development and forestry, involving total financial outlay of Rs. 118.90 crore and
NABARDs support of Rs. 35.97 crore.

Bulk lending / Revolving Fund Assistance


NABARD provides Bulk lending facilities to NGOs. As on 31 March 2004, 32 agencies have
been sanctioned assistance of Rs. 27.32 crore against which Rs.15.20 crore has been disbursed.

Thrust Areas:
Kisan Credit Card
As a pioneering credit delivery innovation, KCC Scheme aims at provision of adequate and
timely support from the banking system to the farmers for their cultivation needs including
purchase of inputs in a flexible and cost effective manner. As on 31 March 2004, cumulatively
413.79 lakh cards were issued to the farmers by Coop Banks, CBs and RRBs. A Personal
Accident Insurance Scheme is available to KCC holders to cover them against accidental
death/permanent disability.

Don Farm Sector


Rural Non Farm Sector (RNFS) is a thrust area of NABARD because of its employment
generation potential in rural areas. NABARD provides refinance as well as promotional support
for development of the sector.

v Credit Support
The refinance products of NABARD cover manufacturing, processing and service activities in
the Small Scale Industries (SSI) sector with focus on cottage, village and tiny industries, rural
artisans and Rural crafts as also rural housing finance. Refinance for Rural Housing is extended
for financing new constructions and renovating existing constructions in rural areas. Refinance
is also extended to Government sponsored programmes like Swarnjayanti Gram Swarojgar
Yojana (Industry, Service and Business component) and Prime Ministers Rozgar Yojana.

It is not Where We Reach, but more importantly How we reach mattersGo on- 900 -.

A Management Paradise (un) Ltd .Endeavor.I,MP


v Promotional Support
Promotion of RNFS has been recognized as an important and necessary adjunct to the
refinancing function. The objective of p r o m o t i o n a l programmes is to establish replicable
models for generating/enhancing opportunities for employment and income generation in rural
areas in a sustainable, demonstrative and cost effective manner by providing grant/revolving
fund assistance etc., to NGOs, Voluntary Associations(VAs), Trusts and other Promotional
Organizations.
Achievements upto 31 March 2004
Refinance under RNFS Rs. 15455 crore
5465 REDPs supported over 1.51 lakh rural youths
80 districts covered under DRIP
55 rural clusters identified for development

v Rural Promotion Corpus Fund (RPCF)


RPCF has been established in NABARD with the objective of supporting innovative
promotional activities in a flexible manner leading to promotion of viable enterprises,
sustainable employment opportunities and improved credit flow to rural enterprises. Promotional
activities supported include

v Rural Entrepreneurship Development Programmes (REDP)


NABARD extends grant assistance to agencies with professional competence for conducting
entrepreneurship development programmes for unemployed rural youth. A comprehensive
strategy was adopted to cover one lakh potential entrepreneurs under REDP over a period of 5
years (1999-2000 to 2003-04) through institutionalization of REDP. NABARD has supported
more than 5465 REDPs since inception and trained around 1,51,000 rural youth with grant
assistance of Rs. 11.91 crore.

v District Rural Industries Project (DRIP)


The project was launched by NABARD in 1993-94 in 5 select districts with the objective of
generating sustainable employment opportunities through enhanced credit flow to rural non farm
sector together with promotional measures in a coordinated manner. Under the project, a
common action plan is prepared and implemented in coordination with all government and nongovernment agencies involved in promotion of rural industrialization in the district. In view of
the success of the programme, it was decided to extend the project to 100 more districts in 5
It is not Where We Reach, but more importantly How we reach mattersGo on- 901 -.

A Management Paradise (un) Ltd .Endeavor.I,MP


years commencing from the year 2001-02. During 2003-04, 16 districts were covered bringing
the total districts covered to 80. The programme has facilitated setting up of more than 4 lakh
units and generating employment opportunities for over 9 lakh persons as on March 2003.

v Cluster Programme for Rural Industries


NABARD is actively associated with the National Programme for Rural Industrialization
(NPRI) launched by GoI for development of rural clusters and has identified 65 clusters in select
districts with the involvement of VAs. Promotional programmes have been launched in 55
clusters so far. Different aspects such as skill and technology up gradation, design
diversification, timely and adequate supply of quality raw materials, marketing, etc. are taken
care of after conducting diagnostic studies and preparing action plans within the time frame with
active involvement of all Government and development agencies, banks, NGOs and artisans.
The programme is estimated to benefit 16,000 artisans and small entrepreneurs. NABARD has
sanctioned Rs.155 lakh to the implementing agencies for carrying out promotional interventions
in NABARD adopted clusters.

v Skill Upgradation & Design Development for Handloom Weavers (SUDHA)


The scheme was introduced in 2002-03 with a view to upgrading skills of traditional handloom
weavers and supporting development of new designs for handloom products for improving their
marketability both within and outside the country. Under the scheme, grant assistance is given to
Apex Handloom Co-operative Societies and NGOs involved in promotion of handloom sector
for market surveys, design development, skill upgradation training, engaging design consultants
and marketing efforts. During the year 4 programmes with grant assistance of Rs. 10.71 lakh
were sanctioned.

v Swarozgar Credit Card (SCC) Scheme


The SCC Scheme formulated by NABARD in consultation with RBI and GoI envisages
adequate and timely credit, both working capital and block capital, to small artisans, handloom
weavers, service providers, fishermen, self-employed persons, rickshaw owners and other micro
entrepreneurs, in rural and urban areas in a flexible, hassle free and cost effective manner from
the banking system. The facility also includes a reasonable component for consumption needs.
As on 31 March 2004, 28,925 cards were issued by CBs, Coop Banks and RRBs involving
credit limit of Rs. 64.26 crore.

It is not Where We Reach, but more importantly How we reach mattersGo on- 902 -.

A Management Paradise (un) Ltd .Endeavor.I,MP

Microfinance Initiatives
The SHG Bank Linkage Programme started as a pilot project in 1992-93 for linking 500 Self
Help Groups (SHGs) with banks with the objective of extending formal banking services to the
unreached rural poor by evolving a supplementary credit delivery strategy in a cost effective
manner. Today, it covers 10.79 lakh SHGs, making it the largest microfinance programme in the
world in terms of sustainability and outreach.

Partners
The social intermediation for forming and nurturing SHGs is handled by a large number of
NGOs and several community based organizations like Farmers Clubs, local bodies, field level
workers of government agencies and bank staff. The major partners are banks (504 nos.), NGOs
and other agencies (2800 nos.).

Achievements as on 31 March 2004


v Physical
3,61,731 new SHGs financed by banks during the year 2003-04. 1,71,669 existing SHGs
provided with repeat finance.
1.67 crore poor families i.e. approximately 8.09 crore people estimated to have been assisted
through bank credit upto 31 March 2004.
Approximately 90 % of SHGs are exclusively women groups.

v Financial
Bank loan of Rs. 1855.53 crore disbursed during the year 2003-04 to new SHGs and Rs.
697.79 crore to existing SHGs.
Cumulatively Rs. 3904.20 crore bank loan disbursed to SHGs.
Average bank loan per SHG works out to Rs. 36,180.
Over 95% on-time repayment of loans.

These are some of the improvements till date after the formation of NABARD.

It is not Where We Reach, but more importantly How we reach mattersGo on- 903 -.

A Management Paradise (un) Ltd .Endeavor.I,MP

DCC Bank:
These operate at the district levels and hence they are known as District Central Cooperative
Bank. The most important function of the bank is that it is affiliated to all the base level societies
at the primary level. These DCCBs are again affiliated to State Cooperative Banks.
The number of DCCBs as on 31st March, were 367 operative in the country in coordination
with a huge number of the PACs.

State Cooperative Banks (SCBs):


The State Cooperative Banks form the apex of the Cooperative Credit Structure. The SCBs are
in direct control of the DCCBs in the State. They are a vital link between Nabard and Dccbs.
They are not only interested in helping the rural cooperative credit movement but also in
promoting other cooperative ventures while extending the principles of Cooperation. On an
average around 29 state cooperative banks lend about Rs. 34, 550 crores annually to central
cooperative banks and primary cooperative societies.

It is not Where We Reach, but more importantly How we reach mattersGo on- 904 -.

A Management Paradise (un) Ltd .Endeavor.I,MP

EPICenter
"Today is the best day of your life"
Today is the best day of your life. Why? Because you are alive. Because today you can live.
Because today you can choose to take control of your life. Every morning when you wake-up there
lies before you a golden opportunity, a new day.

Out of this day you make your memories, you live your dreams of yesterday, you dream of
tomorrow, you live your life.

Realise that what you do today becomes the building blocks of your past and lays the foundations
for your future. Today is the day that you dreamed of yesterday. Today will be one of the "good old
days" you reminisce over tomorrow.

Imagine that there was a bank that credited your account each morning with Rs. 86,400.

It carried over no balance from day to day. Every evening it deleted whatever part of the balance
you had failed to use during the day. What would you do? DRAW IT ALL OUT, of course!

Well, each of us has such a bank. Its name is TIME. Every morning, it credits you with 86,400
seconds. Every night it writes off, as lost, whatever you failed to invest to good purpose. It carries
over no balance. It allows no overdraft, you cannot borrow from tomorrow. You must live on
today's deposit and invest it so as to get from it the utmost in health, happiness, and success. The
clock is running. Make the most of today.

Thoughts to Live/Work By.:


It is not Where We Reach, but more importantly How we reach mattersGo on- 905 -.

A Management Paradise (un) Ltd .Endeavor.I,MP

7)

a) Labour Cooperatives:
The origin of Labor Cooperative was in the year 1948. that was when the Mumbai Government
made the special provision for the labor contract cooperatives, wherein any contract above Rs.
20, 000 was awarded to the cooperative.

The structural Framework of the Labor Cooperatives is as follows:


i)

State Federation: This is apex level of all the labor cooperatives. Every state has its
own labor coop. head.

ii)

District Federations: they are the intermediate link between primary societies and
State Federations.

iii)

Primary Labour Cooperatives Societies: they are formed for the benefit of the
laborers so as to enable them to collectively bargain for their services and prevent
exploitation at the hands of the contractors and the middlemen.

Earlier the laborers were exploited by the middlemen or the contractors who pocketed most of
the amount which the laborers deserved to receive. That is why they were left with the meager
amount of the money on their hands. Such was the case of the landless laborers, the marginal
and non-marginal farmers and the contract workers. Thus the people of the weaker sections of
the society and they became poor day by day.

Advantages of the Labor Cooperatives:


a) Labor cooperatives directly deal with the work awarding agencies so that the laborers are
saved from the exploitation.
b) They are a good learning centers for their members because they promote coeducation
amongst members.
c) There is an equitable distribution of the money.

It is not Where We Reach, but more importantly How we reach mattersGo on- 906 -.

A Management Paradise (un) Ltd .Endeavor.I,MP

b) SIDBI:
 SIDBI was setup as a wholly owned subsidiary of IDBI by an act of parliament in
1989, and it commenced its operations form 2nd April, 1990.
 SIDBI basically stands for Small Industries Development Bank of India. The
main role of SIDBI was promotion and functioning of small scale industries and
cooperative organizations.
 After completing 12 years of service, it has been delinked from SIDBI act with
effect from 27th March, 2000
 It hah been helping the small scale industries including the tiny sector and village
or cottage industries through suitable schemes tailored to help them in setting up
of new projects, expansion, modernization and rehabilitation of existing units.
 The following are the functions of SIDBI: a) Refinance
c) Training

b) Direct Assistance

d) Development

 To provide the seed capital and National Equity Fund for promotion of
Cooperatives.
 To provide direct assistance for funding Cooperatives.
 Special loans for technology upgradations.
 Refinancing term loans for cooperatives by SFCs, SIDOs, and Commercial
Banks.
 SIDBIs Venture Capital Fund promotes new ventures in Agro-processing plants.

d) Housing Cooperatives:

Housing Co-operative
Food, clothing and shelter are the prime necessity of human beings. Housing is a significant
concern of every individual. There is an acute problem or shortage of housing in India due to
the following reasons:
1.

Tremendous growth in population.

2.

Migration of people from rural areas to urban areas.

It is not Where We Reach, but more importantly How we reach mattersGo on- 907 -.

A Management Paradise (un) Ltd .Endeavor.I,MP


3.
Discontinuance of joint family system leading to requirement of separate house to each
family unit.
4.

Highly speculative trend in cost of land.

5.

Hike in prices of building material resulting in increase in cost of house everyday.


Co-operative housing is a solution to the above given problems because it is a wellorganized significant movement of the common men.

Features
1.

They are legally established associations of persons or members.

2.

The members democratically control them.

3.

Co-operatives societies provide for minimum required facilities like drinking water, tank
water, waste disposal, sanitation, electricity, common car parking etc.

Three Tier System Of Co-operative Housing (State Level)

1.

Primary Co-operative Housing Society


They are of 3 types as per the classification given in the Rule 10 of Maharashtra State Cooperative Society Act 1960. The sub-classification is on the basis of their objectives and
activities. The 3 main types are as follows:

a.

Tenant Ownership Housing Society

It is not Where We Reach, but more importantly How we reach mattersGo on- 908 -.

A Management Paradise (un) Ltd .Endeavor.I,MP


Housing society where the land is held either on leasehold or free hold basis by the societies
the houses are owned by the members. Due to the ownership of land with the societies the
members become tenants, but as they own the houses. This form is called as TenantOwnership.

b.

Tenant Co-Partnership Housing Society


Housing society where land and buildings are held by the society and only the flats or shops
are owned by the members. There is a partnership between the society and the members in
the ownership of flat or shop or buildings.

c.

Other Housing Society


Housing co-operative society not coming under the 2 above mentioned categories are
included in other housing society.
E.g. House Mortgage Society, House Construction Society etc.
Constitution of Co-operative Housing Society
Every housing society is managed as per the byelaws of the society and the provisions in the
co-operative act. Incase of primary co-operative housing society the below given points have
to be covered in the byelaws.

1.

Name and registered address of the society.

2.

Raising of funds by way of entrance fees, share capital, loan, donations, deposits from
member and contribution from the members for construction.

3.

Membership (Eligibility Criteria, Minimum number of shares to be purchased, Maximum


number of shares which an individual can purchase, rights of members etc.)

4.

Share Capital (Denominations of shares, transfer of shares, refund of share capital etc.)

5.

Management
a.

General Body Meetings

b.

Managing Committee

6.

Maintenance of record as per co-operative act.

7.

Proper appropriation and distribution of profits.

It is not Where We Reach, but more importantly How we reach mattersGo on- 909 -.

A Management Paradise (un) Ltd .Endeavor.I,MP


Managing Committee (Functions)
1.

Admission of members.

2.

Transfer of shares.

3.

Preparation of annual budget the annual statement of accounts.

4.

To raise funds for the working of the society.

5.

Investment of surplus funds.

6.

Supervision over staff.

7.

Appointment of sub-committees.

8.

To prepare draft amendments to byelaws.

9.

To supply information to government, co-operative department, financing institution.

10. To arrange for election of next committee well in time.


District Level Housing Federation
It is a federal society and all types of co-operative housing society are eligible for its
membership. They are affiliated to the federation and it is compulsory for primary cooperative housing society to become members of the district federations.
Objectives
1.

To give legal and technical guidance to the member society.

2.

To help the proposed co-operative housing society in registration.

3.

To help the co-operative housing society in getting approval of plans from Town planning
department and completion certificate from municipal authority.

State Level Housing Co-operative Society

It is not Where We Reach, but more importantly How we reach mattersGo on- 910 -.

A Management Paradise (un) Ltd .Endeavor.I,MP


1.
To guide the primary housing co-operative society in the state regarding legal, technical
and management problems.
2.

To help the housing societies in getting better building material at reasonable rates.

3.

To coordinate the activities of primary and district level co-operative federations.

4.

To undertake publicity and propaganda to promote organization of co-operative housing


society.

5.

To develop small housing complexes in the state.

Dational Co-operative Housing Federation (DCHF)


The National Co-operative Housing Federation (NCHF) was formed in the year 1969 as per the
recommendations of a study group on co-operative housing. The state level and district level
housing societies as well as the state level institution financing the housing societies can become
a member of the National Co-operative Housing Federation (NCHF).
Objectives
1.

Publicity and propaganda of co-operative housing movement by publishing books,


brochures etc.

2.

To undertake research and development work for reduction in the construction cost.

3.

To establish financial institution at state level to help the primary co-operative housing in
getting loans at lower rate of interest.

4.

To involve institution like LIC and other central level financing institution in the cooperative housing movement.

In Maharashtra the following institutions are active in helping the co-operative housing
societies:
1.

Maharashtra Housing Finance Co-operative Society.

2.

Housing Development Finance Corporation (HDFC)

It is not Where We Reach, but more importantly How we reach mattersGo on- 911 -.

A Management Paradise (un) Ltd .Endeavor.I,MP


3. Housing And Urban Development Corporation (HUDCO)
4.

Maharashtra Housing Area Development Authority (MHADA)

e) Regional Rural Banks:

Regional Rural Banks


Cooperatives have been encouraged since 1904 and the commercial banks were made to accept
the responsibility of financing rural economic activities from 1968 under social control to relieve
the poor peasants from the clutches of money lenders. Previously banks felt that financing
agriculture was not their job and that his responsibility would be withdrawn soon. So the results
of control and the working of cooperatives had not been significant. Hence, the government of
India had nationalized the 14 major commercial banks with the objective to channelise the
resources the resources of commercial banks to rural areas. The impact of bank nationalization
on the growth of scheduled commercial banks in rural areas is clear: the share of rural bank
offices in total bank offices jumped from 17.6 per cent in 1969 to 36 per cent in 1972. The share
rose steadily thereafter, and attained a peak of 58.2 per cent in March 1990. Consequent to the
adoption of intensive agricultural programmes under IADP and DPAP, Green Revolution etc.
the demand for financial inputs has increased enormously in the rural areas. Therefore it was felt
that cooperative and commercial banks alone would not be in a position to meet all the credit
needs of the expanding rural economic sector. Between 1966 and 68 various committees
suggested that the rural credit structure was weak, therefore some system of rural banks should
be created to fill up the credit gap in rural areas. These banks should extension in the rural areas
for rural people as such they must be located in rural areas and understand the rural economic
environment. Thus Regional Rural Banks were a new type of institution, which combined
a. Local feel and familiarity with rural possess problems which co-operative banks have.
b. Degree of business organization ability to mobilise deposit, access to money market and
modernised outlook which commercial banks have.
The Government of India promulgated the Regional Rural Banks Ordinance on 26th September
1975, which was later replaced by the Regional Rural Bank Act 1976. At to the end of June
1985, 183 Regional Rural Banks with a network of 10,245 branches have been opened in the
states of the Indian Union. The total number of Regional Rural banks functioning in the country
as at the end of June 1999 was 196 covering 451 districts spread over 23 states with the network
It is not Where We Reach, but more importantly How we reach mattersGo on- 912 -.

A Management Paradise (un) Ltd .Endeavor.I,MP


of 14,467 branches These banks have been established by the Government of India in terms of
the provisions of Regional Rural Banks Act, 1976. The distinctive feature of a rural bank is that
though it is a separate body corporate with perpetual succession and common seal, it is closely
linked with the commercial bank which has sponsored the proposal to establish it. The central
Government, while establishing a rural bank at the request of a commercial bank, specifies the
local limits within which it shall operate. The rural Bank may establish its branches or agencies
at any place within the notified area.

Objectives of Regional Rural Banks


Regional Rural Banks were established with the following objectives in mind:
i.)

Taking the banking services to the doorstep of rural masses, particularly in hitherto
unbanked rural areas.

ii.)

Making available institutional credit to the weaker sections of the society who had by far
little or no access to cheaper loans and had perforce been depending on the private
money lenders.

iii.)

Mobilize rural savings and channelise them for supporting productive activities in rural
areas.

iv.)

To create a supplementary channel for the flow the central money market to the rural
areas through refinance

v.)

Generating employment opportunities in rural areas and bringing down the cost of
providing credit to rural areas.

With these objectives in mind, knowledge of the local language by the staff is an important
qualification to make the bank accessible to the people

Capital Structure
The authorized capital of each Regional Rural Bank is Rs.1crore, divided into 1 lakh fully paid
up shares of Rs.100 each. The Central Government may, after consultation with the Reserve
Bank and the sponsoring bank, increase or reduce such authorized capital, but it shall not be
reduced below 25 lakhs. The issued capital of each Rural Bank is Rs.25lakh. Fifty percent of the
capital issued by a Rural Bank is subscribed by the Central Government and thirty five percent
by the sponsoring Bank. The Board of Directors of a Rural Bank may, after the consultation
with the Reserve Bank and the sponsoring Bank and with the prior approval of the Central
It is not Where We Reach, but more importantly How we reach mattersGo on- 913 -.

A Management Paradise (un) Ltd .Endeavor.I,MP


Government, increase the issued share capital from time to time. The additional capital shall be
subscribed in the same proportion as is specified above. The shares if the Rural Bank shall be
deemed to be included in the securities enumerated in Section 20 of the Indian Trusts Act, 1882
and shall be deemed to be approved securities for the purposes of the Banking Regulation Act,
1949.

Business of a Rural Bank


A Rural Bank carries on the normal banking business i.e. the business as defined in Section 5 (b)
of the Banking Regulation Act, 1949 and engages in one or more forms of business specified in
Section 6(1) of that Act. A rural bank may, in particular, undertake the following types of
business, namely:
 The granting of loans and advances, particularly to small and marginal farmers and
agricultural labourers, whether individual or in groups and to co-operatives societies
(including agricultural marketing societies, agricultural processing societies, Cooperative farming societies, primary agricultural credit societies or farmers service
societies) for agricultural purposes or agricultural operations or for other connected
purposes.
 The granting of loans and advances, particularly to artisans, small entrepreneurs and
persons of small means engaged in trade, commerce or industry or other productive
activities within the notified area of a Rural Bank.

Management of Regional Rural Banks


The Management of Regional Rural Banks is largely governed by the RBIs Act, 1976, Banking
Regulation Act 1949 and the guidelines of RBI and NABARD and sponsor banks. The general
superintendence, direction and management of affairs and business of RRBs are vested in Board
of Directors. They exercise the powers and discharge all the functions of the RRBs. In
discharging its functions, the Board of Directors act on business principles and shall have due
regard to public interest.
The Board will consist of a chairman and not more than 8 directors. The Central Government
will appoint a Chairman and three Directors, the concerned State Government nominates not
It is not Where We Reach, but more importantly How we reach mattersGo on- 914 -.

A Management Paradise (un) Ltd .Endeavor.I,MP


more than two Directors and the sponsor bank will nominate not more than three Directors. The
Chairman is responsible for the overall management of the management of the bank and hold
office for a period of 5 years. The Chairman is required to devote the whole time to the affairs of
the RRBs and subject the superintendence, control and jurisdiction of the board of directors. The
tenure of office of a director will be 2 years and he shall hold office until his successor is
nominated and will also be eligible for renomination.
The RRBs Act has also facilitated the creation of business committees by the Board of
Directors. These committees may consist wholly of directors or wholly of other persons or partly
of directors and partly of other persons for such purpose as it may decide. Most RRBs constitute
Business Committees for examining the feasibility of advances, mobilization of deposits,
deployment of funds in other institutions and to find ways and means of recovery performance.

Banking Environment
Banking Organization is an open adaptive system. It has its own internal and external
environments. Internal environments of RRBs consist of Banking, Personnel, Public Relations
etc. The external environment consists of uncontrollable economic, social, political and legal
factor governing the success or otherwise of RRBs
The RRBs external Environment consists of:

RRBs
Act
Banking
Regulation
Act

General
Laws
National
Politics

Religion

Legal
Environment

Social
Environment
RRBs

Political
Environment

Economic
Environment

Regional Politics
Financial
Markets

Education

Culture

Competition
from other banks
Economic
Administration and
relations with other
institutes

It is not Where We Reach, but more importantly How we reach mattersGo on- 915 -.

A Management Paradise (un) Ltd .Endeavor.I,MP

RRBs external Environment comprises of:


1. Legal Environment dealing with rules, regulations and legislative measures such as
General Laws, RRBs Act, Banking Regulation Act etc.
2. Economic environment consisting of change in economic activity such as competition,
changes from other banks, financial markets and the prescriptions of the lead banks also
effect the working of RRBs. Change in the economic environment will affect refinance
from NABARD.
3. Political environment dealing with Regional and national Politics. RRBs activities are
affected by monetary and fiscal policies of government.
4. Social Environment describing the religious activities, social attitudes, behavior,
education and deep-rooted connections. This plays a major role in rural villages where
due to illiteracy and poverty, social and cultural forces influence business patterns.

Progress of Regional Rural Banks


The RRBs were inaugurated on 2nd October 1975. Since the inception of these banks till June
1985, 185 banks have been operating through 8250 branches. Their deposits and advances
totaled Rs.1009 crores and Rs.1159 crores respectively, at the end of June, 1985. There are 196
RRBs now in the states covering 495 districts with 14,311 branches. The number of profitmaking RRBs rose to 172 during 2000-01 from 44 during 1996-1997 and their profit also rose to
Rs 681 from Rs 69.68 crore during this period The credit-deposit ratio was 111 % which is
much beyond the capacity of commercial banks to achieve. More than 90% of the outstanding
advances have gone to the weaker sections like small and marginal farmers and landless
laborers.
According to Mr. K.S.V. Krishnamachari, Dy. General Manager of the RRBs Dept. of the State
Bank of India, the future of the RRBs in their present form is at stake and recent press reports
indicate that the Government of India is seriously considering a proposal to set up a `National
Rural Bank' amalgamating the existing 196 RRBs in the country.

It is not Where We Reach, but more importantly How we reach mattersGo on- 916 -.

A Management Paradise (un) Ltd .Endeavor.I,MP

Indicators

1994

1995

1996

1997

1998

1999

Number of Commercial Banks

276

284

288

296

300

301

(a) Scheduled Commercial Banks

272

281

287

295

299

301

of which: Regional Rural Banks

196

196

196

196

196

196

(b) Non-Scheduled Commercial Banks

Number of Bank Offices in India

63755

64234

64937

65562

66306

67041

(a) Rural

33126

33021

32982

32929

32890

32894

(b) Semi-Urban

13374

13581

13832

13979

14208

14390

(c) Urban

9533

9717

9964

10263

10555

10816

(d) Metropolitan

7722

7915

8159

8391

8653

8941

3868.6

4338.2

5056

6054.1

7222

Deposits of Scheduled Commercial 3236.3


Banks in India (Rs. crore)
Credit

of

Scheduled

Commercial 166844 211560 254015 278401 324079 368837

Banks per office (Rs. lakh)

It is not Where We Reach, but more importantly How we reach mattersGo on- 917 -.

A Management Paradise (un) Ltd .Endeavor.I,MP

Future of Rural Banking


As per the estimate of Agricultural Credit Review Committee (Khusro Committee) the demand
for agricultural credit in 2001-2002 would be of the order of Rs. 1,10.873 Crores (Rs. 53,534
Crores for short terms & Rs. 57,339 Crores for term loans) & the supply of institutional credit
would be Rs. 89,447 Crores implying a shortfall of Rs.21,426 crores. The demand for finance
will also go up on account of intensive use of modern technology, increase in consumption of
fertilizers, insecticides & pesticides, creation of irrigation facilities, growing use of high yielding
variety crops & diversification of agriculture to various allied activities. Despite the multi
agency approach adopted for providing rural credit, certain inbuilt formalities viz.
documentation, restricted working hours, loan amount, purpose of loan & proximity factors have
been causing inconvenience to the deserving rural poor. Yet, exploitation of the rural poor
continues by the local money-lenders. Of this, the formal sector accounts for about 20 %.
The nature of this segment demands a level of customization that the formal banking network
has failed to provide. The uniform treatment meted out and the centralized structure of the
system are the basic causes for the failure to cultivate a healthy banking habit in the rural areas.
All these causes and effects, create a situation where an innovative means of credit
delivery is called for. The process of customization that is the need of the day has been
titled micro banking
Micro finance has been defined by the task force set up by the NABARD as "provision of thrift,
credit and other financial services and products of very small amounts to the poor in rural, semiurban or urban areas for enabling them to raise their income levels and improve living standards.
Micro finance institutions can include NGOs (Non-Government Organisations), co-operatives,
banks (commercial, RRBs, other nationalised and public sector banks) and NBFCs (NonBanking Financial Companies). The NABARD felt that banks would be unable to efficiently
organize such grass-root level groups and thus NGOs and Voluntary Agencies were introduced
into the picture. New micro credit companies such as Basix and the SEWA-aided bank represent
a primarily NGO-driven effort to charge market linked, risk adjusted rates of interest on small
It is not Where We Reach, but more importantly How we reach mattersGo on- 918 -.

A Management Paradise (un) Ltd .Endeavor.I,MP


loans to small borrowers. At the same time they ensure hurdle free access to borrowers and high
repayment rates for themselves.
In addition to financial intermediation, many micro finance institutions provide social
intermediation such as group formation, training in financial literacy and management
capabilities. Micro finance is therefore not just a banking tool but also a development tool.
Along with benefits to the rural population, the financial institutions advancing the credit also
enjoy better recovery rates.
The most prevalent method of providing micro finance in India is through Self-Help
Groups (SHGs
A SHG is a group of individuals ranging from 5 to 20 members, who come together for a
mutually beneficial purpose. They are homogenous in some respect and have certain pre-defined
social binding factors.


Members of a SHG contribute to a common fund from which collateral-free loans are given
to needy members as per the group decisions.

After at least 6 months if a bank is convinced, the SHG can become eligible for linkage to
the bank for availing credit and can open a savings account in its name and can receive up to
4 times its savings balance as credit. The members of the SHG in turn receive credit as per
their needs. This linkage was introduced by the NABARD in 1991-92 through pilot project.

The SHG decides the rate to be charged to its members.

Similarly, the bank negotiates about appropriate repayment period with the SHG and the
SHG decides on the repayment schedule for its members, generally in weekly installments.

If members require larger amounts of loans they can approach the bank for individual loans,
with the SHG accepting responsibility for proper credit utilization, repayment by the
member and monitoring of the same

Micro finance is thus a potent method of rural credit delivery with tremendous potential for
serving the rural masses.

Thoughts to Live/Work By.:


It is not Where We Reach, but more importantly How we reach mattersGo on- 919 -.

A Management Paradise (un) Ltd .Endeavor.I,MP

Management Of Cooperatives November 2003


Section I
Concepts:
1) Open Membership:
Open membership implies that there shall be no restrictions on the admission of members, open
to all persons who are able to use their services and willing to accept the responsibilities of
members, without gender, social, racial, political or religious discrimination. It seeks a
homogenous interest of the members who come together for a mutual benefit.
This principle is undergoing changes with time. A little amount of compulsion and restriction is
imposed for the smooth working and speed of the movement. The gates of membership cannot
be kept open for persons with vested interests e.g. Moneylender, as a member of credit society.
A person can be compelled to join or withdraw, from such a co-operative only on a larger
interest.

2) Professionalisation of cooperatives:
Refer paper 2002

3) Benefits of Cooperatives to members:


1.

It is a voluntary organization. It is an association of persons (one person one vote)

2.

It is democratic, every individual has a say in the co-operative society.

3.

It is an undertaking

It is not Where We Reach, but more importantly How we reach mattersGo on- 920 -.

A Management Paradise (un) Ltd .Endeavor.I,MP


4.
Less stress on Profits because the main objective is to meet common economic ends
through mutual help.
5.

It is not based on Exploitation

6.

There is no place for middlemen

7.

Common objective of people

8.

Concept of equality

9.

Wider interest of Community

10. No outside interference


11. Each for every one (self help through mutual help)

4) Role of Leadership in Cooperatives:


Refer Paper 2004

5) Role of Registrar in Cooperatives:


Refer paper 2001

EPICenter
"Whatever you are doing, act as if you want to do it, act as if it was important."

You owe it to yourself to develop an attitude that will get you positive attention. An attitude that
supports you. An attitude that allows you to be alert to opportunities. An attitude that causes
other people to want to deal with you, so from today
It is not Where We Reach, but more importantly How we reach mattersGo on- 921 -.

A Management Paradise (un) Ltd .Endeavor.I,MP


"Whatever you are doing, act as if you want to do it, act as if it was important."

Case Study
In Paltan village, most of the farmers grow sugarcane as the main crop. They had to travel more
than 50 km. to sell to the nearest factory. Considering this difficulty they decided to establish a
sugar factory in their village. They enrolled all the farmers in their village and the neighboring
area as the members of cooperative sugar factory. They also purchased a second hand sugar
plant and started a sugar factory. Initially all the farmers were happy. However during the
crushing season the farmers had to wait for more than 3 days and consequent to the delay they
got lesser price of their produce. In the second year the plant failed and all the farmers had to
suffer a huge loss. The government then appointed an administrator to look after the factory.
Also refer to paper 2001, the Critical factors in the Problems of Sugar Industries.

a) What are the deficiencies in establishing the sugar factory?


Solution:
1) The most important deficiency of the case is that the farmers established the sugar
factory not because of entrepreneurial skills but because of need. In other words we can
say that it was a forced decision. Hence because of this everybody is always concerned
about their own produce and the meaning of cooperative is lost.
2) They enrolled all the farmers growing the crop; hence this is sounds quite by force. The
test of the cooperative is that it is based on voluntary participation.
3) A sugar factory does not only mean setting up a sugar processing plant, but it also
includes many other supplementary things like warehousing, large vacant areas for
bullock carts, treatment of waste materials etc.

It is not Where We Reach, but more importantly How we reach mattersGo on- 922 -.

A Management Paradise (un) Ltd .Endeavor.I,MP


4) Instead of setting up a factory with secondhand machinery, the farmers could have gone
for establishing proper transport system. For which they could have purchased a new
vehicle or tractor. Setting up a factory does not seem feasible in villages where electricity
can be a major problem.
5) The case clearly shows the lethargic attitude of the farmers. Even after waiting for more
than 3 days, none of the farmer went back to the original factory.
6) From the given case, we can certainly make out that there was an increase in production
of sugarcane or more people turned towards village plant. Hence here we see a lack or
failure of planning and backups amongst the farmers.
7) A perfect case of Love thy Neighbor policy. The farmers decided to pay money for the
plant because other farmers were also paying. The other farmers acted as Opinion leaders
for them. Hence lack of personal attitude is missing in this case.
8) The farmers investing their money have probably entered the Debt Trap. And the Paltan
village is proving to be a moneylenders haven. As they had to start the plant they
invested their surplus money expecting good returns, but now the plant has failed, and
they also suffered the loss on the sale of their crops. Hence double loss. But they must
also keep in mind the money required to start the plant again.
9) The farmers can even make money by selling Molasses which is no where mentioned in
the case. Hence an opportunity of earnings is missed out which clearly a deficiency.
10) The Deficiency from the Government side is witnessed. The Administrator should have
been appointed at an earlier stage. But now it seems quite late.
11) The farmers need to be apply some Common Sense. Even after the machine failed, no
attempts were made to revive it, seems quite Surprising.

Thoughts to Live/Work By.:

It is not Where We Reach, but more importantly How we reach mattersGo on- 923 -.

A Management Paradise (un) Ltd .Endeavor.I,MP

b) If you are the Administrator, what steps would you take to restart the factory?
Solution:
a) I would divide the farmers in to 3 groups. Those producing in Large, Medium and Small
Quantities the Crop. All the Large and Medium Scale producing farmers would be
directed to main Factory, whereas all the smaller ones will be catered by the plant at
Paltan village.
b) Again the emphasis would be on Transport system which would be a Sine Qua non with
proper processing services.
c) If the large and medium scale farmers dont want to go to main factory, then I would tell
them to bring the crop relatively earlier than others, some where before the Crushing
Season. But for this proper Schedule should be made out with the assistance of the
farmers.
d) Even if Dezoning has be implemented by the Government, I would set a limit for the
village plant, and see to it that the capacity is not exceeded. The simple reason for this is
that the machinery is secondhand and has already broken down once. Hence this just a
Predictive management.
e) Being a Government appointed administrator, I would request to the Government to pay
a minimum price for the crop so that atleast the fixed costs of the farmers are covered. If
not this, then to avail better loan schemes.
f) Profitable treatment of waste products from the processing of Sugar Cane. For example,
molasses are used to produce Ethanol which acts as a energy resource.
g) If possible, this cooperative should be also converted in to Credit Cooperative, as in the
event of Losses, it will be helpful for the farmers to borrow money at discounted rates.
It is not Where We Reach, but more importantly How we reach mattersGo on- 924 -.

A Management Paradise (un) Ltd .Endeavor.I,MP


h) The other services should be taken care off. Proper Electrification, Warehousing,
Transport and most importantly Water Supply as Sugar cane needs ample of water
should be taken care off.
i) The farmers should be made to realize about their independent attitudes by Providing
them Education.
j) Just like a perfect market, a farmer should have full independence to sell his produce to
anybody. They should have a Choupal sort of thing which will be helpful in providing
Various information about Prices, new techniques and other information.

EPICenter
"Beware of thought pollution"

Get together with a group of your friends and they've all got one of these
horror stories to tell;
Their neighbour was burgled and/or raped. Their friend's mother-in-law was abducted. Their
brother was retrenched. Their company is on the point of closure. Their sister's boyfriend has
been killed in an accident. And so on, and so on.

As each tells a story, another finds a worse one. No one is actually listening to anyone because
they are all thinking about something even more horrific to recount. They are like traders at a
bazaar, each trying to outsell the other.

"You think that's bad, just wait till you've heard this." Soon they have convinced each other that
life is not worth living and the conversation usually ends with "And things are probably going
to get worse."
You have just fallen prey to thought pollution. And it will kill you. Maybe not physically but
certainly emotionally. If you feel that there is no hope for the future then you lose your sense of
purpose. When you have lost your sense of purpose your life loses it's meaning and then you are
merely existing, you are not living.

If there is faith in the future then there is power in the present. However if you have destroyed
your faith in a positive future then you will lose your power in the present. "I mean what's the
It is not Where We Reach, but more importantly How we reach mattersGo on- 925 -.

A Management Paradise (un) Ltd .Endeavor.I,MP


point of doing anything, the situation is terrible and is getting worse" You lose your sense of
enthusiasm and joy, life becomes something to be endured.

Thoughts to Live/Work By.:

Section II

3) Critically evaluate the role of 8ABARD in development of credit cooperatives in India?


Solution:
Finance in agriculture is as important as development of technologies. Technical inputs can be
purchased and used by farmer only if he has money (funds). But his own money is always
inadequate and he needs outside finance or credit.
Professional moneylenders were the only source of credit to agriculture till 1935. They use to
charge unduly high rates of interest and follow serious practices while giving loans and
recovering them. As a result, farmers were heavily burdened with debts and many of them
perpetuated debts. There were widespread discontents among farmers against these practices and
there were instances of riots also.
Although the co-operative banks started financing agriculture with their establishments in
1930s real impetus was received only after Independence when suitable legislation were passed
and policies were formulated. There after, bank credit to agriculture made phenomenal progress
by opening branches in rural areas and attracting deposits.
Since independence India has made substantial progress in respect of agricultural finance.
Reports of all India rural credit survey committee and all India rural credit review that farmers
were fully dependent of various sources of credit.

Agricultural credit in early years was

characterised by the growth of cooperative credit and there was decline in the share of
It is not Where We Reach, but more importantly How we reach mattersGo on- 926 -.

A Management Paradise (un) Ltd .Endeavor.I,MP


moneylenders. Moneylenders were the main sources of credit during 1951-52. However
occasional moneylenders like landlords and traders have come up as a important suppliers of
rural credit.
Moneylender was the most convenient and easiest source of credit. Moneylender did not make
difference between production and consumption credit. His credit was available at the time
needed by, the farmer. His administration was simple and quite flexible. He could asses credit
worthiness of not only borrower but his entire family. The most important advantage to money
lender was the absence of any worthwhile formal source of credit and illiteracy and ignorance of
rural community as whole. The social thinkers in India were confined to freedom movement and
other religious evils and had little time for assessing the damage done by moneylenders who
were essentially Indians during two centuries proceeding independence.
Charging exorbitant interest rates, cheating the illiterate borrowers by inserting larger amounts
than borrowed, non-issue of receipts for repayments were the main reasons of malpractice for
moneylenders decline.

All these activities aimed at grabbing the farmers land, the only

productive asset of that poor man. The development of agricultural sector called forth medium
and long-term credit large scale.

This need was associated with green revolution.

The

moneylenders depending upon their own fund could not entertain term lending.
The factors accounting for decline of moneylenders apart from malpractice could be their
inability to provide term loans. There are certain important positive factors for this. Most
important positive factor on the part of authorities was the realisation of needs for institutional
credit in agriculture. This let to official patronage for co-operative credit societies by way of
extending and liberalising refinance facilities. Further the short lived social control of
commercial banks and their subsequent nationalisation, the establishment of low cost regional
rural banks and restructuring of apex, have helped in a major way to institutionalise agricultural
credit.
Growth of literacy education and communication network has also helped to reduce the
importance of indigenous moneylenders in a big way. It should be kept in mind that the
informal sources havent totally disappeared, but have only declined in relative terms.
(Give the Brief outline of NABARD, its functions and its Objectives, refer paper 2002).

It is not Where We Reach, but more importantly How we reach mattersGo on- 927 -.

A Management Paradise (un) Ltd .Endeavor.I,MP


Critical Evaluation of the Role of DABARD
1.

Limited geographic coverage: The spread of credit societies has been limited only upto
certain states like Himachal Pradesh, Andhra Pradesh, Karnataka, Tamil Nadu, Punjab and
Maharashtra. In other states like U.P, Bihar, Madhya Pradesh, West Bengal, Gujarat have
not made any head way.

2.

Small and marginal farmers: Majority of farmers in India are small and marginal. They
need finance, for their needs but they are unable to obtain it and even societies find it
difficult to lend them. These farmers invariably cultivate for self consumption leaving very
little marketable surplus. In absence of such surplus they cannot repay loans. Hence the
societies go around the large and medium farmers.

3.

Inconvenience in borrowing: Large farmers find it convenient to borrow from commercial


banks and other institutions for such institutions do no face any shortage of' funds and have
certain targets to be fulfilled. Whereas cooperative societies are forced to depend on central
and state co-operative banks and ultimately on government apex body like RBI and
NABARD for financial requirements.

4.

Huge overdues: Banking activity is a two way process. Societies will be able to lend only
when loan and advances are regularly repaid. But actually in India credit societies are
overburdened with huge over dues. This stops continuous activities. The liability to repay
loans may be due to poor crops or non and unproductive use of credit.

5.

Linked with ownership landholding: The co-operative credit societies appear to be linked
with ownership holding of land. That stops tenant cultivators from the purview of cooperative credit. Similarly small and marginal farmers also would not get adequate credit.
Thus the basic purpose of self credit is defeated.
6.

Rapid

expansion

and

diversification:

Due

to

rapid

expansion

and

diversification the qualities of schemes have deteriorated. One of the problems


of deterioration is quality under the anti-poverty programme.
7.

Profitability affected: Rise in establishment expenses, inadequate business potential,


and increase in non performing, advances affect the profitability of banks, The yield on
advances has been decreasing, average cost of deposits and borrowings is increasing,
reducing the margins available to banks.

It is not Where We Reach, but more importantly How we reach mattersGo on- 928 -.

A Management Paradise (un) Ltd .Endeavor.I,MP


8.
Regional Imbalances: The bank has disbursed regional imbalances in the credit. For
e.g. in 1994-95 the disbursement of total credit in southern regions stood 51.6%, while
in northern, eastern and western regions was just 11.3%, 6.4 and 0.5 % respectively
during the same period.
9.

Lack of co-ordination: There is lack of co-ordination not only between the commercial
banks but also between banks and government departments. There is lead bank in
every district but in many cases number of branches belonging to commercial banks.

10.

Problem of recovery: The recovery position of lending bank creates a problem. This
alarming situation calls for a corrective action.

Lastly certain degree of politicalisation of co-operative movement, lack of training and


professional attitude of staff; growth of commercial and regional banks in rural areas
are also responsible for the retarded growth of co-operative credit in India.

4) Discuss the commonality of approach in respect of cooperatives and corporate


sector?
Solution:
In a professional approach, an organisation is a structure which is managed by a group
of individuals who work together towards a common goal. A cooperative organisation
is as good as a corporate in due courses of operations. The following are the common
aspects between Corporations and Cooperatives.
a) Both the Organisations have proper Vision, Mission and Long term Objectives.
b) Delegation of Work and Duties.
c) Interlinking the Activities for achieving the Objectives.
d) To some extent, Cooperatives also have Profit motives for its members.
e) Corporates, if they function properly, then they are also fulfilling the
philanthropic motives as done by the cooperatives.
f) Both are Associations of People.
g) Both have Social Responsibilities.
h) Proper Accounting System.
i) Both are Enterprises.
These are some of the common aspects between corporate and cooperatives.
It is not Where We Reach, but more importantly How we reach mattersGo on- 929 -.

A Management Paradise (un) Ltd .Endeavor.I,MP


(For differentiation aspects, see paper 2001)

5) Do you think that agro-based industrial cooperative organisations can be the


growth centres for the rural development in India?
Solution: Refer Previous Papers.

6) How far the cooperative marketing institutions are successful in eliminating the
exploitation of farmers in agricultural marketing in India?
Solution:
See Paper 2004
7) Write Short notes on:
a) Khadi and Village Industries Commission:
KVIC was setup in 1957. It is a statutory body created under the act of parliament in 1956. It is
charged with the responsibility of planning, promotion and organizing programmes for the
development of khadi and other village industries in the rural areas in coordination with other
agencies for rural development.
The main Objective of setting up this commission was: 1.

To reduce the unemployment problem in India.

2.

To boost the Rural Economic Development through promotion of small village industries.

3.

To create awareness about the hidden struggle through promotion of Gandhian Values.

Activities
1.

To prepare programs on promotion of Khadi and Village Industries and implement those
on a National scale.

2.

To help in training and development of village artisans and managers involved in this
industry.

It is not Where We Reach, but more importantly How we reach mattersGo on- 930 -.

A Management Paradise (un) Ltd .Endeavor.I,MP


3.
To make arrangement for raw-material storage and to facilitate in the distribution of
finished goods.
4.
5.

To study the problems of village economies and to suggest recommendations.


The commission provides assistance to industries like Tanning, Pottery, Paddy,
Processing, Khadi Sari, Sugar and Jaggery.

2) Urban Cooperative Banks:

The urban co-operative banks are co-operative banks, which are situated in the urban areas.
They play an important role for the benefit of small industrialist, trader, middle class
population etc.
The Role
1. They organize and bring together middle and working class population in urban and semiurban areas and inculcate in them the habit of thrift and self-help and acquaint them with the
elements of banking principles.
2.

Mobilization of savings and fulfilling the needs of finance for rural, industrial and other
development functions.

3.

Providing credit on reasonable terms to protest them from exploitation by the hands of
moneylenders and unscrupulous agency.

4.

To make available essential banking facilities to remote areas and to provide experienced
and effective leadership to the co-operative movement.

Dual Statutory Control


It is not Where We Reach, but more importantly How we reach mattersGo on- 931 -.

A Management Paradise (un) Ltd .Endeavor.I,MP


The urban co-operative banks are subject to a dual statutory control of:
The Registrar of the co-operative society (state government)
The RBI
Co-operative Banks are governed by the respective state co-operative act and also the
banking regulation act 1949. Matters such as registration of banks, approval of the
amendments of the byelaws, amalgamation and division of banks, explosion of members,
appointment of an administrator, audit, and liquidation etc., fall within the statutory
jurisdiction of the registrar under the provision of the co-operative law.
Matters like issue of banking license, banking transaction and business, supervision, policy
matters fall within the statutory authority of the RBI under the provision of the Banking
regulation act 1949.

c) Sugar Cooperatives in Maharashtra:


Indian Sugar Co-operative can be broadly classified into 2 sub sectors:
1. The organized sector e.g. Sugar Factories.
2. The unorganized sector e.g. Manufacture of Gur and Khandsari.
Out of the 453 sugar mills in the country, 252 are in the co-operative sector, 134 are in the
private sector and 67 are in the public sector. The government of India has appointed for tariff
board to examine the position of the sugar industry, due to this board the Indian Legislation
passed the sugar industry protection act in 1932.
Sugar is controlled commodity in India. It is covered under the preview of the essential
commodities act 1955. The government control sugar capacity editions through individual
licensing, determines the price of the major input which is sugarcane, decides the quantity that
can be sold in open market, fixes the prices of sugar etc. Government control is over all aspect of
production and sale of sugar extends to the level of wholesalers need to obtain a license issued
by the government before they can begin to operate.
Problems

It is not Where We Reach, but more importantly How we reach mattersGo on- 932 -.

A Management Paradise (un) Ltd .Endeavor.I,MP


1. De-zoning: The co-operative sugar industry was so far protected from competition from
other sugar factories. Due to the policy of zoning which were compulsory for a farmer to sell
and supply sugarcane to the prescribed factory within the zone of its field. This policy was
dismantled in 1996-97 due to which the factories, which could not give better price to the
farmers had to close down.
2. Shortage of Sugarcane: Many sugar factories are located in the main shadow region due to
which there is a lack in sugarcane produced which leads to un-utilization of processing
capacity.

3. High cost of water and electricity: The high cost of water and electricity increases the cost of
production.
4. Huge financial loses: Due to improper irrigation facility, scanty or no rainfall result in huge
financial loses. Also there are high loses due to improper storage facilities.
5. Corruption: Corruption is one of the major problems, which is faced by the co-operatives.
The main concept of Self help through Mutual help is lost in corruption.
6. Political influence and interference: There is too much of political influence and interference
in the co-operative society.
7. Problems of Trade Union: The trade unions come up with strikes and lockouts, which result
in poor functioning of the co-operatives.

d) Lizzat: A Success story in Cooperatives.


It all began on 15th March, 1959, a warm cloudless summer day. A majority of women
inhabitants of an old residential building in Girgaum, a thickly populated area of South
Mumbai, were busy attending their usual domestic chores.
A few of them, Seven to be exact, gathered on the terrace of the building started a small
inconspicuous function. The function ended shortly; the result: production of 4 packets
of papads and a firm resolve to continue production. This pioneer batch of seven ladies
set the ball rolling.
It is not Where We Reach, but more importantly How we reach mattersGo on- 933 -.

A Management Paradise (un) Ltd .Endeavor.I,MP


Shri Mahila Griha Udyog Lizzat Papad is a womens organization manufacturing
various products from papad, khakhra, appalam, masala, vadi, gehu atta, bakery
products, chapatti, Sasa Detergent, and Sasa liquid Detergent.
The organization which started with a paltry sum of Rs. 80 has achieved sales of over
Rs. 300 Crores. Its exports exceed Rs. 10 Crore.
The membership has expanded from seven sisters to over 40, 000 sisters throughout
India.
The success of the organization stems from the efforts of its member sisters who have
withstood several hardships with unshakable belief in The Strength of a Woman.

EPICenter
"8ever turn your back on your problems. Until you face them you can't solve them
These things that we call problems are merely situations that:
we have never experienced before, it is something new therefore we do not have an effective
strategy to handle it; or
we have previously experienced but we have not yet developed an effective strategy to handle;
or
we do not feel confident or competent to handle.

The problem with most people is that they do not want problems, and when problems arise, which
is unavoidable, they get so upset because they have a "problem" that they cannot handle it
effectively.
Change your attitude towards problems. Realize that without problems there can be no growth.
The fact that you are today who you are and where you are is thanks to your ability to solve
problems.

The quality of your life is not determined by how many problems you have or has had but rather
by how you handle and have handled your problems. But this you must realize
It is not Where We Reach, but more importantly How we reach mattersGo on- 934 -.

A Management Paradise (un) Ltd .Endeavor.I,MP


"8ever turn your back on your problems. Until you face them you can't solve them"

Thoughts to Live/Work By.:

Management Of Cooperatives November, 2004


Section I

Concepts
a) What do you mean by Social Responsibility of Cooperatives?

Refer Paper 2004.

b) What is Dual Statutory Control of Cooperatives?


Solution:
Let's take an example to explain this point. The Registrar, who is a representative of the state
government and also the Reserve Bank of India, controls co-operative banks. Similarly all other
co-operatives have dual control. There is no one fixed line of hierarchy. This is a problem
because if one employee has two bosses, he has twice the problems.

c) What do we mean by Limited Interest on Capital?


Solution:
It is not Where We Reach, but more importantly How we reach mattersGo on- 935 -.

A Management Paradise (un) Ltd .Endeavor.I,MP


Limited interest on capital (Do profit motive):
It is a unique principle where capital is treated as subsidiary where as the prime and ultimate
objective is service to members, the economic outcome arising out of the operations of a society.
The member will be benefited in a manner where it would avoid one member gaining at the
expense of others. Capital is a factor of production and is also entitled a fair rate of return. In cooperatives capital cannot dominate, rather it serves in return for limited interests as service being
the supreme.
The motive of earning a profit is absent in co-operatives otherwise it will mark the basic motive
of co-operation i.e. service.

d) What is a Flat Owners Cooperative Housing Society?


Solution:

Food, clothing and shelter are the prime necessity of human beings. Housing is a significant
concern of every individual. There is an acute problem or shortage of housing in India due to
the following reasons:
1.

Tremendous growth in population.

2.

Migration of people from rural areas to urban areas.

3.

Discontinuance of joint family system leading to requirement of separate house to each


family unit.

4.

Highly speculative trend in cost of land.

5.

Hike in prices of building material resulting in increase in cost of house everyday.


Co-operative housing is a solution to the above given problems because it is a wellorganized significant movement of the common men.
Features

1.

They are legally established associations of persons or members.

2.

The members democratically control them.

3.

Co-operatives societies provide for minimum required facilities like drinking water, tank
water, waste disposal, sanitation, electricity, common car parking etc.

It is not Where We Reach, but more importantly How we reach mattersGo on- 936 -.

A Management Paradise (un) Ltd .Endeavor.I,MP


Three Tier System Of Co-operative Housing (State Level)

1.

Primary Co-operative Housing Society


They are of 3 types as per the classification given in the Rule 10 of Maharashtra State Cooperative Society Act 1960. The sub-classification is on the basis of their objectives and
activities. The 3 main types are as follows:

a.

Tenant Ownership Housing Society


Housing society where the land is held either on leasehold or free hold basis by the societies
the houses are owned by the members. Due to the ownership of land with the societies the
members become tenants, but as they own the houses. This form is called as TenantOwnership.

b.

Tenant Co-Partnership Housing Society


Housing society where land and buildings are held by the society and only the flats or shops
are owned by the members. There is a partnership between the society and the members in
the ownership of flat or shop or buildings.

c.

Other Housing Society


Housing co-operative society not coming under the 2 above mentioned categories are
included in other housing society.
E.g. House Mortgage Society, House Construction Society etc.

It is not Where We Reach, but more importantly How we reach mattersGo on- 937 -.

A Management Paradise (un) Ltd .Endeavor.I,MP


e) Cooperation among Cooperatives What do we mean?
Solution:
Co-operation among Co-operatives:
Its a new concept that concentrates on developing and strengthening of different co-operative
sectors by making them inter dependent. Co-operatives serve their members most effectively to
strengthen their co-operative movement by working together through Local, Regional, National
and International Structures. They are also known as Federation of Cooperatives.

Case Study
In Janta Sahakari Bhandar, the purchasing committee consists of the Managing Director and the
Accounts Officer. The committee normally obtains quotations from a particular three firms,
which belong to one person only. As a result the quality of goods supplied is inferior, resulting
in large Quantity of unsold goods. Apart from these there is also no proper accounting
procedure. Stock taking is not done periodically. The accounts of the suppliers are settled
directly by the accounts officer. Staff members of Janta Sahakari Bhandar are also demonstrated
and do not attend to customer needs properly. This has resulted in huge losses.

a) Identify the Main Issues?


b) Suggest Remedial measures thereof.

For both Questions Refer to paper 2002 case study and Relate the answers to the
above questions.

Thoughts to Live/Work By.:


It is not Where We Reach, but more importantly How we reach mattersGo on- 938 -.

A Management Paradise (un) Ltd .Endeavor.I,MP

Section II

3) What are the benefits of cooperatives from different view points?


Solution:

Cooperation is the unique innovation of the nineteenth century in purpose, form, management
and objectives. The purpose of cooperation is socio-economic in nature and it aims to save the
Have nots from the exploitation of Haves.

The social purpose of cooperatives has economic objectives of promotion of the economic
interests of the members organizing them, they are also more social than other types of business
organization in focusing their activities on men behind them and serving as a remedy against
exploitation.

Cooperatives come with various benefits, the following are the beneficiaries of these benefits:

a) Towards Business, Occupation, Vocation:


1) The objective of any cooperative is to promote the economic interest of the
members, whose fulfillment is aimed by either conducting any business such as
that of the consumer cooperative store or cooperative marketing without
exploitation of credit societies.
It is not Where We Reach, but more importantly How we reach mattersGo on- 939 -.

A Management Paradise (un) Ltd .Endeavor.I,MP


2) Creating employment opportunities or promoting occupational skills of the
members.
3) Cooperative society can discharge its responsibilities towards business by
undertaking fair and ethical businesses and also promoting the business so that it
benefits the society.
4) An important function of any cooperative is to lend. And by lending at cheap
rates proves beneficial.
5) Ensuring supply of quality goods at prices lower than the market price is also a
benefit.
6) An important benefit of a cooperative is that it pays fair wages to the employees
for improving the quality of their work.
7) The basis of cooperative is to remedy exploitation of weak by strong in the trade.

b) Towards Customers: Customers of the cooperative are largely its users or members.
There are also customers who are not the member of the cooperatives like consumers of
milk, wholesale dealers in sugar factory, and buyers of oil producers co-op. society. Etc.
1) Cooperatives are to ensure a fair deal to the customers.
2) The Fair pricing of the product is also a benefit.
3) The supply of pure and unadulterated goods should be ensured.
4) An important benefit to customers is that are supplied with complete information
about the product and their users.

c) Towards Members Users:


1) As cooperative society is meant for the poor and the weak, the face value of the
share is kept low and ceiling is fixed on the number of shares a member can hold.
2) The cooperative societies undertake all the measure to improve the occupational
skills of the members.
3) They also inculcate a sense of thrift in the members and provide opportunity to
them to keep their savings by accepting deposits.
4) A cooperative society also discharges its responsibilities towards members by
involving itself in their economic and social life by attending to the marriages,
helping them to meet unexpected accident expenses arising out of death in the
family.
It is not Where We Reach, but more importantly How we reach mattersGo on- 940 -.

A Management Paradise (un) Ltd .Endeavor.I,MP


d) Towards Employees:
In cooperatives there are two types of employees. The first is the office staff that consists of
managers, accountants, clerk and peon. The other group consists of skilled and unskilled
workers and technicians.

1) Cooperatives recognize the role of its group of employees in making the


cooperative succeed in fulfilling the objectives.
2) Surplus is distributed amongst members in the form of bonus.
e) Towards the Competitors:
1) They operate along with competition in private as well as public sectors.
2) They endeavor healthy competition prompting the competitors also to follow
such market forces.
3) They do not follow unethical monopolistic restrictive and unfair trade practices.
f) Towards the Government:
In India, Government has been actively involving itself in the growth of the cooperative
movement. Government help is needed at every stage. In India this assistance is from
Womb to Tomb.
1) Through Cooperatives, Government aid is fully utilized.
2) They assist the government in distributing and providing the essential goods like
food grains, sugar, and kerosene.
3) They assist the government in all its economic and social welfare measures.
4) By paying taxes regularly is also a benefit.

g) Towards the public at large:


1) Cooperatives are small in nature and they aim at educating the members in simple
living and thrift, the impact of cooperatives is on ethical and moral life of the
people.
2) They are a tool for solving the problems of unemployment and poverty.
3) They check the concentration of economic power in the hands of few and
resultant division of the society.
4) They undertake charitable activities within their limited capacity.
5) Economic development :With a very vast section of the community taking out
life at subsistence level, co-operation has helped in expediting the process of
It is not Where We Reach, but more importantly How we reach mattersGo on- 941 -.

A Management Paradise (un) Ltd .Endeavor.I,MP


economic development through inducement of human resources available in
nonofficial way.
Co-operatives organizations are based on equity, equality and mutual self-help.
Though they are economic in character, but they have moral, ethical, social,
cultural and political motivations as well.
6). Equality - democracy
Equal rights and opportunities for people to participate in a democratic way, which
improves the use of the societys resources and foster mutuality, understanding
and solidarity.

7)

Liberty - voluntaries

Activities in the society are voluntary, and this is the best way to promote peoples
participation, commitment and responsibility
8) Social and economic emancipation, mutual self-help
People have the will and the capability to improve living conditions according to the
peaceful step-by-step way, by means of consciousness-raising activities and of
joint action for co-operative power.
9) Meeting economic needs
The economic organizations of the community are developed with the aim of serving
the needs of the people.
10) Social responsibility
Economic and other organizations are characterized by responsibility for the
community as a whole.

It is not Where We Reach, but more importantly How we reach mattersGo on- 942 -.

A Management Paradise (un) Ltd .Endeavor.I,MP


11) Price Reduction
Relief to some extent could be provided to the customers by allowing price
reduction.
12) Provide employment
Provides employment to a section of the society, which is unemployed or living
below the poverty line.
13) Provide relief
Provides relief to some extent to certain weaker sections of the society such as
orphans etc.

4) What are the objectives and problems of Marketing Cooperatives in India?


Solution:
Co-operative Marketing Society
When producers of agricultural commodities or any other product form a society with an
objective of carrying out marketing of their produce, such society is called as co-operative
marketing society. The need for co-operative marketing arose due to many defects observed and
experienced in the private and open marketing system. Those are
1.

Several malpractices prevail in the marketing of agricultural produce. For example,


arbitrary deductions from the produce, manipulation of weights and measures and cheating
the farmers, collusion between the broker and the buyer while fixing the prices, delay in
payment of amounts due to farmers, etc. The result is the farmers are indebted to trader moneylender. In such circumstances co-operative marketing society can largely help the
farmers reduce the malpractices and offer honest and correct services.

2.

There exists a chain of intermediaries between the producer and the final consumer. They
include village merchant, itinerant trader, wholesaler, commission agent, pre-harvest

It is not Where We Reach, but more importantly How we reach mattersGo on- 943 -.

A Management Paradise (un) Ltd .Endeavor.I,MP


contractor and retailer. They take their own margins for the services, they render. But these
margins are generally exorbitant, making the commodities costly for the consumers and
reducing the producer's share in the consumer's price. A co-operative marketing society can
eliminate some or all of the intermediaries and can reach to the consumers and establish
direct trade relations with them. This will make commodities cheaper to the consumers and
also ensure good quality of produce to them because much of the handling is avoided.
3.

There are some services such as transport, storage, financing, grading, packing,
loading/unloading which are carried out by some private functionaries who charge high rates
for these services. A co-operative marketing society performs these services efficiently and
at cheaper rates.

4.

A co-operative marketing society provides market finance to farmers and ensures better
returns to their produce. Besides marketing society can act as an agent of credit co-operative
society and help to recover loans advanced by credit societies. At present, most of the
financial needs of the farmers are fulfilled by trader - moneylenders at very high rates of
interest and with the condition that they will sell their produce through them. This can be
avoided, if there is co-operative marketing society.

Organization:
Under the system of co-operative marketing whole responsibility of marketing is taken up by the
farmers themselves, organized on co-operative basis. The area of operation of marketing society
is usually fixed with reference to local conditions - area based or commodity based. The
commodity-based societies related to grapes, oranges, banana, pomegranate, etc. have wider
jurisdiction covering the major areas growing each crop. There are societies at the producer's
level and they federate at state or national level to deal with bigger markets including foreign
markets for export of their produce.
Membership :
Membership of a co-operative marketing society is open to individual farmer who produces the
crop for which the society is formed. Other co-operative societies in the area can also become
institutional members.
Resources:
It is not Where We Reach, but more importantly How we reach mattersGo on- 944 -.

A Management Paradise (un) Ltd .Endeavor.I,MP


The sources of fund of the society are as under:
1.

Share capital

2.

Deposits.

3.

Loans from higher financial institutions including NABARD.

4.

Grants or subsides from the Govt. for godowns, etc.

5.

Reserve funds.

The marketing societies require short-term, medium-term and long-term capital


1.

Short-term capital is needed for financial advances to members for production, packing,
transport, etc. to meet contingent expenses.

2.

Medium- term capital is required for purchasing motor trucks, etc

3.

Long-term loan is required for installation of machinery, construction of building for


godown, storage, etc.

Functions:
1.

To arrange for the sale of members produce to the best possible advantage.

2.

To undertake activities in connection with grading, pooling and procurement of produce of


the members.

3.

To provide storage facilitates to their members by renting or owning the godowns and
thereby facilitate to grant advances against pledge of produce.

4.

To protect members from all types of malpractices eliminates the middleman in the chain
of marketing.

5.

Co-operative marketing society ensures grading, etc. and supply of good quality material
to consumers.

6.

It teaches business methods to farmers and serves them as agency for supply market
information.

It is not Where We Reach, but more importantly How we reach mattersGo on- 945 -.

A Management Paradise (un) Ltd .Endeavor.I,MP


7.
The society is able to stabilize prices over a long period by adjusting the supply with the
demand.
8.

Marketing societies are also encouraged to undertake export trade so that they can give
better prices to their members.

The Co-operative Development Corporation


The National Co-operative Development Corporation has been promoting and financing a wide
range of economic activities in rural areas through co-operatives. The Co-operative is a unique
institution in the country catering to the development of the rural economy and agriculture sector
through co-operative. There is no other institution in the country which is exclusively for
meeting the requirement of co-operatives.
NCDC has been playing special attention to weaker sections co-operatives in various part of the
country. The promotional and development role of NCDC had lead to continuous diversification
and expansion of co-operative programs under its preview.
Few Successes:
Inspite of the difficulties encountered in the marketing of perishables like fruits, vegetables,
milk, etc. there are few examples of good success.
1. Maha-grape - co-operative federation marketing grapes in Maharashtra.
2.

Co-operatives marketing pomegranate.

3.

Co-operatives marketing banana in Jalgaon district.

4.

Vegetables co-operatives in Thane District.

5.

Milk co-operatives in Maharashtra and Gujarat.

6.

Co-operative cotton marketing societies.

Weak Co-op. Marketing:

It is not Where We Reach, but more importantly How we reach mattersGo on- 946 -.

A Management Paradise (un) Ltd .Endeavor.I,MP


Although, many advantages are envisaged in the co-operative marketing the structure has
remained relatively weak as compared to credit co-operatives. There are only about 1000
marketing societies as against 20,000 credit societies in Maharashtra. Their marketing is
more difficult involving many technical and commercial aspects. Marketing of perishable is still
more different. Arranging quick transport, arranging storage to avoid losses, to keep watch on
demand - supply position to ensure good prices to members are all matters need for good
marketing. For want of these managerial aspects, desired numbers of co-operative marketing
societies have not come up and those which were started could not succeed. Several marketing
surveys/studies at farmer's levels have revealed that among several marketing channels, cooperative channel has offered greater share of consumer's prices to the producers. Whichever,
marketing is unorganized, farmer - producers have expressed that marketing co-operative
societies should be formed. This was particularly reported in the cases of marketing of
perishables.

EPICenter
"You may not be able to control what happens but you can, and should, always control your
behaviour."

The quality of your life is not determined by what happens to you. The quality of your life is
always determined by your response to the event. Your response to any event is always in your
control.
You choose your behaviour.

Each morning and throughout the day make a conscious decision about how you choose to
behave.

Realise that "You may not be able to control what happens but you can, and should, always
It is not Where We Reach, but more importantly How we reach mattersGo on- 947 -.

A Management Paradise (un) Ltd .Endeavor.I,MP


control your behaviour."

Thoughts to Live/Work By.:

6) What are the important characteristics of a cooperative leader? Analyze their roles in
different stages of cooperative development.
Solution:

A cooperative organization basically has to run on the lines of Dual Control. That is, it has to
operate in the environment wherein both management principles and principles of cooperation.
The cost of Globalization is such that we have to compensate it through cooperatives. When
you starve with a lion, it is the lion that starves at last: Principle of Democracy. Hence, a place
where multinationals are fighting with their vast surpluses, cooperatives are finding it difficult to
even make their mark.

Be willing to make decisions. Thats the most important quality in a good leader. Dont fall
victim to what I call the ready-aim-aim-aim-aim syndrome. You must be willing to fire.
T. Boone Pickens
A cooperative organization is a democratic unit. Policy making in a cooperative is a function of
leaders chosen or elected with the consent of atleast a majority of members of the cooperative.

Leadership as some define: It is a relationship amongst Leader and Followers. But basically it
is a Interpersonal influence exercised in a situation and diverted through the communication
process towards attainment of the specified goals.

Role of Leadership:
It is not Where We Reach, but more importantly How we reach mattersGo on- 948 -.

A Management Paradise (un) Ltd .Endeavor.I,MP


An individual with the necessary intelligence, common sense and dedication may emerge as a
Leader of a cooperative body.
Basically a leaders role is divided in to various compartments, but can be classified in to 3:

Hence as given above, the roles are basically classified in to 3 roles: a) Encourager
b) Suggester
c) Implementer
a) Encourager:
Some examples of persistence:
The Coca-Cola Company sold only four hundred Cokes in its first year of business.
Dr. Seusss first book was rejected by twenty-three publishers.
Henry Ford went bankrupt twice before hitting it big in the automotive industry.

Just do it.
8ike Slogan

It is not Where We Reach, but more importantly How we reach mattersGo on- 949 -.

A Management Paradise (un) Ltd .Endeavor.I,MP


In the formative stage of a cooperative, a leader has to play the role of the encourager. He
has to motivate the people so that they can participate in the movement and also encourage
them to give their best of the ideas, resources and the skills required to manage the
cooperative.

b) Suggester:

General Dwight D. Eisenhower used a simple devise to illustrate the art of leadership. Laying
an ordinary piece of string on a table, hed illustrate how you could easily pull it in any
direction. Try and push it, though, he cautioned, and it wont go anywhere. Its just that way
when it comes to leading people.

A leader is a dealer in hope.


8apoleon Bonaparte

This behavior marks the perpetual existence of the cooperative. At this point, the leader should
himself suggest the Encouraging ideas for the betterment of the cooperative.

Implementer:
Corporation: an ingenious device for obtaining individual profit without individual
responsibility.
Ambrose Bierce

When Robert Fosse was replaced as director-choreographer on the short-lived Broadway show
The Conquering Hero, he complained, I conceived this show. I outlined it. I cast it. I
choreographed it. I drew the first sketches for the design. I directed it. And the first thing that
goes wrong, they blame me!

It is not Where We Reach, but more importantly How we reach mattersGo on- 950 -.

A Management Paradise (un) Ltd .Endeavor.I,MP


The leader has to play the role of the Implementer, when the Governments role is minimized,
and implement all the policies for the cooperative.

Role of Leadership in Co-operatives


1.

Educating the people on co-operative principles.

2.

Influencing the masses to form co-operative organization.

3.

Organizing the group and explaining to them the objectives and goals of the venture.

4.

Motivating the masses.

5.

Giving direction to the movement and coordinating and controlling the activities.

6.

Explaining a vision to the people and showing uttermost commitment and faith towards it.

7.

Reforming the role of a councilor and a troubleshooter.

Examples of effective leaders


Dr. Vasant Rao Dada Patil Vasant Rao Dada Co-operative Sugar Factory, Sanghli
(largest sugar factory in Asia).
Dr. Balasaheb Vikha Patil Co-operative Sugar Factory at Pravaranagar, Ahmednagar
district.
Dr. Vaikunthbhai Mehta
Dr. Sharad Pawar Baramati

It is not Where We Reach, but more importantly How we reach mattersGo on- 951 -.

A Management Paradise (un) Ltd .Endeavor.I,MP


Influence of co-operative movement on the political life in India:
Examples
1.

Sharad Pawar Defence minister of India, CM of Maharashtra for 12 years, Deputy


Chairman of National disaster management committee of India.

2.

Vilas Rao Deshmukh CM of Maharashtra, controls co-operative lobby in Latur, controls


Co-operative Sugar Industry.

3.

Dr. Jayant Patil Finance Minister of Maharashtra, controls co-operative sugar factory in
Islampur.

4.

Dr. Patangrao Kadam Founder of Bharti Vidya Peeth, a dean university, Minister at State
Cabinet, controls education lobby.

5.

Dr. D.Y. Patil Engineering colleges, Dental colleges.

Qualities of a Leader:

1) Because of vested interests of the members, the leader selected by the cooperative is like a
puppet in the hands of the managing committee.
It is not Where We Reach, but more importantly How we reach mattersGo on- 952 -.

A Management Paradise (un) Ltd .Endeavor.I,MP


2) In countries like India, there are no proper training facilities to train people in leadership.
3) Due to dual control, the political influence is affecting the role of a leader in managing a
cooperative. The leader who succumbs to the pressure from politician will drastically affect
the smooth functioning of the cooperative.

7)
a) Primary Land Development Banks:
b) International Cooperative Alliance:
International Co-operative Alliance (ICA)
International Co-operative Alliance is an independent non-governmental association which
unites, represents and serves co-operatives worldwide. The ICA was founded in London in
1895. Its members are National and International Co-operative Organization in all sectors of
activity including agriculture, banking, energy, industry, fisheries, housing, tourism and
consumer co-operatives: ICA has more than 230 member organization from over 100 countries
representing more than 730 million individuals worldwide
In 1946, the ICA was one of the 1st Non-governmental organisations to be accorded UNs
Consultative Status. Today it is one of the 41 holding-holding category of consultative status
with the UNs economic and social council.
Aims of ICA
The major objective of ICA is to promote and strengthen autonomous co-operatives throughout
the world. ICA seeks to: 2. Promote and Protect Co-operative Values and Principles.
3. Facilitate the development of economic and other mutually beneficial relations between
its member organizations.
It is not Where We Reach, but more importantly How we reach mattersGo on- 953 -.

A Management Paradise (un) Ltd .Endeavor.I,MP


4. The economic and social progress of its members and their communities.
Specialized Organization under ICA
1. International Co-operative Agricultural Organization (ICAO)
2. International Co-operative Banking Association (ICBA)
3. Consumer Co-operative International (CCI)
4. International Co-operative Energy Organization (ICEO)

c) Recommendations of All India Rural Credit Survey Committee:


All India Rural Credit Survey Committee (1954)
To recommend the developmental majors for co-operative sector in Rural India, the
Government of India appointed a committee under the chairmanship of A.D. Gorewalla in 1951.
This committee submitted its recommendations in 1954.
Recommendations
1.

There should be mergers of primary co-operatives to make them financially sound

2.

The purpose of the loan should be taken into account while sanctioning the loan

3.

No money lender/Commission Agent should be provided credit. Care should be given to


give credit only to the weaker sections of the society

4.

Primary co-operative society should be provided with more finances by the National
Credit Institutions

5.

Members who dont pay arrears in time should be dismissed from the society on legal
grounds

6.

There should be a link between Agricultural Credit Societies and Marketing Societies

d) Role of Board of Directors:


The Board of Directors is elected representatives of the members who are the real owners of
the organization. The main function of Board of Directors is policy formulation and deciding
the overall objective and goals of the organization. The implementation or execution of the
It is not Where We Reach, but more importantly How we reach mattersGo on- 954 -.

A Management Paradise (un) Ltd .Endeavor.I,MP


policies is carried out by the managerial hierarchy lead by the general manager. The duties of
the Board of Directors are to see that the policies formulated are implemented properly. But
many times it happens so that the Board Of Directors interference in day to day functioning
of the executive due to which conflict situation is created as the executive feels that they lack
in independent decision making capacity and have been treated as mere clerks.
The Chairman of the Board has to perform a crucial function of co-ordination because his
main task is to provide necessary guidance and also to give the executive autonomy in the
implementation of the decisions by the board.

Responsibility of the Board of Directors/Managing Committee


1.

It should be cost conscious.

2.

All members should be given equal opportunity to discuss and voice their views.

3.

Decision taken should be collective and no members must be allowed to jeopardize the
decisions.

4.

Decisions taking should be with full consent and proper understanding of the problem.

5.

The Board Of Directors should be responsible towards:

a.

Responsibilities towards Employees

b.

Responsibilities towards Consumer

c.

Responsibilities towards Government

d.

Responsibilities towards Shareholder

e.

Responsibilities towards Environment

f.

Responsibilities towards Society

Functions
1.

Admission of new members and allotment of shares.

2.

Approval for transfer of shares.

3.

Preparation of guidelines for the conduct of the operation as per byelaws rules and acts.

4.

Approval of annual budget and its submission to general body.

It is not Where We Reach, but more importantly How we reach mattersGo on- 955 -.

A Management Paradise (un) Ltd .Endeavor.I,MP


5.
Sanction of the expenditure for the management in the allotted budget.
6.

Ensuring maintenance record and its submission to the registrar.

7.

Raising resources for managing the day-to-day affairs.

8.

Sanctions of investment of funds.

9.

Appointing staff and agencies.

10. Appointing sub committees or exclusive committee and delegating necessary powers to
them.
11. Electing the chairmen amongst themselves.

EPICenter
"With your thoughts you make your life."
You create the emotional reality of your life by the way that you interpret events and
circumstances. The one thing that is certain is that you seldom know what is real, you only know
what you think is real. The problem is that your response to any situation is always based on your
perception of reality.

To take control of your life you must learn to question your "reality" "Is it true?" "Do I know
enough to make this decision or to come to this conclusion?" "Why do I feel this way?" "What
effect did the situation have on me?" "How important is it really?" Learn to question your
thoughts because
"With your thoughts you make your life."

Thoughts to Live/Work By.:

It is not Where We Reach, but more importantly How we reach mattersGo on- 956 -.

A Management Paradise (un) Ltd .Endeavor.I,MP

Management Of Cooperatives November, 2005


Section I
Concepts
1)
a) What do we mean by Political and Religious 8eutrality?
Solution:
One of the Most important Principles preached by International Cooperative Alliance is
Religious and Political Neutrality. It simply means, that when a Person is considered for a
Membership in a Cooperative, that decision cannot be affected by the religion that person or
the other members of the Cooperative belong to.

2) What do we mean by Builders Cooperative Society?


Solution:
Societies wherein the land is owned by the Society while the houses are owned by the
members is called as Builders Cooperative Society. As the land is owned by the society, the
members have the status of tenants, but the land is owned by the builder. Builders
Cooperative Society is also called as Tenant Co-partnership Housing Society. As the
members are owners of their houses, hence Tenant Co-partnership.
It is not Where We Reach, but more importantly How we reach mattersGo on- 957 -.

A Management Paradise (un) Ltd .Endeavor.I,MP

3) Role of Leadership on Cooperatives.


Solution:
Refer paper 2004

4) Founder Rochdale and Cooperatives:


Solution:
England the birthplace of many new things was also responsible for presenting the co-operative
movement to the world as an alternative philosophy to trade unionism, for fighting capitalistic
exploitation.
In 1844, a group of 28 weavers met in the Chartist Hall and decided to establish a co-operative
store. The members of this store would purchase the requirements collectively to economize on
their expenditure, for they knew that it would be difficult for them to get a raise in their wages.
They were greatly influenced by the writings of Robert Owen, Dr. William King and William
Thompson.
Thus Robert Ovens dream of an alternative to capitalism was brought into reality by the
villages of Rochdale, 28 weavers came together and established Rochdale equitable pioneers
society 1844 contributing 1 pound each. Thus gathering a capital of 28 pounds and registered it
under the Friendly Society Act. It was a consumer co-operative. They hired a place and started a
shop and sold items of daily necessities such as candles, tea, butter, soap etc in bulk and sold
them to consumer at reasonable prices. It opened with a legal amount under unfavorable
circumstances but slowly made good progress in the years to come. The membership raised to
74 in 1 year and by 1888 it had 1123 members with a capital of 3.4lakhs pound and declared
2.5lakh ponds as a profit.
Objective of Rochdale Model
1. To take up production and distribution of the commodities required by the members.
It is not Where We Reach, but more importantly How we reach mattersGo on- 958 -.

A Management Paradise (un) Ltd .Endeavor.I,MP


2. To make available houses to its members to improve their living conditions and to better
their status.
3. To meet the clothing requirements of its members.
4. To arrange for education to its members.
5. To manufacture articles, so as to generate employment to its members.

5) What do we mean by Open Membership?


Solution:
Open membership implies that there shall be no restrictions on the admission of members, open
to all persons who are able to use their services and willing to accept the responsibilities of
members, without gender, social, racial, political or religious discrimination. It seeks a
homogenous interest of the members who come together for a mutual benefit.
This principle is undergoing changes with time. A little amount of compulsion and restriction is
imposed for the smooth working and speed of the movement. The gates of membership cannot
be kept open for persons with vested interests e.g. Moneylender, as a member of credit society.
However, a person can be compelled to join or withdraw, from such a co-operative in
larger interest.
EPICenter
"Courage is not action in the absence of fear. Courage is action in spite of your fear"
Courage is an attitude that you can, and should cultivate for it is something that you need every
day of your life. Often courage is thought of as something displayed on the battlefield or in times
of crises, some daring act in the face of danger. Yet every day we each need courage just to face
the daily routine of living.

Have the courage to face the world each day, realise that your contribution is significant and that
you do make a difference.
It is not Where We Reach, but more importantly How we reach mattersGo on- 959 -.

A Management Paradise (un) Ltd .Endeavor.I,MP

Thoughts to Live/Work By.:

Case Study
Sum Plaza Cooperative society is located at Irani Road, Dahanu and comprises of 54 flats. It
is a registered cooperative housing society. There are many irregularities in the society
which often go unnoticed. Some honest and conscious members have asked for an
emergency general meeting of the society to discuss the following matter.
1. Mr. Anil kumar a member of a society who has been allotted a flat on the ground floor
has dug a basement under his flat without seeking permission either from the society or
the corporation. His contention is that he is not using any extra floor space index and
therefore does not have to seek permission.
2. The Secretary of the society has defaulted on paying societies dues of Rs. 20, 000.
However, since he is an office bearer, no notice has been served.

Questions:
a) Give your views and correct legal position in the case of digging a basement without
the permission.
Solution:
a. Given the case study, Mr. Anil Kumar is at fault as he has got the land dug without
asking or seeking the permission from the society or the corporation:
The following are the Arguments against Mr. Anil Kumar:
It is not Where We Reach, but more importantly How we reach mattersGo on- 960 -.

A Management Paradise (un) Ltd .Endeavor.I,MP


b. Mr. Anil Kumar resides in a cooperative society, wherein every vital decision relating to
the society is taken by all the Society members together.
c. Secondly, it is also a registered society. Hence if members of Cooperatives are not
involved in any major decision the registrar should have been informed of the Alteration
of Conditions.
d. Well to say, Mr. Anil Kumar is alone not responsible for the problem, it is also
aggravated by the other non-active honest members. Sensing a breach of trust from Anil
Kumar, they should have lodged a complaint, but alone honesty will never be able to
make any difference. Hence other members are equally responsible.
e. In any society, whether cooperative or not whenever if anybody wishes to shovel land,
they have to either inform and take permission from the members if a cooperative or
form Corporation if not a cooperative.
f. Just like Government charges Water Tax on use of Water, Entertainment Tax on
Entertainment, similarly it also charges Property Tax for holding a property. The simple
logic here is that all the land within the Geographical Boundaries of the Country belongs
to the Country and no single person or group of person can claim an absolute ownerhip
rights on it. Thus we all have Creative rights on Land thus upholding the Cause for
Destructive Rights.
g. An important reason for this is when one digs up Surface of Earth, they are harming the
resources beneath it. And anyways, whatever is present beneath the land is a part of
National Wealth which cannot be exploited for personal economic uses.
h. Similarly, by digging up the basement, Mr. Anil Kumar has acted against the interest of
the society. By digging up the ground floor, he may have disturbed the structure of a
beam or founding pieces of the society. And it may as crucial as foundation stone which
is root of any building/ Structure.
i. Hence Mr. Anil Kumar, has intentionally by his actions violated the trust of members of
society as well as Government and should be held liable for a Breach of Trust.

Thoughts to Live/Work By.:


It is not Where We Reach, but more importantly How we reach mattersGo on- 961 -.

A Management Paradise (un) Ltd .Endeavor.I,MP

2) Can the District Registrar take action and remove the Secretary for such an act of
default? Explain.
Solution:
Yes, the Deputy Registrar has the powers to remove the Secretary for his vested interest
Actions.
The following are the Points to be noted for the given case:
1. One of the prime Functions of Secretary is to ensure the working of Society is never
faulted. The Secretary us a Watch Dog with an Exception. That is if anything goes
wrong, he has powers to keep a check on any Wrong Doings.
2. Let us look at other points, there are also other office bearers like Treasurer ( one who
maintains all the Accounts of the Society), Chairman ( The CEO of Society), and
Managing Committee ( The Functional and Advisory Group) to look after the Smooth
Administration of Society. But in this case, it is to be noted that nobody from the Office
bearers has realized the Defect on Secretarys part of payment. The Treasurer being the
Accounts head of the Society should have known this much earlier, but which is not
happening in the given case.
3. Hence we can only say that, the Office bearers are not interested in doing the work of the
Society.
4. The other side of the case is that other member, the so called Honest Lot is only busy
complaining and asking for meeting whereas no proper action has been taken in this
case. After knowing that Office Bearers are not interested in the work they still asked for
the Meeting, but failing to realize that the meeting will be called by them only. Hence it
sis not difficult to make out that the members of the society are not interested in Working
It is not Where We Reach, but more importantly How we reach mattersGo on- 962 -.

A Management Paradise (un) Ltd .Endeavor.I,MP


of the society nor are they interested in Interfering in other members Life. Hence the
Members of the Sum Plaza Cooperative Society are living a Compartmentalized Life.
Also Refer to Duties and Responsibilities Members in the Paper.
5. As said earlier, Deputy Registrar has powers to Strip off the Secretary off his position,
but one thing has to be kept in mind, that after Such Strong Action, Who will come
forward for Societal Work.
6. A Registrars Prime duty is to check whether all cooperatives are working properly or not.
This working does not simply imply Proper Books of Accounts, but also means a proper
Congenial Environment in the Society.

(Refer Functions of Registrar, in the Same

Paper). The Friendly Duties of Registrar are also as important as Statutory Duties
7. Hence after saying that, if Nobody is willing to take up the Societal work, than control of
the Society will shift on to Registrar or Deputy Registrar as the Case may be.., in
which case there will be no irregularities in working, but will the members be satisfied,
and even if this happens then the Spirit of Cooperation will be Hampered amongst the
Members.

Section II

3) Define Cooperative marketing and state its objectives.


Solution:
For this refer paper 2004.

4) Explain the Importance of a result oriented professional management in Cooperatives


Sector.
Solution:
Refer paper 2002.

It is not Where We Reach, but more importantly How we reach mattersGo on- 963 -.

A Management Paradise (un) Ltd .Endeavor.I,MP


5) Explain Briefly 8ABARDs contribution in development of Cooperatives.
Solution:
Refer paper 2002 and paper 2001

6) Explain the important Provisions and Powers of the Registrar.


Solution:

The Registrar occupies crucial position in the co-operative system. He is the facilitator in the
implementation of the declared policy of the state-government towards rural development
through the co-operative sector. He is the most important media through which a democratic
government fulfills the social and economic aspirations of the people. He has also to coordinate between the government and the non-official leadership. He has also to maintain
administration realism and thus give correct advise to the popular government.
The Registrar of the co-operative society executes the co-operative law.


The powers in regard to Formation/Registration, to widening up of co-operatives remain


with the Registrar, while the financial help and other assistance is done by the co-operative
department.

The Registrar is assisted by the Assistant Registrar, District Registrar and other officers
appointed by the Co-operative Department.

The powers and functions of the Registrar are prescribed by the respected Co-operative
Acts. The Government can also delegate some additional powers to the registrar for
promoting co-operative movements.
Functions of the Registrar can be divided into:

1.

Statutory Functions: The statutory functions of the registrar are to carry out duties
entrusted on him by the Maharashtra Co-operative Acts 1960. These are registration,
amendment of byelaws, audit of Accounts, execution of awards, liquidation process etc. In
certain conditions the registrar has the authority to suspend the management of the society

It is not Where We Reach, but more importantly How we reach mattersGo on- 964 -.

A Management Paradise (un) Ltd .Endeavor.I,MP


for its malpractices in accounts. He also has to see the day-to-day affairs of the society are
carried out without hindrances. He is also responsible for the settlement of disputes between
and within societies.
2.

Development and Promotional Functions: The co-operative movement has to play an


important role in the social and economic development of India. The registrar being a
facilitator and coordinator of the developmental process, he has to do various functions as a
friend, guide and philosopher in relation to co-operative society.

Criticisms on the Role of the Registrar


The Registrar is being criticized that instead of being a friend, philosopher and guide of
societies he has become the inspector general of co-operative police carrying out functions
like imposition of penalties, suspension and filling legal charges against societies.
Excessive control of Registrar has lead to ill effects like corruption, rivalry between
societies, high handedness, bureaucratic delays and red-tapism.

EPICenter
"How do you make a fish aware of the water? Take it out of the water."

It is not Where We Reach, but more importantly How we reach mattersGo on- 965 -.

A Management Paradise (un) Ltd .Endeavor.I,MP


The fish does not think about where it is, it merely is, however take a fish out of the water and it
cannot survive, it struggles to get back to the water, put it back in and I think that it will be one
grateful fish.

How often do we only realise the value of what we had once we no longer have it. We take so
many things for granted, while we had it we did not appreciate it but as soon as it was gone we
missed it. Then we wish we could get it back.

How does a fish find the water? It doesn't, it is in the water, the water is in it. Never look for the
wonder of life, just take the time to become aware if it, for only when you are aware of the
wonder of life will you see it. Only when you are aware of the wonder of life can you truly start
to live.

Don't wait until you lose it before you appreciate it remember this question "How do you make a
fish aware of the water? Take it out of the water."

7)
a) Rights and Duties of Members:
The coop members are the legal owners of this cooperative. They have the power and
authority to make or break the coop.
Powers, Duties & Responsibilities of Members:

Adopt and/or amend the constitution and by-laws and agreements.

Elect and recall directors.

It is not Where We Reach, but more importantly How we reach mattersGo on- 966 -.

A Management Paradise (un) Ltd .Endeavor.I,MP


Vote upon appropriations for various purposes, to increase or decrease the capitalization, to
decide upon pooling practices and contractual arrangements between members and the
cooperative and to decide whether credit should be extended to patrons.

Hold directors and officers liable for any damage to the members.

Elect a competent committee to examine the records and audit the book periodically.

Examine the annual reports.

Dissolve or merge the cooperative.

Provide the necessary capital.

Control the cooperative.

Patronize the cooperative.

Assume business risks.

Pay operating costs.

Keep informed, and

Maintain the cooperative.

It is not Where We Reach, but more importantly How we reach mattersGo on- 967 -.

A Management Paradise (un) Ltd .Endeavor.I,MP


b) Sugar Cooperatives in Maharashtra:
Reference: Paper 2001

c) 8eed and Importance of Consumer Cooperatives:


Consumer Co-operative has its origin in the co-operative society formed by the 28 weavers
in England in 1844. The movement gradually developed in European countries and became
remarkably successful in Sweden, Denmark, Britain and Finland.
In Maharashtra the consumer movement began after the First World War mostly in urban
areas.
Objectives
1.

To encourage the habit of thrift, self-help and co-operation amongst the members of the
society.

2.

To raise funds by the way of share capital, deposits, loans, donations and entrance fees.

3.

To procure and produce by self or in partnership, consumer goods of daily use and
undertake retailing of such goods among the members and other consumers.

4.

To provide to the members such services as they may require as consumers.

5.

To undertake processing of consumers goods and its distribution amongst its members and
other consumers.

6.

To try to improve the quality of goods produced by the society and to construct or
purchase or obtain on hire godowns required for storing of goods.

7.

To take effective measures to ensure price stabilization and supply of goods to the
consumers at reasonable prices.

8.

To undertake distribution or sale of essential commodities under the authority given by the
state government or the local authority.

Dational Co-operative Consumer Federation (DCCF)


The National Co-operative Consumer Federation is the highest body of consumer cooperative in India. It was formed in the year 1966.Its headquarters are located in Delhi with
branches in various cities such as Mumbai, Chennai, Kolkatta, Nagpur etc.
It is not Where We Reach, but more importantly How we reach mattersGo on- 968 -.

A Management Paradise (un) Ltd .Endeavor.I,MP


Chief Function
1.

To help and guide state federations in their operations.

2.

To act as the chief spokesman of consumer co-operative in India.

3.

To procure and distribute consumer products such as:


a.

Pulses, spices, food grains, tea and other agricultural goods.

b.

Controlled cloth.

c.

Non-controlled textiles and readymade garments.

d.

Imported goods.

The federation maintains a consultancy and promotional cell, which is entrusted with the
responsibility of farming operational and business procedures, administrative norms etc. It
also publishes a monthly news bulletin, which contains information on the market rates of
goods supplied by the consumer co-operatives.
Problems
1.

The consumer co-operative movement in India is government sponsored and initiated.

2.

Consumers in India are indifferent to their own needs and still believe in the private trade
system.

3.

Consumer co-operatives are not well integrated and are scattered isolated.

4.

The procurement and purchase operations are technically faulty.

5.

They also face problems of low efficiency and low level of quality of products.

6.

There is also intense competition from the private traders who create various problems for
the consumer movement.

Different Types
1.

Primary Co-operative Society (Tolani College Co-operative)

2.

Departmental Stores or Super Bazaars (Apna Bazaar, Sarkari Bhandar)

3.

Central/Whole Sale Consumer Co-operative Stores

4.

State Federations of Consumer Co-operative

5.

National Co-operative Consumer Federation (NCCF Apex)

It is not Where We Reach, but more importantly How we reach mattersGo on- 969 -.

A Management Paradise (un) Ltd .Endeavor.I,MP

d) Contract Farming:
Contract farming can be defined as an agreement between farmers and processing and/or
marketing firms for the production and supply of agricultural products under forward
agreements, frequently at predetermined prices. The arrangement also invariably involves
the purchaser in providing a degree of production support through, for example, the supply
of inputs and the provision of technical advice. The basis of such arrangements is a
commitment on the part of the farmer to provide a specific commodity in quantities and at
quality standards determined by the purchaser and a commitment on the part of the company
to support the farmers production and to purchase the commodity. The intensity of the
contractual arrangement varies according to the depth and complexity of the provisions in
each of the following three areas:

Market provision: The grower and buyer agree to terms and conditions for the future sale
and purchase of a crop or livestock product;

Resource provision: In conjunction with the marketing arrangements the buyer agrees to
supply selected inputs, including on occasions land preparation and technical advice;

Management specifications: The grower agrees to follow recommended production methods,


inputs regimes, and cultivation and harvesting specifications.

With effective management, contract farming can be a means to develop markets and to
bring about the transfer of technical skills in a way that is profitable for both the sponsors
and farmers.

Thoughts to Live/Work By.:


It is not Where We Reach, but more importantly How we reach mattersGo on- 970 -.

A Management Paradise (un) Ltd .Endeavor.I,MP

Management Of Cooperatives November, 2006


Section I
Concepts

a)

Concept of One Person, One vote.

Solution: It Simply means that the number of people will decide on a Decision and not on the
Amount or Extent of Ownership in a Cooperative. This principle actually distinguishes the
Cooperatives from the Corporations. As here, the focus is also on the Collective Interests, and
then followed by Individual Interests, whereas in Corporates, it is Vice Versa.

It is not Where We Reach, but more importantly How we reach mattersGo on- 971 -.

A Management Paradise (un) Ltd .Endeavor.I,MP


The Aim of this principle is that Not even a Single Member is Deprived on Affecting the
Decision Making Process which might prove to be Significant to the Organization and overall to
all the Members,

b)

Objectives of Industrial Cooperatives.

Solution:
An industrial co-operative is an organization of workers and craftsmen engaged in the
cottage/village industries or other small-scale industries to undertake production,
purchase of supplies and raw materials, marketing of products and supply other
services to the members.
Aims and Objectives
Industrial co-operatives have 2 main aims:
1.

Social Aim: Social Aim is to safeguard workers interest and to protect them from
exploitation by the capitalist and the traders. Thus the social aim is to render services to the
members as well as the society.

2.

Economic Aim: The second aim is Economical, which focuses mainly on creation of
employment. There is lot of disguised unemployment and many layoff stages for the worker
in a year. This spare time can be made productive due to the industrial co-operatives.

Objectives
1. To develop self-reliance, co-operation and self-respect among the members.
2. To purchase and supply raw materials, tools and equipments needed by the workers.
3. To obtain contracts from the government and other organization.
4. To purchase machinery and other equipments required by members and to hire them to the
members for quality production.
5. To grant advances if needed by member workers against the securities of raw materials and
finished products.
It is not Where We Reach, but more importantly How we reach mattersGo on- 972 -.

A Management Paradise (un) Ltd .Endeavor.I,MP


6. To construct take on rents godowns for storage of finished products and raw materials.
7. To undertake all such activities needed for the welfare and well being of the member and
the society.
Kinds of Industrial Co-operatives
1. Workers co-operatives
2. Palm and gur society
3. Handicraft society
4. Leather co-operative
5. Other industrial co-operative
Organization Structure

It is not Where We Reach, but more importantly How we reach mattersGo on- 973 -.

A Management Paradise (un) Ltd .Endeavor.I,MP

c)

Functions of State Cooperative Bank

Solution: Refer Paper 2002

d)

Limited Interest on Capital

Solution: Refer Paper 2001

e)

Functions of the Chairman of Cooperatives.

Solution: Refer Paper 2002

Thoughts to Live/Work By.:

It is not Where We Reach, but more importantly How we reach mattersGo on- 974 -.

A Management Paradise (un) Ltd .Endeavor.I,MP


Case Study
Jai Kisan Cooperative Dairy is situated in Karveer Village in Kolhapur district. The Cooperative
dairy was set up in 1980 and functioned successfully till 1990. For the past five years it has been
suffering huge losses. The registrar of the Cooperative Societies has been receiving complaints
from the members of the society, accusing the management of faulty dealings. The registrar
terminates the managing body of the cooperative on the basis of the complaints.

Imagine that you have been appointed as the administrator of the cooperative diary and answer
the given questions by using your managerial competence.

a)

Identify the real problems before the dairy cooperative with the help of suitable

assumptions.
Solution:
1. Basically, the period of 1980s marked the importance of Green revolution, White
revolution etc. hence it can be well assumed that the villagers were enthralled by the
success of the Cooperatives, also were the successful models like Amul, on a Smaller
scales collective ventures were also successful. Hence, this transition to a Cooperative,
might not be a need, although they started on a very High Note, and were also able to
maintain that ensures that, with whatever motives they started, Once in to it, they were
Thorough Co-Operators.
2. There can also be Situational Analysis Done,. As in the dairy is situated in Kolhapur,
where people generally practice Farming. They do have small farmlands, and basically
practice Subsistence Farming. Hence although, the need was their to come together, they
still were performing on the Individual Level.
3. No Proper Planning.
4. Normally, when we account for a Machinery, we also take Depreciation in to account.
Although this D/P, actually helps to replenish the Asset when its life is over. But here in
this case there was no such thought given. Here, we even dont how the Current
Machineries were handled.
5. The Failure over here can be result of Anything, it can be a Mechanical Failure or the
Shortage of Output.

It is not Where We Reach, but more importantly How we reach mattersGo on- 975 -.

A Management Paradise (un) Ltd .Endeavor.I,MP


6. The members, over the period of time, should have started with some new ventures, like
introducing Credit Cooperatives which can share Financial Responsibilities, or they
could have also gone in for a Combination of a Existing Dairy and a Credit Co-Op.,
which would have proved useful during difficult times.
7. Members Disinterest: Members after all knowing about all the Faulty dealing of the
Management were still on the Complaining note. They could have easily taken a
Collective action to dissolve the management, or at least fixing those dealings instead of
being a mere Spectator.
8. Registrar Stepped in Late: The Corrective measure should have been proactive but it
wasnt. It is said that When Rome Burned, Nero Fiddled, similar case can bee seen
over here. When everything was on verge to break out, another failure of Registrar
stepping in was added.
9. No Decision is Still a Decision: The Members and the Villagers now cant keep on crying
and Dragging over the spilt milk. Instead what they should now focus on is that what can
be revived and how can the past be not made another future, at least not with the same
Intensity.

b)

What professional management solutions would you have suggest to achieve a

turnaround for the dairy cooperative?


Solution: Given Above are all the problems, Please refer to Them and Other Papers on Similar
Lines, to Answer this.

Thoughts to Live/Work By.:

It is not Where We Reach, but more importantly How we reach mattersGo on- 976 -.

A Management Paradise (un) Ltd .Endeavor.I,MP

Section II

3. What are the Problems faced by the Consumer Cooperatives in India?


Solution: Refer Paper 2005, Points are Precise, but you need to Elaborate on the same.

4. Explain the Principles of Cooperation in Detail.


Solution: Refer Paper 2001. Also refer to Social Responsibility part.
1) Open and Voluntary membership.
2) Democratic administration.
3) Self help and mutual help.
4) Principles of service.
5) Distribution of surplus.
6) Political and religious neutrality.
7) Principle of education.
8) Co-operation amongst co-operatives

But also Include: . Voluntary and opened membership, Democratic member control,
Member Economic Participation, Autonomy and Independence, Education, Training
and Information, Education for its members, Education for prospective co-operative
members, Co-operation among Co-operatives, Concern for community, Limited
interest on capital (Do profit motive), Equitable distribution of surplus, Self help and
Mutual help..

Dote: the above points are Principles of Cooperatives, but you need to include them also to
a limited Extent, so as not lose the sight of Cooperatives

It is not Where We Reach, but more importantly How we reach mattersGo on- 977 -.

A Management Paradise (un) Ltd .Endeavor.I,MP

5. Discuss the differences between Cooperatives and other forms of business.


Solution: Refer Paper 2001

6. Explain the rights and duties of a member of Cooperative Society.


Solution: Rights will form the part of principles of Cooperatives.

Duties of the Members:


1) Members Are Owners and Patrons: Members are obligated to patronize their
cooperative.
This varies from a loosely implied obligation to a legally binding contract between members and
their association to patronize it on a specific basis.

2) Understand the Cooperative


Members are responsible for understanding their cooperative- its purpose, objectives, benefits,
limitations, operations, finances, and long-range plans.
They should read the articles of incorporation and bylaws to understand the purpose of their
cooperative in serving its members.
Members must also understand that laws limit their rights or powers and those of their boards of
directors. They must operate within the framework of statutes and codes in performing any
function. Members also need to understand that bylaws or policies of their elected directors may
further limit their operations by establishing member obligations, regulations, and quality
controls exceeding those prescribed by legal statutes.

Members must understand the need for long-range planning for services, facilities, and capital.
Cooperatives have had a part in bringing many technological changes to the business of
agriculture; changes continue, however, indicating that cooperatives need to update their longrange plans continually through research and study based on projected member needs.

Members can accelerate this long-range planning by keeping directors and hired management
informed.

It is not Where We Reach, but more importantly How we reach mattersGo on- 978 -.

A Management Paradise (un) Ltd .Endeavor.I,MP

3) Adopt Legal Papers


Members are responsible for understanding, adopting, and amending legal papers. They should
read and understand the cooperatives articles of incorporation and bylaws to know how the
business is to operate and what services it can provide.

They need to analyze proposed bylaw amendments and determine how they will affect the
association before voting to approve them. Members are responsible for knowing the contents of
legally enforceable marketing contracts or other legal papers before signing them so they can
fulfill their obligation to the business.

4) Select and Evaluate Directors


Members should be intimately familiar with the cooperative and as owners assume a positive,
broad role in its management. Although the cooperative is a democratically controlled form of
business, members cannot make all decisions directly. They elect directors from their
membership to represent them in most affairs of management.

Members should understand and evaluate board actions. Directors not assuming responsibilities
entrusted to them, including complying with laws governing cooperatives, should be removed.
Members must exercise sound judgment and avoid unnecessary pressure on directors. Member
control of any cooperative is, to a large degree, determined by directors actions. Members must
encourage and challenge directors and management to provide needed services.

5) Provide Decessary Capital


Basically, members must provide equity capital for the cooperative business. Most cooperatives
have a small beginning and find it necessary to borrow much of the needed funds.

Later, as they become established and business services expand, cooperatives generally find it
neither necessary nor wise to rely on member capital to meet all financial needs. The member or
equity capital is used as a base to apply for a loan. Members investment in their cooperative
must increase this base before borrowing power can be increased.
Members must be prepared to make necessary changes in capital structure, including increases
and decreases, when member investment requirements change.
It is not Where We Reach, but more importantly How we reach mattersGo on- 979 -.

A Management Paradise (un) Ltd .Endeavor.I,MP


Members must understand these financing plans if they are to provide adequate equity capital.

6) Use and Support the Cooperative


Members must supply an adequate volume of business if the cooperative is to operate efficiently
and successfully. Through membership or marketing agreements, they should commit most or
all of their marketing or supply purchases to the cooperative each year.

The strength of the bond between members and their cooperative is a most important factor in
cooperative success. Underlying this bond is interest that develops loyalty to the organization
and its objectives.

The sense and pride of ownership responsibility means more than merely patronizing the
business and reaping financial benefits. It involves more than building an organization with
adequate volume to employ talented personnel to operate the business in the best interest of
members. It is molding the cooperative into a force that influences the growth, development, and
business climate of the members and community and helps shape the destiny.
Lack of member support is one weakness that can retard cooperative growth. Effective member
involvement beyond just patronage gives the cooperative the necessary boost maintain its
competitiveness as a business, and keep atuned to its member-owners needs.

Also the following things should be kept in mind:


(1) Know how groups function and make them successful through providing opportunities to
initiate ideas and discuss proposed actions;
(2) accent member relations because most members have not experienced the inside feeling of
involvement of the founders-especially in todays larger cooperatives;
(3) run the business efficiently;
(4) provide upto- date service; and
(5) remember that little things count big in developing member loyalty.

7) Participate in Affairs
Members participation in affairs of their association increases the feeling of ownership and
responsibility for success.

It is not Where We Reach, but more importantly How we reach mattersGo on- 980 -.

A Management Paradise (un) Ltd .Endeavor.I,MP


But encouraging this participation requires planning and considerable effort. Boards and
managers must want members to participate and must understand the requirements to keep them
active. Managements function in providing for the two way flow of information can create
member interest, enthusiasm, loyalty, and responsible participation.

Meetings-annual, educational, and special-offer a good means of member group participation,


especially where member questioning is promoted. An important aspect of successful member
participation is providing adequate time for explanation and discussion. While suggestions and
complaints will surface, this gives management an opportunity to determine the underlying
causes.

Members want to question the decision makers directly, thus increasing the importance of this
exchange with management.

Members themselves must make an effort to participate.


They need to attend all meetings for members, and encourage others to go. They should discuss
the problems, express opinions, and generate ideas for better or new services. Members should
vote on issues presented and accept the decision of the majority.
Members should accept special committee assignments when asked. This will help them realize
the cooperative belongs to them and that it should be treated as a business.

Members can be good business citizens of their association by learning about it and other
cooperatives. They should plan, participate, and share costs in educational programs targeted at
nonmembers and the general public. Public knowledge about cooperatives affects attitudes and
influences legislation. Members and other cooperative-minded people should let legislators
know their wishes.

8) Provide Information
Members are a cooperatives unique resource, and when properly informed, they make
worthwhile contributions.
Members must be provided both good and bad facts about their cooperative- w h a t it is, how it
functions, what are its problems, and options or choices they can help make.

It is not Where We Reach, but more importantly How we reach mattersGo on- 981 -.

A Management Paradise (un) Ltd .Endeavor.I,MP


Todays cooperatives are larger and require greater operating skills as they assume more of
producer-members work and services. The entire team operating the cooperative-members,
directors, manager, and employees-must know the limitations and capabilities of the cooperative
for it to appeal to members as a way of improving their net income and welfare.
Top management-including directors-can most effectively develop this member resource.

Management must initiate member education to achieve two-way communication and member
participation.
.
Members must keep an open mind and be alert for new services needed and new ways to make
the cooperative more effective.
They have a responsibility to provide this information to their directors and hired management.
Members place great responsibility on directors they elect. They must share this responsibility
by supplying directors with information on needed purchasing, marketing, or other related
services. Informed directors can better serve members by establishing board policy, planning for
needs with hired management, and voting when decisions need to be made.

9) Help Obtain Dew Members


A cooperative must continually replace members who move away, quit, or die. In some cases,
additional members may be needed to financially strengthen the association or increase its
volume.
Present members can help by encouraging their neighbors to patronize the cooperative and
become members. The business operation of a cooperative and the statutes under which it is
organized influence its membership policies and requirements. These policies have a bearing on
member responsibilities to increase or maintain membership.

People joining a cooperative should understand what a cooperative is, how it operates, its
possible benefits, and its limitations. Contrary to some beliefs, few people join cooperatives
because of social objectives but rather for economic benefits services and increased income.
Most people want to be shown the advantages arising from cooperative membership, and unless
there are real advantages, few prospective members will join; or if they do, they will not
patronize it.

It is not Where We Reach, but more importantly How we reach mattersGo on- 982 -.

A Management Paradise (un) Ltd .Endeavor.I,MP

Thoughts to Live/Work By.:

7. Short Dotes.
a) Cooperative Leadership: Refer Paper 2004

b) Role of DABARD in relation to Cooperatives: Refer Paper 2002, 2001

c) Cooperative Movement in United Kingdom: Refer Paper 2005, Rochdale Model, and
also add the following:
The rules and regulation followed on the transaction of the Rochdale stored where:
Open Membership
Business on demonstration basis
Limited Interest on capital
Division of profits as per transactions by members
Cash payment
Supply of best quality goods
Educating Members
Religious Neutrality.
The Rochdale Pioneers of England framed the principles based on their experience on a
consumer co-operative store; popularly know as the Rochdale Principles. These principles have
been adopted in almost all countries, which have same or other type of co-operatives.
Principles
It is not Where We Reach, but more importantly How we reach mattersGo on- 983 -.

A Management Paradise (un) Ltd .Endeavor.I,MP


Co-operative Principles govern the life activity of a co-operative enterprise. The Principles are
as follows:
1. There should be democratic control
2. All should be welcomed in it
3. Limited interest should be paid on capital
4. All trading should be made in cash
5. No politics should be allowed
6. Co-operation should be kept free from religion
7. There should be no faults in co-operation education

d) Sugar Cooperatives in Maharashtra: Refer Paper 2001

It is not Where We Reach, but more importantly How we reach mattersGo on- 984 -.

A Management Paradise (un) Ltd .Endeavor.I,MP

India's White Revolution In Milk Production


That the island nation of Sri Lanka has sought India's expertise and know-how for the
development of its dairy industry, is a testimony to the success of the "White Revolution"
conceived by the National Dairy Development Board (NDDB) of India. The Anand bused
NDDB is now in the process of helping Sri Lanka launch an "Operation Flood" aimed at
ending milk shortage and improving genetic quality of its milk animals. The US$ 20 million
joint venture between India and Sri Lanka, named the "Kiriyo Milk Industries of Sri Lanka", is
headed by Dr. V.Kurien, Chairman, NDDB. The NDDB, which has invested Rs. 250 million in
this joint venture, has helped set up three hundred thousand litres per day plant in this island
nation which imports 80% of its milk and dairy products involving an outgo of foreign exchange
to the tune of five billion Sri Lankan rupees. The joint venture dairy project would adopt the
wider acclaimed Anand Co-operative model.
In the immediate future, the project aims at trebling Sri Lanka's milk production from 30,000
litres per day to 10,000 litres per day production capability. Besides setting up milk cooperative societies throughout the island nation by adopting "Operation Flood" strategy, the joint
venture also aims at improving the genetic quality of the milk animals as the milk yield in Sri
Lanka is five litres per day per animal as compared with 30 litres per day yield in India.
At present, there are 300,000 dairy formers in Sri Lanka and the aim is to increase the number to
one million in a phased manner. On another front, the project seeks to provide an assured
market to milk producers at remunerative prices which is expected to increase milk production.
It is a tribute to the excellence India has achieved in all aspects of dairy production that the
European Dairy Association (EDA) has expressed its readiness to explore ways and means to
collaborate with NDDB to expand the dairy sector in Asia.

It has not been lost on the

international dairy community that India, today, is the most energy and coal efficient producer of
high quality milk in the world. As the latest Annual Report of NDDB notes, the most significant

It is not Where We Reach, but more importantly How we reach mattersGo on- 985 -.

A Management Paradise (un) Ltd .Endeavor.I,MP


contribution of the Anand pattern is that India has achieved a substantial advantage in most dairy
commodities

The Amul Saga


Sourced from Amul Official Website
THE success of the Amul brand name has, no doubt, resulted in my being asked to comment on
its history and the reasons for its success. I have, therefore, reflected on the long history of the
brand to see if I could distil reasons why Amul is a name widely recognized and respected, not
just in our cities and towns, but in our villages as well.
Probably the easy, but nonetheless wrong, answer is that Amul has been advertised well.
Certainly it has helped that those responsible for keeping the Amul name in the public eye have
used considerable imagination and, if I do say so, The taste of India is nothing short of
brilliant. However, there is much more to it.
A successful consumer product is the object of thousands, even tens of thousands of transactions
every day. In these transactions, the brand name serves in lieu of a contract. It is the assurance to
the buyer that her specifications will be met. It is the sellers assurance that quality is being
provided at a fair price.
If Amul has become a successful brand if, in the trade lingo, it enjoys brand equity then it is
because we have honoured our contract with consumers for close to fifty years. If we had failed
to do so, then Amul would have been consigned to the dustbin of history, along with thousands
of other brands.
The tough part of the use of a brand as a contract is that every day is a renewal; if, just once, the
brand fails to meet the customers expectations or, more exactly, if it fails to delight the
customer, then the contract loses its value. If Amuls sales continue to rise, it is because that
contract has been honoured, again and again. I would like to think that the granddaughters of
some of our first customers are now contracting with us to buy their butter, cheese, baby food,
chocolates and other fine Amul products. It is also a fact that when we first thought of exporting
It is not Where We Reach, but more importantly How we reach mattersGo on- 986 -.

A Management Paradise (un) Ltd .Endeavor.I,MP


to West Asia and even to the United States, it was because of the loyalty of Amul customers
who, even when far from home, still craved our taste of India.
What goes into the contract that is a brand name? First is quality. No brand survives long if its
quality does not equal or exceed what the buyer expects. There simply can be no compromise.
Thats the essence of the contract. In the case of a food product, this means that the brand must
always represent the highest hygienic, bacteriological and organoleptic standards. It must taste
good, and it must be good.
Second, the contract requires value for money. If our customer buys an Amul product, she gets
what she pays for, and more. We have always taken pride in the fact that while we earn a good
income for our owners the dairy farmers of Gujarat we dont do it at the cost of exploiting
the consumer. Even when adverse conditions have reduced supplies of products like butter, we
have resisted the common practice of raising prices, charging what the market would bear.
Rather, we have kept prices fair and done our best to ensure that retailers do not gain at the
consumers expense.
The third element of the contract is availability. A brand should be available when and where
the customer wants it. There is no benefit achieved in creating a positive brand image, and then
being unable to supply the customer who wants to buy it. In our case, over the years we have
built what is probably the nations finest distribution network. We reach hundreds of cities and
towns through a cold chain that not only ensures that our products are available, but they reach
the customer at the farthest end of the country with the same quality as you would find in
Ahmedabad or Vadodara.
The fourth part of the contract is service. We have a commitment to total quality. But,
occasionally, we may make a mistake or, our customer may think weve made a mistake, and
the customer, as they say, is always right. That is why, for Amul, every customer complaint
must be heard not just listened to. And, every customer complaint must be rectified to the
extent humanly possible.
For close to fifty years now, Amul has honoured its contract with the consumer. The contract
that is symbolised by the Amul brand means quality. It means value for money. It means
availability. And it means service.

It is not Where We Reach, but more importantly How we reach mattersGo on- 987 -.

A Management Paradise (un) Ltd .Endeavor.I,MP


How did the Amul brand become what it is? To answer that, we must journey back in time, to
the history books, to the time of Indias independence because Amuls birth is indelibly linked
to the freedom movement in India. It was Sardar Vallabhbhai Patel who said that if the farmers
of India are to get economic freedom then they must get out of the clutches of the middlemen.
The first Amul co-operative was the result of a farmers meeting in Samarkha (Kaira district,
Gujarat) on 4 January 1946, called by Morarji Desai under the advice from Sardar Vallabhbhai
Patel, to fight rapacious milk contractors. It was Sardars vision to organize farmers, to have
them gain control over production, procurement and marketing by entrusting the task of
managing these to qualified professionals, thereby eliminating the middle men, the bane in
farmers prosperity.
The decision was taken that day in January 1946: Milk producers co-operatives in villages,
federated into a district union, should alone handle the sale of milk from Kaira to the
government-run Bombay Milk Scheme. This was the origin of the Anand pattern of cooperatives. The colonial government refused to deal with the co-operative. The farmers called a
milk strike. After fifteen days the government capitulated. This was the beginning of Kaira
District Co-operative Milk Producers Union Ltd., Anand, registered on 14 December 1946.
Originally the Anand pattern included dairy co-operative societies at the village level, and a
processing unit called a union at the district level. Inspired by the Kaira Union, similar milk
unions came up in other districts too. In 1973, in order to market their products more effectively
and economically, they formed the Gujarat Co-operative Milk Marketing Federation Limited
(GCMMF Ltd.). GCMMF became the sole marketer of the original range of Amul products
including milk powder and butter. That range has since grown to include ice cream, ghee,
cheese, chocolates, shrikhand, paneer, and so on. These products have made Amul a leading
food brand in India.
The brand name AMUL, from the Sanskrit Amoolya, meaning priceless, was suggested by a
quality control expert in Anand. The first products with the Amul brand name were launched in
1955. Since then, they have been in use in millions of homes in all parts of India, and beyond.
Today Amul is a symbol of many things: Of high quality products sold at reasonable prices, of
availability, of service.
There is something more, though, that makes the Amul brand special and that something is the
reason for our commitment to quality and value for money. Amul is the brand name of 2 million
It is not Where We Reach, but more importantly How we reach mattersGo on- 988 -.

A Management Paradise (un) Ltd .Endeavor.I,MP


farmers, members of 10,000 village dairy co-operative societies throughout Gujarat. This is the
heart of Amul, it is what gives strength to Amul, and it is what is so special about the Amul
saga.
In the early days of Kaira Union there was no dearth of cynics. Could natives handle
sophisticated dairy equipment? Could western-style milk products be processed from buffalo
milk? Could a humble farmers co-operative market butter and cheese to sophisticated urban
consumers? The Amul team farmers and professionals confounded the cynics by processing
a variety of high-grade dairy products, several of them for the first time from buffalo milk, and
marketing them nationally against tough competition.
What began way back in 1946 was really an effort to carve out a truly Indian company that
would have the involvement of millions of Indians and place direct control in the hands of the
farmers. It was a mandate for producing, owning and marketing and above all, building your
own truly Indian Brand. And successfully at that.
You will appreciate that when the lives of lakhs of farmers depend on a brand, and when your
history is grounded in the Independence movement, when not only competitors but even your
own government questions you, then your resolve to be the best is like the finest steel.
Amul, therefore, is a brand with a difference. That difference manifests itself in a larger than life
purpose. The purpose freedom to farmers by giving total control over procurement, production
and marketing. Amul and all other milk products produced by co-operatives were born in
struggle. It was the producers struggle for command over the resources that they create, a
struggle to obtain equitable returns and a struggle for liberation from dependence on middlemen.
It was a struggle against exploitation. A refusal to be cowed down in the face of what others
believed to be the impossible.
Amuls birth was thus a harbinger of the economic independence of our farmer brethren. Amuls
mission was the development of farmers, nutrition to the nation, and heart in heart, the real
development of India.
Given Indias vast geographical spread, the country had very few dairy plants at the time of
independence. As the then Prime Minister Lal Bahadur Shastri had said, One Amul is not
sufficient. Many Amuls are the need of the hour. This led to replication of the Anand pattern
through the Operation Flood programme which has, amongst others, three major achievements
It is not Where We Reach, but more importantly How we reach mattersGo on- 989 -.

A Management Paradise (un) Ltd .Endeavor.I,MP


to its credit, namely: making dairying Indias largest self-sustainable rural employment
programme, bringing India close to self-sufficiency in milk production, and trebling the nations
milk production within a span of two and a half decades to make India the worlds largest milk
producer.
Today, 173 milk producers co-operative unions and 22 federations play a major role in meeting
the demand for packed milk and milk products. Quality packed milk is now available in more
than 1,000 cities throughout the length and breadth of India. And this is milk with a difference
pasteurized, packaged, branded, owned by farmers carrying the milk drop logo, like Amul, a
symbol of quality.
Over the course of Operation Flood, milk has been transformed from a commodity into a brand,
from insufficient production to self sufficient production, from rationing to plentiful availability,
from loose, unhygienic milk to milk that is pure and sure, from subjugation to a symbol of
farmers economic independence, to being the consumers greatest insurance policy for good
health.
What of the future? Indias population has risen from 350 million in 1950 to 1,000 million
today. As cities draw people to new employment opportunities, the current urban-rural ratio of
26:74 is likely to become 33:67 by the year 2010. As per available projections, the population by
the year 2010 would touch 1,190 million people. This means that by the year 2010, rural India
will be required to support some 800 million people, an increase of 11% over 1999s 720 million
rural people.
Based on the current population demographics and projections, we estimate that there will be
260 million women in the age-group of 15-59 years in India by the year 2010 and this would
further increase to 302 million by the year 2020, of which only 100 million would be literate.
This means that rural women will comprise 21% of Indias total population. In our country, most
rural women contribute to agricultural and dairying activities apart from the household work
and their activities are not included in Indias GDP despite their significant contribution.
Dairying is, therefore, very important to our rural women. For many, it is their main source of
employment and income, incomes that they often manage themselves.
Population gives us one picture. The other is provided by the demand for household
commodities. By 2010, the national requirement for food grains will touch 266 million metric

It is not Where We Reach, but more importantly How we reach mattersGo on- 990 -.

A Management Paradise (un) Ltd .Endeavor.I,MP


ton, rising to 343 mmt by 2020. For milk, estimated consumption will be 153 mmt by 2010 and
271 mmt by 2020. For edible oils, demand will soar to 9 mmt by 2010 and 13 mmt by 2020.
It should be clear that agriculture will remain the most important engine of our economy. Amul
and its co-operative sister brands are aware of this challenge. The future, they say, is at best a
mystery. But, it should be clear that the needs of a nation on the move must be met. The country
is young. There are more working women. The needs of an ever-growing population have to be
met with sustainable economic development. And the demand for milk and milk products,
therefore, is only going to grow further. Couple this with the nutritional needs of the new and the
old generations and it is equally clear that there will be a need for more value added milk
products. This calls for production to be enhanced at even faster rate than it is at present.
There is also something very special about milk, something which requires that any brand for
milk and milk products to act not simply as a seller, but as a trustee. Milk is not a white good or
a brown good. It is not something people save their entire lives in order to buy like a car, or a
house. Milk is not a status symbol; rather it is the symbol of nutrition. Milk is a nearly complete
food, providing protein, vitamins, minerals and other nutrients so essential to maintaining good
health.
We realise the value of milk on the day the milkman does not bring it to our doorstep, when our
children have to go to school without it, when we go without our daily cup of coffee or tea. And
what would our lives be like without ghee, butter, cheese, curd, lassi, chaas and the like. Milk is
not only an ingredient in our favourite recipes, it is an essential ingredient of life itself. And, by
its very indispensable nature, it has one of the biggest markets a whopping 82 mmt at a very
conservative consumption of just 214 grams per day per person in India alone.
Our commitment to the producer, and our contract with the consumer are the reasons we are
confident that co-operative brands, like Amul, will have an even bigger role to play in the next
fifty years. Resources need to be deployed with a purpose and a commitment to deliver better
results. There is no limit for a marketing exercise then. It must build India and its culture a
second time round.
An India, that is the land of our dreams

It is not Where We Reach, but more importantly How we reach mattersGo on- 991 -.

A Management Paradise (un) Ltd .Endeavor.I,MP

Thoughts to Live/Work By.:

EPICenter
"The greatest thing about the future, is that it is never too late to change it."
The future is given to us as an endless set of possibilities and it is up to each one of us the
determine the future that we want and then to take the appropriate action to ensure it's
achievement.

As humans we have the greatest power of all living creatures, we have the power of choice. We
can choose how we live, how we behave, what we think and what we say.

The first step in taking control of our lives and thus of our future is to realize that we can take
control. Far to many people feel as if they have no control of their lives, that they are merely
pawns in the game of life and are controlled by circumstance.

This is not true, but until you can accept your own power it will be true for you. The first thing
that you have to do is to learn to love and respect yourself, to realise that you are a wonderful
person and that you have unlimited potential. Accept that you are engineered for success,
designed for accomplishment and endowed with the seeds of greatness. Within you lies the ability
to the most wonderful things.
Every choice you make changes the future for every choice has a consequence. Only when you
make the choice to acknowledge your power and use it to control your every action until it
becomes a habit will you be able to act with confidence and take control of your future.
While you can never go back and make a new start it is never to late to start working towards a
new ending. Realise that the past is not a reliable indicator of the future, you can at any stage
change your future by changing your actions now.

It is not Where We Reach, but more importantly How we reach mattersGo on- 992 -.

A Management Paradise (un) Ltd .Endeavor.I,MP


The magic of life is that it does not matter how you have lived up to now what matters is how you
live from now. It does not matter what you have done up to now what matters is what you do
from now.
So choose the future that you would like to have, work out what you have to do to get it and then
get to work to create the future that you want because
"The greatest thing about the future is that it is never too late to change it."

MANAGEMENT OF CO-OPERATIVES NOV 2007


D.B.

(1) All questions in section 1 are compulsory


(2) Attempt any three out of five questions from Section 2.
(3) Figures to the right indicate maximum marks.

Section 1
(1) Analyze the theoretical meaning and practical significance of the following:
(a) Interest on capital
For this question, refer 2004[section 1]

(b) Role of treasurer in a cooperative :


The position of treasurer of a cooperative is similar to that of Chief Financial Officer at a
corporate business, but without pay. The treasurer, ideally, has a background in finance
It is not Where We Reach, but more importantly How we reach mattersGo on- 993 -.

A Management Paradise (un) Ltd .Endeavor.I,MP


or business, understands financial statements, internal controls and budgeting and is
knowledgeable about investments, investment procedures, yields, risks and investment
costs. A working knowledge of corporate taxes is also a plus. The treasurer must have
the dedication and the available time to fulfill his duties and must maintain a line of
communication with management, the attorney and the accountant.

(c) Open membership


For this question, refer paper 2003[section 1]

(d) Professionalism in cooperatives


For this question, refer paper 2002[section 2]
(e) Dominal member in a cooperative
Traders with whom the society establishes business dealings are enrolled as nominal
members. Nominal members do not have the right to participate in decision making
and share in the profits.

Nominal membership in Urban Cooperative Banks was introduced only to enable


borrowers to occasionally borrow against pledge of tangible securities such as gold,
government securities and NSCs, etc. Besides, if a person has to stand as a surety to a
borrower, he has to necessarily be a nominal member under the cooperative law. Thus,
nominal members are those, who are not interested in the management of the bank but
desire to avail of loan facilities at infrequent intervals, who donot subscribe to the share
capital of the bank, and hence, are not eligible for dividend. Besides, they also do not
have voting rights, nor can they contest elections to the Boards of the Banks.

(2) Imagine that you are the C.E.O of a large and respectable organization and that one of your
task is to address your own employees, driving home the point of the necessity of adopting the
basic principles of cooperation in the corporate society.
Your speech should cover the following :

a) The important percepts of cooperation.


b) The present short-comings in the corporate way of life.
c) The role of co-operation in making corporate more focused and purposive.
It is not Where We Reach, but more importantly How we reach mattersGo on- 994 -.

A Management Paradise (un) Ltd .Endeavor.I,MP

Section 2
(3) Cooperation is one of the proven ways of reducing conflicts in a society. Elaborate
by giving suitable percepts and practical examples.
Conflict is when two or more values, perspectives and opinions are contradictory in nature and
haven't been aligned or agreed about yet, including:
1. Within yourself when you're not living according to your values;
2. When your values and perspectives are threatened; or
3. Discomfort from fear of the unknown or from lack of fulfillment.
Conflict is inevitable and often good, for example, good teams always go through a "form,
storm, norm and perform" period. Getting the most out of diversity means often-contradictory
values, perspectives and opinions.

It is not Where We Reach, but more importantly How we reach mattersGo on- 995 -.

A Management Paradise (un) Ltd .Endeavor.I,MP


Conflict is a problem when it:
1. Hampers productivity.
2. Lowers morale.
3. Causes more and continued conflicts.
4. Causes inappropriate behaviors.
Co-operation means working together or, as defined by one dictionary `working together to
share the profits. Perhaps this is the essence of teamwork that people put the teams objectives
before their own and share both the financial and psychological rewards of their efforts. Cooperation implies that individuals are committed and willing to be involved in the work they do,
and that they are ready to share their skills and information with the rest of the team, knowing
that the others will reciprocate. People trust each other and encourage others to use their ideas.
Everyone in the team is open about their strengths and weaknesses, knowing that they are
accepted. This in itself places a great responsibility on the team to foster and maintain the spirit
of cooperation and one finds that the members of the team remind and help individuals who
may be falling behind with some task.
(4)What do we learn from the theory and practice of cooperation in England, Germany
and Israel?

Refer to History of Co-operatives in England, Germany and Israel.

(5) Spell out and discuss the major problems in areas in sugar cooperatives in
Maharashtra.
For this question, refer paper 2003[ section 2]

(6) The Registrar of cooperative societies is the friend, philosopher and guide of the
cooperative movement. Discuss.
For this question, refer paper 2001 [section 1]

(7) Write short notes (any two):


(a) KVIC
For this question, refer paper 2001[short notes]

(b) Chairman of a cooperative society


It is not Where We Reach, but more importantly How we reach mattersGo on- 996 -.

A Management Paradise (un) Ltd .Endeavor.I,MP


For this question, refer paper 2002[section 1]

(c) Consumer cooperatives roles and functions


Consumer Co-operative has its origin in the co-operative society formed by the 28
weavers in England in 1844. The movement gradually developed in European countries and
became remarkably successful in Sweden, Denmark, Britain and Finland.
In Maharashtra the consumer movement began after the First World War mostly in urban
areas.
 Objectives
1.

To encourage the habit of thrift, self-help and co-operation amongst the members of the
society.

2.

To raise funds by the way of share capital, deposits, loans, donations and entrance fees.

3.

To procure and produce by self or in partnership, consumer goods of daily use and
undertake retailing of such goods among the members and other consumers.

4.

To provide to the members such services as they may require as consumers.

5.

To undertake processing of consumers goods and its distribution amongst its members and
other consumers.

6.

To try to improve the quality of goods produced by the society and to construct or
purchase or obtain on hire godowns required for storing of goods.

7.

To take effective measures to ensure price stabilization and supply of goods to the
consumers at reasonable prices.

8.

To undertake distribution or sale of essential commodities under the authority given by the
state government or the local authority.
 Chief Function

1.

To help and guide state federations in their operations.

2.

To act as the chief spokesman of consumer co-operative in India.

3.

To procure and distribute consumer products such as:

It is not Where We Reach, but more importantly How we reach mattersGo on- 997 -.

A Management Paradise (un) Ltd .Endeavor.I,MP


a.
Pulses, spices, food grains, tea and other agricultural goods.
b.

Controlled cloth.

c.

Non-controlled textiles and readymade garments.

d.

Imported and confesticated goods.

The federation maintains a consultancy and promotional cell, which is entrusted with the
responsibility of farming operational and business procedures, administrative norms etc. It
also publishes a monthly news bulletin, which contains information on the market rates of
goods supplied by the consumer co-operatives

(d) The impact of Globalisation on cooperatives


Cooperatives have historically played a significant role in promoting agricultural and rural
development. Cooperatives enable farmers to retain control on the value chain for their own
agricultural produce. A cooperative form of organisation, not only helps in reducing transaction
cost, it also enhances the bargaining power of producers vis--vis their customers as well as
input-service providers. Cooperatives have several inherent advantages over other forms of
organisations in terms of ensuring members participation, mobilizing peoples resources and in
securing integration of various functions. They also effectively ensure equitable distribution of
benefits of development.

Any visitor to rural India can perhaps observe that on parameters such as literacy levels, life
expectancy at birth and per capita income, those rural areas which have dairy cooperatives,
seem to have done much better, as compared to other rural regions. The vulnerable section of
rural populace, including children, expectant and nursing mothers in villages having dairy coops, have much better nutritional status than their counterparts in other villages (Singh & Das,
1984). Even the landless households in cooperative villages have significantly better quality of
life than their counterparts in other villages. Farmer-suicides are unheard of, in districts where
dairy cooperatives flourish.

Today, in the era of globalisation, privatisation and liberalisation, cooperatives are facing a new
set of challenges. It is true that cooperatives need a supportive environment, in order to flourish.
At the same time, cooperatives should also address their own unique set of issues and prove to
the world that they are worthy of support. The quality of leadership in cooperatives has to be
It is not Where We Reach, but more importantly How we reach mattersGo on- 998 -.

A Management Paradise (un) Ltd .Endeavor.I,MP


dedicated, mature, committed and loyal to the cause of farmers. While the central and state
government have done their best to help the cooperative sector, their continuous support is
essential for cooperatives to continue their lead role in rural development. The various state
governments should ensure that they provide complete autonomy to cooperative organisations
and avoid intervening directly in their affairs through deputations.

With one-third of our rural population, living below the poverty line and with most of the rural
producers surviving at the mercy of market intermediaries, it is unlikely that any free market
model of development will help them. Under the new world trade regime, ushered in by WTO,
only cooperatives can protect the poor farmer and the consumer, from the adverse effects of free
world trade. In the absence of a strong cooperative sector, both the producers and the consumers
would be subjected to exploitation by the private sector, as used to happen in the pre-cooperative
days.

The cooperative sector has a bright future in India, if they retain their core identity as memberowned autonomous organisations governed by truly elected representatives, managed
professionally and led by visionary leaders. Today, cooperatives have to strengthen and intensify
their competitive advantages to retain their foothold in the fiercely competitive world economy.
An enabling legal framework would go a long way towards facilitating cooperatives in
enhancing their contribution towards national development.

It is not Where We Reach, but more importantly How we reach mattersGo on- 999 -.

A Management Paradise (un) Ltd .Endeavor.I,MP


A Management Paradise Endeavor..
Project Sponsored by www.ManagementParadise.com

Because it has lived its Life intensely,


the parched grass still attracts the gaze of the passers-by,
the flowers merely flower,
and they do this as well as they can,
the white lily, blooming unseen in the valley,
does not need to explain in itself to anyone;
it lives merely for the beauty.
Men, however, cannot accept that merely

If tomatoes wanted to be melons,


They would look completely ridiculous.
I am always amazed
That so many people are concerned
With wanting to be what they are not;
Whats the point of making yourself look ridiculous?

You dont always have to pretend to be strong,


theres no need to prove all the time that everything is going well,
you shouldnt be concerned about what other people are thinking,
Cry if you need to,
its good to cry out all your tears.
(because only then will you be able to smile again)
-

Mitsuo Aida

MP Team

..regardsi, MP..

It is not Where We Reach, but more importantly How we reach mattersGo on- 1000 -.

A Management Paradise (un) Ltd .Endeavor.I,MP

HOW CAN YOU HELP US?


US?
Weve tried to do our best in the past years to help Management
Students by publishing books and circulating notes and sponsoring
festivals and helping them in their project reports and the list goes on
and on.
And weve got valued love and support of our members in return.
We hope the community feeling keeps fostering and students keep
contributing to the forum in terms of helping each other and
enriching their knowledge.
Your research reports are Your sweat, Your efforts,
efforts, Your brainchild.
Why leave it on your desktop where it wouldnt get any recognition?
Why hide a gem of information from your own community?
Upload Your Research reports, Spread the word,
Get RECOGNITION, Help Others and Help Yourself !

It is not Where We Reach, but more importantly How we reach mattersGo on- 1001 -.

A Management Paradise (un) Ltd .Endeavor.I,MP

When Companies
Companies are laying people off, were recruiting
If you dream to revolutionize Management Education mail
youre resume on career@managementparadise.com with the
subject line of I LOVE MP. Make sure you mention the job
profile that interests you the most and the expected CTC.

If you are bitten by the Entrepreneurial Bug and


have a business idea in which you need help. Mail us
with your business plan or alliance proposals on
bizdev@managementparadise.com

We DO NOT charge our members, we earn through


advertisements. Want to support MP? Make a
donation or advertise on MP.
For more details, mail us on
marketing@managementparadise.com
marketing@managementparadise.com
It is not Where We Reach, but more importantly How we reach mattersGo on- 1002 -.

BMS
COACHING
CLASSES
A Management
Paradise (un) Ltd
.Endeavor.I,MP
Do You Run BMS COACHING CLASSES?
Want to Include your Coaching classes detail in this book ?
Reach your Audience !
Contact us on bizdev@managementparadise.com
bizdev@managementparadise.com

Score
 BMS EXPERT
(Prof.Jhabak)
Join us in sem6
Tel no. 9324343830
TUTORIALS FOR BMS.

Padma commerce classes


Prof. Omna Dair
B/9, shree manikanda, near ayyapa temple, vasai(w)
9860127163/9220575660/02502380545
TUTORIALS FOR BMS.

It is not Where We Reach, but more importantly How we reach mattersGo on- 1003 -.

A Management Paradise (un) Ltd .Endeavor.I,MP

Ms Monica Jain
19 Rupaytan Bldg, above ICICI Bank, Irla S V Road, Vile Parle W, Mumbai
400056
Contact no. 26715067/26710986/9869318083
TUTORIALS FOR BMS.

Vinayak Tutorials
Fy.j.c to Tyb.com, CA, Icwa, Mcom, CFA, CFP, MBA
shop no 9 Gokul bldg near movietime theatre on link road after dmart
9892204980

Prof.IYER.R.S
Conducts private tuitions for BMS/MBA/BBA/BCOM
CAT/GMAT/CET/MAT.
CPT/CA Inter/CF
[Experience: 17 years]
Contact no.9323040686

It is not Where We Reach, but more importantly How we reach mattersGo on- 1004 -.

A Management Paradise (un) Ltd .Endeavor.I,MP

ASPA Gurukool
Bms/B&I/BFM
Dombivali
Contact no.9833933025/26/27

BAF/BBI/BMS
www.jmdtutorials.com
Deepak and Prakash fulwadya
(many branches)
9820356676 ,9819028266
9820797729,9967008172

Devashis pvt tutorials(Andheri)


Devang sir-

Specially for BMS students


Mr Dikhil- 9920938950
(Santacruz)

It is not Where We Reach, but more importantly How we reach mattersGo on- 1005 -.

A Management Paradise (un) Ltd .Endeavor.I,MP

Janis Personal Coaching


Mr Mahesh-9820256030
35 Hari sadan,
3 flr prathana samaj,
Mumbai-4 charni road
TUTORIALS FOR BMS.

Mr Mukund-67305941
Jogershwari
TUTORIALS FOR BMS.

Kaila Classes
Andheri (W)
Prof. Kaila-9820136223
TUTORIALS FOR BMS.

Prof. Amey-9869192201
Worli
( Specially for BMS)

It is not Where We Reach, but more importantly How we reach mattersGo on- 1006 -.

A Management Paradise (un) Ltd .Endeavor.I,MP

Rashmin sir
Kandivali E
Contact no.9324951243
TUTORIALS FOR BMS.

Gerrys Classes- Chembur


Mr Gerald Dcunha-9833997266
Ground FloOr, building Do 1,
Tolaram Dagar,chembur Colony,
Chembur E, Mumbai 400074
TUTORIALS FOR BMS.

Vishal Mehta and Mihir Mehta


9920400029/9833155355
Specially for BMS students
[EXP: 6 yrs]
Special Features:
Free test series conducted
Affordable fees
Khanna classes
MCOM BMS MBA CA B&I
Shop no.26,
Sindhi society,
near pink corner
Chembur
Phone no. 9820976738

It is not Where We Reach, but more importantly How we reach mattersGo on- 1007 -.

A Management Paradise (un) Ltd .Endeavor.I,MP

Knowledge Junction
Mr Amit Kapadia-25025825

journeyofeducation@yahoo.co.in
5 Vidyavihar Building, Behind Standard Charted Bank, Derasar
Lane, Ghatkopar E, Mumbai - 400075
PRIVATE TUTIODS FOR BMS ADD CA

Perfect Coaching Institute


32684128, 9870140530
Plot 15/133, Opposite Laxmi Shopping Centre Narayan Nagar, L B S
Marg, Ghatkopar W, Mumbai - 400084
GROUP TUTIODS FOR BMS

Brilliant Classes

Mr Ganesh 9869110769,9833074427

H/103 Kalpita Society, Sahar Road, Andheri E, - 400059


TUTORIALS FOR BMS

Raj Tutorials
For BMS, BAF & BBI Studies

By

Prof. Raj

M. Com; FCA; Grad. CWA; ICS

 98211 33788 & 2612 1511


: Best In The Town :

It is not Where We Reach, but more importantly How we reach mattersGo on- 1008 -.

A Management Paradise (un) Ltd .Endeavor.I,MP

BMS COACHING CLASSES


Do You Run BMS COACHING CLASSES?
Want to Include your Coaching classes detail in this book ?
Reach your Audience !
Contact us on bizdev@managementparadise.com

It is not Where We Reach, but more importantly How we reach mattersGo on- 1009 -.

You might also like